You are on page 1of 220

Kinematics .....................................................................................................................................................

2
Introduction to Motion in One Dimension ............................................................................................... 2
Motion under Gravity ............................................................................................................................. 11
Motion in Two Dimensions ..................................................................................................................... 22
Circular Motion ....................................................................................................................................... 35
Relative Motion....................................................................................................................................... 41
Solved Examples on Mechanics:- ............................................................................................................ 49
Newtons Laws of Motion:- ......................................................................................................................... 63
Classification of Forces:- ......................................................................................................................... 64
Newtons First Law of Motion:- .............................................................................................................. 67
Newtons Second Law of Motion:-.......................................................................................................... 69
Newtons Third Law of Motion:- ............................................................................................................. 77
Friction:- .................................................................................................................................................. 90
Free body diagram .................................................................................................................................. 97
Centripetal Force .................................................................................................................................. 108
Conservation of Linear Momentum:- ....................................................................................................... 130
Work, Energy and Power:- ........................................................................................................................ 138
Work:- ................................................................................................................................................... 139
Energy (E):- ............................................................................................................................................ 147
Power:- .................................................................................................................................................. 174
Gravitation ................................................................................................................................................ 191
Gravitational Field and Intensity ........................................................................................................... 194
Satellites and Planetary Motion............................................................................................................ 206
Fluid Mechanics ........................................................................................................................................ 213
Thermal Physics:- ...................................................................................................................................... 218

Kinematics
Introduction to Motion in One Dimension
A body:A certain amount of matter limited in all directions and consequently having a finite size, shape and occupying some definite space
is called a body.
Particle:A particle is defined as a portion of matter infinitesimally small in size so that for the purpose of investigation, the distance between
its different parts may be neglected. Thus, a particle has only a definite position, but no dimension. In the problems we are going to
discuss, we will consider a body to be a particle for the sake of simplicity.
Motion:The position of object can change on a straight line (like on
x-axis with respect to origin) or on a plane with respect to
some fixed point on frame. So we can define motion as
follows:An object or a body is said to be in motion if its position continuously changes with time with reference to a fixed point (or fixed
frame of reference).
But note that, the moving object is either a particle, a point object (such as an electron) or an object that moves like a particle. A
body is said to be moving like if every portion of it moves in the same direction and at the same rate.
Motion in one dimension:When the position of object changes on a straight line i.e. motion of object along straight line is called motion in one dimension.
To understand the essential concepts of one dimensional motion we have to go through some basic definitions.
Frame of reference:One can see the platform from a running train, and it seems that all the objects placed
on platform are continuously changing their position. But one, who is on platform,
concludes that the objects on the platform are at rest. It means if we will take the trains
are reference frame the objects are not stationary and taking reference frame as
platform the objects are stationary. So the study of motion is a combined property of
the object under study and the observer. Hence there is a need to define a frame of
reference under which we have to study the motion of an object.

Definition
A frame of reference is a set of coordinate axes which is fixed with respect to a space point (a body or an object can also be treated
as a point mass therefore it can become a site for fixing a reference frame), which we have arbitrarily chosen as per our observer's
requirement. The essential requirement for a frame of reference is that, it should be rigid.
Position of an object
The position of an object is defined with respect to some frame of reference. As a convention, we define position of a point
(essentially we treat body as a point mass) with the help of three co-ordinates X, Y and Z. Hence X, Y, Z is a set of coordinate axes

representing a 3-dimensional space and each point in this space can be uniquely defined with the help of a set of X, Y and Z
coordinate, all three axes being mutually perpendicular to each other. The line drawn from origin to the point represents the position
vector of that point.
Position vector
It describes the instantaneous position of a particle with respect to the chosen frame of reference. It is a vector joining the origin to
the particle. If at any time, (x, y, z) be the Cartesian coordinates of the particle then its position vector is given by vector

= xi + yj

+ zk.
In one-dimensional motion: vector

= xi,

y = z = 0 (along x-axis)

In two-dimensional motion: vector

= xi + yj

(in x-y plane z = 0)

In the figure above, the position of a point P is specified and vector OP is called the position vector.

Displacement
Consider a case in which the position of an object changes with time. Suppose at certain instant 't' the position of an object is x1 along the x axis
and some other instant 'T' the position is x1 then the displacement x is defined as,
x = x2 - x1

It can be seen in the figure above where x1 and x2 are instantaneous position of the object at that time.
Now consider the motion of a point A with respect to a reference point O. The motion of point A
makes its radius vector vary in the general case both in magnitude and in direction as shown in
figure above. Suppose the point A travels from point 1 to point 2 in the time interval t. It is seen
from the figure that the displacement vector
vector

in time t:
2

of the point A represents the increment of

Difference between distance and displacement


To understand the difference between distance and displacement, we study the motion of vertical throw of a ball

with

respect to point O, as shown in the left figure, to height h.


After some time it will come again to the same point O. The displacement of ball is zero but there is some
distance traversed by the ball. It's because distance is a scalar quantity but displacement is a vector quantity.
Uniform and Non Uniform Motion

Speed is the rate of change of distance without regard to directions. Velocity is the rate at which the position vector of a particle
changes with time. Velocity is a vector quantity whereas speed is scalar quantity but both are measured in the same unit m/sec.
The motion of an object may be uniform or non-uniform depending upon its speed. In case of uniform motion the speed is constant,
whereas in the non-uniform motion, the speed is variable.
In uniform motion in one dimension the velocity (v) is mathematically defined as
v = (x2 - x1)/(T-t)

...... (1)

Where x1 and x2 are instantaneous displacement as shown in figure above at time 't' and 'T' respectively.
Graphical representation of the uniform motion
Form the equation (1) we have the following equation

x2 = x1 + v(T - t)
where v is constant. Take t = 0, the equation becomes x2 = x1 + vT, from this equation it follows that the graph of position of object
'x2' against 'T' is a straight line, cutting off x1 on the position axis where x1 is the distance of the particle from the origin at time t = 0.
v = slope of the graph which is constant

If the graph is not a straight line, it will represent non-uniform motion.

Motion of a body cannot be correctly identified unless one knows the position of body as specified by a
fixed frame of reference.

Velocity Vector in Non Uniform Motion


In any non-uniform motion, we can define an average velocity over a time interval. Average
velocity

is the ratio of the displacement x (that occurs during a particle time interval t)

to that interval of time i.e.

Now refer to the example, related to figure 2.3, the ratio of


direction of the vector <

> coincides with that of

/t is called the average velocity <

> during the time interval t. The

. Average velocity is also a vector quantity.

Note: The ratio of total distance traveled and time taken during the motion is called average speed. Average speed is a scalar
quantity.
If

at

any

time

t1 position

vector

of

the

particle

is

1 and

at

time

2 position

vector

is

2 then

for

this

interval
Instantaneous velocity
Instantaneous velocity is defined as the rate of change of displacement.

Velocity
The velocity at any instant is obtained from the average velocity shrinking the time interval closer to zero. As t tends to zero, the
average velocity approaches a limiting value, which is the velocity at that instant, called instantaneous velocity, which is a vector
quantity, mathematically we can define it as

The magnitude v of the instantaneous velocity is called the speed and is simple the absolute value of
In the example related with figure given below, the instantaneous velocity is

Hence instantaneous velocity is the rate at which a particle's position is changing with respect to time at a given instant. The velocity
of a particle at any instant is the slope (tangent) of its position curve at the point representing that instant of time, as shown in figure
above.
Speed
Speed is defined as rate of change of distance with time.
In any interval of time, average speed is defined as
<speed> = (total distance)/(total time taken) = s/t. As s > |
Think

(i)Can

body

have

constant

|, hence <speed> > <velocity>


speed

(ii)Can a body have a constant velocity and still have a varying speed?

and

still

have

varying

velocity?

The dispalcement remains unaffected due to shifting of origin from one point to the other.

The displacement can have positive, negative or zero value.

The dispalcement is never greater than the actual distance travelled.

The displacement has unit of length.

Velocity can be considered to be a combination of speed and direction.

A change in either speed or direction of motion results in a change in velocity.

It is not possible for a particle to possess zero speed with a non-zero velocity.

A particle which completes one revolution, along a circular path, with uniform speed is said to possesss zero velocity and nonzero speed.

In case a body moves with uniform velocity, along a straight line, its average speed is equal to its instantaneous speed.

Acceleration
Acceleration is the rate of change of velocity with time. The concept of acceleration is understood in non-uniform motion. It is a
vector quantity.
Average acceleration is the change in velocity per unit time over an interval of time.

Instantaneous acceleration is defined as

Acceleration vector in non uniform motion

Suppose that at the instant t1 a particle as in figure above, has velocity


acceleration

during the motion is defined as

and at t2, velocity is

. The average

Variable Acceleration
The acceleration at any instant is obtained from the average acceleration by shrinking the time interval closer zero. As t tends to
zero average acceleration approaching a limiting value, which is the acceleration at that instant called instantaneous acceleration
which is vector quantity.

i.e. the instantaneous acceleration is the derivative of velocity.


Hence instantaneous acceleration of a particle at any instant is the rate at which its velocity is changing at that instant.
Instantaneous acceleration at any point is the slope of the curve v (t) at that point as shown in figure above.
Equations of motion
The relationship among different parameter like displacement velocity, acceleration can be derived using the concept of average
acceleration and concept of average acceleration and instantaneous acceleration.
When acceleration is constant, a distinction between average acceleration and instantaneous acceleration loses its meaning, so we
can write

where is the velocity at t = 0 and

is the velocity at some time t.

So,

.................. (2)
This is the first useful equation of motion.
Similarly for displacement
.................. (3)
in which
of particle is

is the position of the particle at t0 and

is the average velocity between t0 and later time t. If at t0 and t the velocity

This is the second important equation of motion.


Now from equation (2), square both side of this equation we get

This is another important equation of motion.


Note:

The equation of motion derived above are possible only in uniformly accelerated motion i.e. the motion
in which the acceleration is constant.

Graphical Representation and Equations of Motion


While studying motion of bodies we have to keep two things in mind. We have to study various characteristics associated with
moving bodies while we are to explore and study in detail, about the cause of producing motion. Kinematics is the branch of physics
which deals only with the description of motion of bodies. Mainly there are two type of graphs are belonging to the kinematics. One
is position-time graph and the other is velocity-time graph.
These graphs will have validity only if motion under study is along a straight line. Then, displacement, velocity and
acceleration vectors are collinear and can be treated as algebraic quantities. Let x-axis be the path of motion. Then, x-coordinate
represents the magnitude of position vector.
Position - Time Graph:-

If we plot time t along the x-axis and the corresponding position (say x) from the origin O on the y-axis, we get a graph which is
called the position-time graph. This graph is very convenient to analyse different aspects of motion of a particle. Let us consider
the following cases.

(i) In this case, position (x) remains constant but time changes. This indicates that the particle is stationary in the given reference
frame. Hence, the straight line nature of position-time graph parallel to the time axis represents the state of rest. Note that its slope
(tan ) is zero.
.

(ii) When the x-t graph is a straight line inclined at some angle () with the time axis, the particle traverses equal displacement x
in equal intervals of time t. The motion of the particle is said to be uniform rectilinear motion. The slope of the line measured by
x/t = t represents the uniform velocity of the particle.

(iii)

When the x-t graph is a curve, motion is not uniform. It either speeds up or slows down depending upon whether the slope (tan

successively increases or decreases with time. As shown in the figure the motion speeds up from t = 0 to t=t1 (since the slope tan
increases). From t=t1 to t=t2, AB represents a straight line indicating uniform motion. From t=t2 to t=t3, the motion slows down and
for t>t3 the particle remains at rest in the reference frame.
The Velocity - Time Graph:The velocity-time graph gives three types of information.
(i) The instantaneous velocity.
(ii) The slope of the tangent to the curve at any point gives instantaneous
acceleration.
a = dv/dt = tan
(iii)

The area under the curve gives total displacement of the particle.

Now, let us consider the uniform acceleration. The velocity-time graph will be a
straight line.

The acceleration of the object is the slope of the line CD.


a = tan = BC/BD = (v-u)/t
v = u +at

... (1)

The total displacement of the object is area OABCD


s = Area OABCD = Area of rectangle OABD + Area of triangle BCD
s = ut + (1/2)at2

...

(2)

Again, s = Area OABCD


= 1/2(AC + OD) x OA = 1/2(v + u)xt
The acceleration-time graph:Acceleration time curves give information about the variation of acceleration with time. Area under the acceleration time curve gives
the change in velocity of the particle in the given time interval.

Refer this simulation that shows a car traveling along a graph:The purpose is to show that, the car can traverse the graph, then it's first derivative is a continuous function.

The displacement can have positive, negative or zero value.

The displacement is never greater than the actual distance travelled.

A change in either speed or direction of motion results in a change in velocity.

It is not possible for a particle to possess zero speed with a non-zero velocity.

The velocity-time graph for a uniformly accelerated body is a straight line.

Motion under Gravity


Whenever a body is released from a height, it travels vertically downward towards the surface of earth. This is due to the force of
gravitational attraction exerted on body by the earth. The acceleration produced by this force is called acceleration due to
gravity and is denoted by g. Value of g on the surface of earth is taken to
the 9.8 m/s2 and it is same for all the bodies. It means all bodies (whether an
iron ball or a piece of paper), when dropped (u=0) from same height should fall
with same rapidity and should take same time to reach the earth. Our daily
observation is contrary to this concept. We find that iron ball falls more rapidly
than piece of paper. This is due to the presence of air which offers different
resistance to them. In the absence of air both would have taken same time to
reach the surface of earth.
When a body is dropped from some height (earth's radius = 6400 km), it falls freely under gravity with constant acceleration g (= 9.8
m/s2) provided the air resistance is negligible small. The same set of three equations of kinematics (where the
acceleration

remains constant) are used in solving such motion. Here, we replace

by

conveniently. When the y-axis is chosen positive along vertically downward direction, we take

and choose the direction of y-axis


as positive and use the equation

as

v = u + gt, v2 = u2 + 2gh, and h = ut + 1/2gt2


where h is the displacement of the body and u is initial velocity of projection in the vertically downward direction. However, if an
object is projected vertically upward with initial velocity u, we can take y-axis positive in the vertically upward direction and the set of
equations reduces to
v = u - gt, v2 = u2 - 2gh, and h = ut - 1/2gt2
In order to avoid confusion in selecting

as positive or negative, it is advisable to take the y-axis as positive along vertically upward

direction and point of projection as the origin. We can now write the set of three equations in the vector form:

and

where h is the displacement of the body.


Illustration:
The motion of a particle is described by the equation u = at. The distance travelled by the particle in the first 4 second.
Solution:
Because for the motion u = at. So acceleration is uniform which is equal to a.
Therefore, Distance traveled = 1/2[(a)(4)2] = 8a
Illustration:
A body moving with a constant retardation in straight line travels 5.7 m and 3.9 m in the 6th and 9th second, respectively. When will
the body come momentarily to rest?
Solution:
A body moving with initial velocity u and acceleration a, traverses distance Sn in nthsecond of its motion.
Sn = u + (1/2)(2n - 1)a

=> 5.7 = u + (1/2)(2 x 6 - 1) a

or 5.7 = u + (11/2) a
and 3.9 = u + (1/2)(2 x 9 - 1)a

or,

3.9 = u + (17/2) a

Solving eqns. (1) and (2) we get, u = 9 m/s and a = -0.6 m/s2.
If the body stops moving after t seconds, then from the relation v=u+at
Thus, 0 = 9 + (-0.6)t

or, t = (9/0.6)s = 15s

Illustration (JEE Advanced):


A stone, thrown up is caught by the thrower after 6s. How high did it go and where was it 4 s after start? g = 9.8 m/s 2
Solution:
Time to go up and come back = 6s
Thus, time to reach the highest point = (6/2) s = 3s
From point of projection to the highest point we have
u = ?, v = 0, a = -9.8 m/s2, t = 3s
Using the relation, v = u + at
0= u 9.8 3
Thus, u = 29.4 m/s2
Maximum height, H = u2/2g = [(29.4)2/2(9.8)] m =44.1 m
Let h = height of stone at 4s.

Using the relation, S = ut + at2


So, h = [(29.4)(4)-1/2 (9.8) (4)2]m
= [117.6-78.4] m = 39.2 m
From the above observation we conclude that, the height would be 39.2 m.

Acceleration of all these bodies is constant.

Acceleration is always directed downward.

All bodies, when dropped (u=0) from same height should fall with same rapidity and should take same time to reach the earth.

Motion in a Straight Line with Acceleration

Velocity of a body is defined as the time rate of displacement, where as acceleration is defined as the time rate of change
of velocity. Acceleration is a vector quantity. The motion may be uniformly accelerated motion or it may be non-uniformly
accelerated, depending on how the velocity changes with time.
Uniform acceleration
The acceleration of a body is said to be uniform if its velocity changes by equal amounts in equal intervals.
Non-uniform acceleration
The acceleration of a body is said to be non-uniform if its velocity changes by unequal amounts in equal intervals of time.
Average velocity

Average acceleration

Illustration:
A particle moves with a velocity v(t) = (1/2)kt2 along a straight line. Find the average speed of the particle in time T.
Solution:

Illustration:
A particle having initial velocity is moving with a constant acceleration 'a' for a time t.
(a)Find the displacement of the particle in the last 1 second.

(b)Evaluate it for u = 2 m/s, a = 1 m/s2 and t = 5 sec.


Solution:
(a) The displacement of a particle at time t is given s = ut + 1/2at2
At time (t - 1), the displacement of a particle is given by
S' = u (t-1) + 1/2a(t-1)2
So, Displacement in the last 1 second is,
St = S - S'
= ut + 1/2 at2 [u(t-1)+1/2 a(t-1)2 ]
= ut + 1/2at2 - ut + u - 1/2a(t - 1)2
= 1/2at2 + u - 1/2 a (t+1-2t) = 1/2at2 + u - 1/2at2 - a/2 + at
S = u + a/2(2t - 1)
(b) Putting the values of u = 2 m/s, a = 1 m/s2 and t = 5 sec, we get
S = 2 + 1/2(2 x 5 - 1) = 2 + 1/2 x 9
= 2 + 4.5 = 6.5 m
Illustration:
Position of a particle moving along x-axis is given by x = 3t - 4t2 + t3, where x is in meters and t in seconds.
(a)Find the position of the particle at t = 2 s.
(b)Find the displacement of the particle in the time interval from t = 0 to t = 4 s.
(c)Find the average velocity of the particle in the time interval from t = 2s to t=4s.
(d)Find the velocity of the particle at t = 2 s.
Solution:
(a) x(t) = 3t - 4t2 + t3
=> x(2) = 3 x 2 - 4 x (2)2 + (2)3 = 6 - 4 x 4 + 8 = -2m.
(b) x(o) = 0
X(4) = 3 x 4 - 4 x (4)2 + (4)3 = 12 m.
Displacement = x(4) - x(0) = 12 m.
(c) < v > = X(4)X(2)/(4-2) = (12-(-2))/2 m/s = 7 m/s
(d) dx/dt = 3 - 8t + 3t2
v(2) (dx/dt)2 = 3 - 8 x 2 + 3 x (2)2 = -1m/s
Illustration:
Two trains take 3 sec to pass one another when going in the opposite direction but only 2.5 sec if the speed of the one is increased
by 50%. The time one would take to pass the other when going in the same direction at their original speed is
(a) 10 sec

(b) 12 sec

(c) 15 sec

(d) 18 sec

Solution:
Using the equation,

t = d/vr
We have,
3 = d/v1+v2
2.5 = d/1.5v1+v2
Solving we get,
v1 = 2d/15 and v2 = d/5
When they are going in same direction,
vr = v2 v1 = d/15
Thus, t = d/vr = d/(d/15) = 15 s
From the above observation we conclude that, option (c) is correct.
Analysis of Uniformly Accelerated Motion

Case-I:
For uniformly accelerated motion with initial velocity u and initial position x0.
Velocity time graph

In every case tan = a0


Position time graph

Initial position x of the body in every case is x0 (> 0)


Case II:

For uniformly retarded motion with initial velocity u and initial position x0.
Velocity time graph

In every case tan = -a0


Position time graph

Initial position x of the body in every case is x0 (> 0)


Illustration:

A particle is moving rectilinearly with a time varying acceleration a = 4 - 2t, where a is in m/s2 and t is in sec. If the particle is starting
its motion with a velocity of -3 m/s from x = 0. Draw a-t, v-t and x-t curve for the particle.
Solution:
a = 4-2t

v = 4t-t2-3

x = 2t2 t3/3 3t
Acceleration
Acceleration is the rate of change of velocity with time. The concept of acceleration is understood in non-uniform motion. It is a vector
quantity.

Average acceleration is the change in velocity per unit time over an interval of time.

Instantaneous acceleration is defined as

Acceleration vector in non uniform motion

Suppose that at the instant t1 a particle as in figure above, has velocity


acceleration

during the motion is defined as

and at t2, velocity is

. The average

Variable Acceleration

The acceleration at any instant is obtained from the average acceleration by shrinking the time interval closer zero. As t tends to zero average
acceleration approaching a limiting value, which is the acceleration at that instant called instantaneous acceleration which is vector quantity.

i.e. the instantaneous acceleration is the derivative of velocity.


Hence instantaneous acceleration of a particle at any instant is the rate at which its velocity is changing at that instant.
Instantaneous acceleration at any point is the slope of the curve v (t) at that point as shown in figure above.
Equations of motion
The relationship among different parameter like displacement velocity, acceleration can be derived using the concept of average
acceleration and concept of average acceleration and instantaneous acceleration.
When acceleration is constant, a distinction between average acceleration and instantaneous acceleration loses its meaning, so we
can write

where

is the velocity at t = 0 and

is the velocity at some time t

Now,

Hence,
... (2)
This is the first useful equation of motion.
Similarly for displacement
... (3)
in which

is the position of the particle at t0 and

velocity of particle is

is the average velocity between t0 and later time t. If at t0 and t the

... (4)
From equation (3) and (4), we get,

... (5)
This is the second important equation of motion.
Now from equation (2), square both side of this equation we get

[Using equation (4)]


Using equation (3), we get,
... (6)
This is another important equation of motion.
Caution:The equation of motion derived above are possible only in uniformly accelerated motion i.e. the motion in which the
acceleration is constant.
Refer this simulation for motion in a straight line:Illustration:
The nucleus of helium atom (alpha-particle) travels inside a straight hollow tube of length 2.0 meters long which forms part of a
particle accelerator. (a) If one assumes uniform acceleration, how long is the particle in the tube if it enters at a speed of 1000
meter/sec and leaves at 9000 meter/sec? (b) What is its acceleration during this interval?
Solution:
(a) We choose x-axis parallel to the tube, its positive direction being that in which the particle is moving and its origin at the tube
entrance. We are given x and vx and we seek t. The acceleration ax is not involved. Hence we use equation 3, x = x0 + <v> t.
We get
x = v0 + (vx0) + vx) t, with x0 = 0 or
t = 2x/(vx0+vx),
t = ((2)(2.0 meters))/((1000+9000)meters/sec) = 4.0/10-4 sec

Ans.

(b) The acceleration follows from equation 2, vx = vx0 + axt


=> ax = (v0-vx0)/t = ((9000-1000)meters/sec)/(4.010(-4) sec)
= 2.0 107 meter/sec2 Ans.
Pause: The above equations of motion are, however, universal and can be derived by using differential calculus as given below:

Or,
Let at t = 0,
then,
Or,
Further we know that,

or
Integrating,

Or,

At, t = 0, x = x0 then c' = x0


Hence,

Thus, we have derived the same equation of motion using calculus.


To understand the use of calculus in solving the kinematics problems we can look into the following illustrations.
Illustration:
The displacement x of a particle moving in one dimension, under the action of a constant force is related to the time t by the
equation t = x + 3 where x is in meter and t is in seconds. Find the displacement of the particle when its velocity is zero.
Solution:
Here t = x + 3 => x = t - 3
Squaring both sides, we get x = t2 - 6t + 9,
As we know velocity, v = dx/dt
Hence we get v = dx/dt = 2t - 6
Put v = 0, we get, 2t - 6 = 0
So, t = 3s

When t = 3s, x = t2 - 6t + 9 = 9 - 6(3) + 9 = 0


Hence the displacement of the particle is zero when its velocity is zero.
Illustration:
A particle starts from a point whose initial velocity is v1 and it reaches with final velocity v2, at point B which is at a distance 'd' from
point A. The path is straight line. If acceleration is proportional to velocity, find the time taken by particle from A to B.
Solution:
Here acceleration a is proportional to velocity v.
Hence a v
=> a = kv, where k is constant
=> dv/dt = kv ............... (1)
=> (dv/ds)(ds/dt) = kv => (dv/ds) v = kv

From equation (1),


dv/v = kdt

or,
Or, ln (v2/v1) = kt
Or, t = ln (v2/v1) /k
= [d ln (v2/v1)/(v2-v1)]

The dispalcement remains unaffected due to shifting of origin from one point to the other.

The displacement can have positive, negative or zero value.

The dispalcement is never greater than the actual distance travelled.

The displacement has unit of length.

Velocity can be considered to be a combination of speed and direction.

A change in either speed or direction of motion results in a change in velocity.

It is not possible for a particle to possess zero speed with a non-zero velocity.

A particle which completes one revolution, along a circular path, with uniform speed is said to possesss zero velocity and non-zero speed.

In case a body moves with uniform velocity, along a straight line, its average speed is equal to its instantaneous speed.

Motion in Two Dimensions


In this part, we discuss motions in two dimensions like the motion of a particle moving on a circular path or on a parabolic path.
Consider a particle moving on x-y plane along a curved pat at time t, as shown in figure given below. Its displacement from origin is
measured by vector

its velocity is indicated by vector

(tangent to the path of the particle) and acceleration

as shown in

the figure.

The vectors

and

are inter related and can be expressed in terms of their components, using unit vector notation as,

From the above relations we can say that two-dimensional motion is nothing but superimpose of two one dimension motions. This is
for the reason that in any two-dimensional motion, all the parameters can be resolved in two mutually perpendicular directions.
Frame of reference:-

A frame of reference is a set of coordinate axes which is fixed with respect to a space point (a body or n object can also be treated
as a point mass and therefore it can become a site for fixing a reference frame), which we have arbitrarily chosen as per our
observer requirement. It is constituted of two components:
(i) Body on which observer is apparently sitting to observe the motion, i.e. the origin.
(ii) A coordinate system fixed on this body so that whatever is being observed can be measured or mathematically
determined with the help of the coordinates of space points, which are defined by a unique set of (x, y, z). To fix a frame of

reference one has to fix the origin to a chosen space point and then fix the co-ordinate system on it. Once the origin and coordinate
system are fixed, then we have done the exercise of fixing the frame of reference and we can proceed to solve problems and study
various parameters i.e. position, velocity, accelerations as functions o time.
Here, an important thing to notice is that earth becomes a natural choice for the frame of reference as in most of the cases it is very
easy to visualize the motion with respect to it. As it is a routine in our daily life we innocently (even in early childhood) refer and
perceive all motion phenomenon with respect to earth only.
While analyzing any motion we will take the help of the coordinate system. What we shall do is that we shall keep ourselves free to
take it there. This is called a frame of reference because we shall refer to all positions in this coordinate system fixed on that body.
Here,

remembers

that

we

fix

all

three

axis

of

coordinate

system

i.e.

X-axis. Y-axis and Z-axis as well as the origin on the body and the coordinates of origin are (0, 0, 0). It is the origin where we mostly
assume that the observer is sitting to observe the motion in the concerned reference frame.
Choice of a frame of reference:Let us come back to the concept of motion. Do you believe that all what you see
moving is in motion and what you see not moving is at rest! Like vehicles on road!
You should be warned that motion or observation of motion is really not that
simple!
Let us take an example: The moon in the night sky! Some time at night you see
that moon is travelling across the clouds towards east or west. And in some other nights it doesn't move at all! We shall all agree
that it cannot be. It cannot sometimes move faster and sometime slower and sometimes become stationary. Then, what is the
phenomenon there?
Motion in Two Dimensions:We start with the first point that whenever we speak of any motion it
should be with respect to some fixed frame of reference. So, when
we see the moon moving. It is not with certainly one can point out
that in the night when it looked stationary there were no clouds. So it
is an effect of the clouds. The fact is that if we have clouds in some
nights moving towards west or east then we see moon travelling
towards east or west in the sky respectively. How to explain it with the help of reference frames? It is like this. When we see two
objects in the sky one moon and other a cloud, then due to relatively bigger size of clouds (as seen by us) one gets his eyes fixed
on clouds and therefore frame of reference gets fixed on the clouds (Figure shown above). It is equivalent to say that we are sitting
on the clouds itself, which looks stationary to us and the feeling comes, as if moon is travelling across this stationary cloud in the
opposite direction to the clods motion (see figure given above). Similarly, we see things going backwards when we travel on a road.
Now let us deliberately fix our eyes on the moon and see. What we are doing now is that we are fixing frame of reference on the
moon. It is equivalent to say that we are sitting on the moon to observe the cloud's motion and now we can see clouds drifting away
(which is the fact). See Figure shown above.

An important observation to remember is that once we choose an object as a frame of reference, we can't see it moving with respect
to itself. Naturally, in most of the cases of observations the Earth becomes a natural choice for the reference frame. Also, based on
the choice of reference frames one can have many apparent motion of an object under consideration.
Here one important point to be noted is that in all previous discussion we have assumed that the reference frames are not
accelerating, which means it's acceleration is zero. These are called inertial or un accelerated frames of reference. Now
obviously, we are left with one more choice of class of reference frames that is the accelerated ones. These are called
non-inertial or accelerated frame of reference. But in all the forthcoming exercises in this chapter we shall deal with and look only
for inertial frames of reference.

Relative motion:Consider a observer S is fixed to the earth. The other observer S' is moving on the earth, say a passenger sitting on a bus. Hence
other frame of reference S' is fixed with bus. Each observer is studying the motion of a cyclist in his or her own frame of reference.
Each observer will record a displacement, velocity and acceleration for the cyclist measured relative to his reference frame. Now the
question is "How will we compare their results"?. To do this we need the concept of relative motion. Let us do it.
In figure shown above at the top, the reference frame S, represented by the x and y-axes, can be thought of as fixed to the earth.
The other reference frame S', represented by x'- and y'-axes, which moves along the x-axis with a constant velocity u, measured in
the s-system, fixed with the bus.
Initially, a particle is at a point called A in the S-frame and called A' in the S'-frame. At a time t later, the reference frame S' has
moved a distance ut to the right and particle has moved to B. The displacement of the particle from its position in the S-frame is the
vector

from A to B. The displacement of the particle from its initial position in the S'-frame is the vector r' from A' to B. These are

different vectors because the reference point A' of the moving frame has been displaced a distance ut along the x-axis during the
motion. From the second figure shown above we see that
=

is the vector sum of

Differentiating equation leads to

But,

is the instantaneous velocity of the particle measured in the


s-frame.
and

is the instantaneous velocity of the same particle measured in the S' frame, so that
=

'+

.................. [A]

and

Hence the velocity of the particle relative to the S-frame,


the velocity

is the vector sum of the velocity of the particle to the S' frame,

, and

of the S' frame relative to the S-frame.

Once again differentiating equation [A]


We get,

Hence the acceleration of particle is the same in all reference frame moving relative to one another with constant velocity

The relative velocity between S and S' must be a constant.

At any instant, the velocity the velocity of a particle as measured by S is equal to the velocity of the particle as measured by S'.

Law of transformation of velocity permits us to transform a measurement of velocity made by an observer in one frame reference to another
frame of reference.

The reference frames which are not accelerating are called inertial frame of reference.
Problem 1:A particle starts with initial velocity (3i + 4j) m/s and with constant acceleration (8i + 6j) m/s2. Find the final velocity and final
displacement of the particle after time t = 4 sec.
Solution:As the velocity and acceleration vectors are not parallel, the velocity and acceleration cannot be treated as a scalar.
Now divide the whole phenomenon in two one-dimension motions.
In x-direction,
Initial velocity = vx0 = 3m/s, ax = 8 m/s2

Hence velocity after 4 second, vx = 3 + 8 4 = 35 m/s


Displacement after 4 second sx = x = 3 4 + 8 (4)2 = 76 m
In y-direction,
Initial velocity vy0 = 4m/s, ay = 6 m/s2
Velocity after 4 second vy = 4 + 6 4 = 28 m/s
Displacement after 4 second sy = 4 4 + 6 (4)2 = 64 m

It means final velocity v = vx i + vy j = (35 i + 28 j) m/s


and displacement = s sx i + sy j = (76 i + 64 j) m

Problem 2:-

There are two boats B1 and B2. They are continuously moving towards a bank. Water flow velocity is 10m/sec away from the bank.
When at one instant of time, observation was taken we found that B1 and b2 are at a position (figure is gen below) which is 100 m
from the bank and after 1 sec, b1 is 50 away and B2 is 25m away from the bank. Find out all possible (motion) velocities observed
from different reference frames choices.
Solution:Here we have four (at least) immediate choices of reference frames with respect to which we can observe velocities.
(i)

Bank (earth)

(ii)

B1

(iii)

B2

(iv) Water

(i) Bank: (See figure shown above) First we shall fix the frame of reference on the bank.
Since B1 travels 50 m towards the bank in 1 sec., its velocity towards the Bank (that is in +y direction) is 50m/sec.
Similarly B2 travels 75 m/s towards the bank
Thus, its velocity with respect to bank or as observed by an observer sitting at rest on the bank is
75m/sec (+y direction)
Since water moves away from bank at speed of 10m/sec.
Therefore, water's velocity w.r.t. the bank is 10m/sec (-y direction).
(ii) Boat B1: Now, let us fix frame of reference on Boat B1.
See the right side figure:

(a) Since B1 cannot move w.r.t. itself (only in case of deformation, a part of this boat can be seen moving w.r.t. other parts)
Thus, VB1 = 0 (relative velocity w.r.t. to itself)
(b) at t = 0, B1 and B2 were in the same line and had no
at t = 1 B2 leads B1 by 25 m
that is B2 has moved 25m in 1 sec w.r.t. B1
Thus, VB2 = 25 m/sec (+ direction)
(c) Bank, at t = 0 was away by 100 m.
and at t = 1 it has come closer by 50 m towards B1
Therefore, bank is approaching B1 by a speed of
Vbank = 50m/sec (-y direction)
(d) To visualize the motion of water w.r.t. B1 we can imagine a wooden piece of negligible mass, floating on the water surface. Its
velocity will be equal to that of water. At t = 0 the piece was in the same line joining B1 and B2 at a distance of 100 m from the bank.
At t = 1 sec it will move away by 10m from the bank that is its distance from b1 will be 60m (see figure shown above)
Following a similar approach find out various relative velocities as seen from frames of reference, which are fixed, on boat B2 (figure
shown below) and when the reference frame is fixed on water. (Figure shown next after this given below)

Problem 3:A windblown rain is filling, 20 degrees from the vertical, at 5 m/s. Passengers in a car see the rain falling vertically. What is the
magnitude and direction of car's velocity?
Solution:-

Here we have two frames S and S'. Frame S is attached with ground while S' attached with car. To determine the magnitude and
directions of car's velocity, we use the concept of relative motion discussed earlier.
Let vRG = velocity of rain with respect to ground
vRC= velocity of rain with respect to car
vc= velocity of car with respect to ground
From the equation for relative velocity in two frames, we get

From the vector diagram we see that car should be travelling along the inclined direction of rain with velocity
vC = 5 sin 20o m/s

Motion of Projectile

Now we discuss some example of curved motion or two dimensional motion of constant acceleration such as the motion of constant
acceleration such as the motion of a particle projected at certain angle with the horizontal in vertical x-y plane (this type of motion is
called projectile motion). Air resistance to the motion of the body is to be assumed absent in this type of motion.
A body projected into the space and is no longer being propelled by fuel is called a projectile.

To analyze the projectile motion we use the following concept "Resolution of two dimensional motion into two one dimension
motion" as discussed earlier. Hence it is easier to analyze the motion of projectile as composed of two simultaneous rectilinear
motions which are independent of each other:
(a) Along the vertical y-axis with a uniform downward acceleration 'g' and
(b) Along the horizontal x-axis with a uniform velocity forward.
Consider a particle projected with an initial velocity u at an angle with the horizontal x-axis as shown in figure shown below.
Velocity and accelerations can be resolved into two components:

Velocity along x-axis = ux = u cos


Acceleration along x-axis ax = 0
Velocity along y-axis = uy = u sin
Acceleration along y-axis ay = -g
Here we use different equation of motions of one dimension derived earlier to get the different parameters.
... (a)

... (b)
2

v =

v02

2g (y-y0)

... (c)

Total Time of flight:When body returns to the same horizontal level, the resultant displacement in vertical y-direction is zero. Use equation b.
Therefore, 0 = (u sin ) t - ()gt2,
As t cannot equal to zero, then, total time of flight,

Horizontal Range:-

Horizontal Range (OA=X) = Horizontal velocity Time of flight


= u cos 2 u sin /g
So horizontal range,

Maximum Height:At the highest point of the trajectory, vertical component of velocity is zero.
Therefore, 0 = (u sin )2 - 2g Hmax
So, maximum height would be,

Equation of Trajectory:Assuming the point of projection as the origin of co-ordinates and horizontal direction as the x-axis and vertical direction as the yaxis. Let P (x, y) be the position of the particle at instant after t second.
Then x = u cos .t and y = u sin .t - 1/2 gt2
Eliminating 't' form the above equations, we get,
y = x tan - (gx2/2u2cos2)
This is the equation of trajectory which is a parabola (y = ax + bx2).
Problem 1 (JEE Advanced):The speed of a projectile when it is at its greatest height is 2/5 times its speed at half the maximum height. Find out the angle of
projection.
Solution :Let be the angle of projection and u its initial speed. Then maximum height will be,

H = u2sin2/2g
So, gH = u2sin2/2
Now, vH = u cos
Or, vH2 = u2 cos2
vH/22

... (1)
2

= u -2g(H/2) = u -gH

vH/22=

u2-(u2sin2/2)

...(2)

Now it is given that,


vH = [2/5] vH/2
Or, vH2 = (2/5) vH/22
Substituting the values from equations (1) and (2), we get,
(u2 cos2) = 2/5 [u2-(u2sin2/2)]
Or, 5cos2 = 2[1-(sin2/2)]
Or, 5(1-sin2) = 2-sin2
Or, sin2 =
Or, sin = (3)/2, Or, = 60o
Thus from the above observation, we conclude that, the angle of projection would be 60o.
Problem 2:A gun moving at a speed of 30m/sec fires at an angle 30o with a velocity 150m/s relative to the gun. Find the distance between the
gun and the projectile when projectile hits the ground. (g = 10 m/sec)
Solution :Vertical component of velocity = 150 sin 30o = 75 m/sec
Horizontal component of velocity relative to gun = 150 cos 30o = 753 m/sec
Horizontal component of velocity relative to ground = 753 + 30 160m/sec
Time of flight = (2 75)/g = 15 s
Range of projectile = 160 15 = 2400 m
Distance moved by the gun and projectile = 2400 - 450 = 1950 m.
Horizontal projection:-

Consider a particle projected horizontally with a velocity

from a point O as shown in side figure.

Assuming the point of projection O as the origin of coordinates and horizontal direction as the X-axis and vertical direction as Y-axis.
Let P (x, y) be the position of the particle after t seconds.
So, x = horizontal distance covered in time t = ut.

............... (1)

y = vertical distance covered in time t = gt2

............... (2)

Eliminate t from equations (1) and (2) then we get,


y = (1/2)(g/u2) (x2)
This is the equation of parabola passing through the origin, with its vertex at the origin O. Hence the trajectory is a parabola.
Problem 3:-

A stone is thrown at a speed of 19.6 m/sec at an angle 30o above the horizontal from a tower of height 490 meter. Find the time
during which the stone will be in air. Also find the distance from the foot of the tower to the point where stone hits the ground?
Solution :Let us consider the motion of stone in the horizontal and vertical directions separately.
(i) Vertical motion (downward direction negative) :
Initial vertical velocity y = 19.6 sin 30o
Acceleration a = g = -9.8 m/s2
Vertical distance covered = h = 490 m
Using, h = ut + 1/2gt2
We have, 490 = - 9.8t + (1/2) 9.8t2
100 = - 2t + t2

or

t2 - 2t - 100 = 0

t = 11.05 sec
(ii) Horizontal motion:
Initial horizontal velocity y = 19.6 sin 30o = 9.8 m/s
Hence distance from the foot of tower to the point where stone hits the ground
= Horizontal component time of flight
= 19.6 cos 30o 11.05 = 188 m
Projectile Motion on an inclined plane:Let the particle strike the plane at A so that OA is the range of the projectile on inclined plane. This initial velocity
into two components:

can be resolved

(i) u cos ( - ) along the plane


(ii) u sin ( - ) perpendicular to the plane.
The acceleration due to gravity g can be resolved into two components:
(i) g sin parallel to the plane
(ii) g cos perpendicular to the plane.
Time of Flight:Let t be the time taken by the particle to go from A to B. In this time the displacement of the projectile to the plane is zero.
Hence, 0 = u sin (-) t - g t2
=> t = 2u sin(-)/gcos
Range:During time of flight, the horizontal velocity u cos remains constant.
Hence, horizontal distance,
OB = (ucos) t = 2u2sin(-)cos/gcos
Now, OA = OB/cos = 2u2sin(-)cos/gcos
The greatest distance of the projectile from the inclined plane is u2sin2 (-)/2gcos .
Problem 4:-

A Particle is projected with a velocity 39.2 m/sec at an angle of 30o to an inclined plane (inclined at an angle of 45o to the horizontal).
Find the range on the incline (a) when it is projected upward (b) when it is projected downward.
Solution :Time of flight will be same in both cases because the acceleration perpendicular to the plane is same. Therefore,
0 = 39.2 sin 30o t - () g cos 45o t2
Or, t = (239.2 sin 30)/(g cos 45) = 42 sec

(a) Range upward


= 39.2 cos 30o t - () g sin 30o t2
= 39.2 3/2 42- (1/2) 9.8 (1/2) (42)2 = 113.7m
(b) Range downward
= 39.2 cos 30o t + () g sin 30o t2
= 39.2 3/2 42 + (1/2) 9.8 (1/2) (42)2 = 270.5m Ans.
Motion down the plane:-

Let the particle be thrown at a velocity v0 at angle with the horizontal as shown in figure.
v0 sin (+)T- 1/2 gcosT2=0 [for y'=0]
=> T = (2v0 sin(+))/gcos
R = v0 cos(+ )T+ 1/2 g sin T2= (v02)/g [(sin(2+)+sin)/(1-sin2)]
Since is the variable and maximum value of sin function is 1, therefore for R to be maximum, sin (2+)=1
and Rmax (v02)/g [(1+sin)/(1-sin2)]= (v02)/(g(1-sin)) down the plane.

Every projectile experiences one single force and that is due to gravity only.

Horizontal velocity of a projectile remains the same throughout its flight (it may be zero also).

No projectile ever experiences any acceleration in the horizontal direction.

Vertical acceleration of every projectile is -g.

The path of projectile is parabolic except for those projected along vertical direction. In that case it is a straight line.

The horizontal and vertical motion of a projectile are independent of each other.

Circular Motion
Now we shall discuss another example of two-dimensional motion that is motion of a particle on
a circular path. This type of motion is called circular motion. The motion of a body is said to be
circular if it moves in such a way that its distance from a certain fixed point always
remains the same.
Direction of motion of body at any instant:If the string breaks suddenly, the stone shall fly tangentially to the path of motion. So,
instantaneous direction of motion of the body is always along the tangent to the curve at that point.
Consider a particle P is moving on circle of radius r on X-Y plane with origin O as centre.
The position of the particle at a given instant may be described by angle , called angular position of the particle, measured in
radian. As the particle moves on the path, its angular position changes. The rate of change of angular position is called angular
velocity, , measured in radian per second.

= d/dt = ds/rdt = v/r


The rate of change of angular velocity is called angular acceleration, measured in rad/s2. Thus, the angular acceleration is
= d/dt = d2/dt2
Relation between These Parameters:-

It is easy to derive the equations of rotational kinematics for the case of constant angular acceleration with fixed axis of rotation.
These equations are of the same form as those for on-dimensional transitional motion.
= 0 + t
............ (a)
= 0 + 0t + t2/2

............ (b)

2 = 02 + 2 ( - 0)

............ (c)

= 0 + (0 + )/(2t)

............ (d)

Here, 0 is the initial angle and 0 is the initial angular speed.


Problem 1:

(a)

What is the angular velocity of the minute and hour hands of a clock?

(b)

Suppose the clock starts malfunctioning at 7 AM which decelerates the minute hand at the rate of 4 radians/day. How much

time would the clock loose by 7 AM next day?


Solution:(a) Angular speed of,
minute hand : mh = 2 rad/hr = 48 rad/day = (/1800) rad/sec
hour hand : hh = (/6) rad/hr = 4 rad/day = (/21600) rad/sec
(b) Assume at t = 0, 0 = 0, when the clock begins to malfunction.
Use equation (ii) to get the angle covered by the minute hand in one day.
So, = 0(1 day) 1/2(1 day)2 = 46 rad
Hence the minute hand complete 23 revolutions, so the clock losses 1 hour.
Problem 2:
A particle is rotating in a circular path having initial angular velocity 5 rad/sec and the angular acceleration = 0.5 , where is
angular velocity at that instant. Find the angular velocity, after it moved an angle ?
Solution:Here angular acceleration is
= 0.5
=> d/dt = 0.5
=> (d/d) (d/dt) = 0.5
=> d/d = 0.5
Or,

=> - 5 = 0.5
=> = 5 + 0.5 = 6.57 rad/sec.
Hence, when acceleration is not constant, use the method of calculus as shown in above illustration.

Motion of a particle in a circular path:-

It is a special kind of two-dimensional motion in which the particle's position vector always lies
on the circumference of a circle. In order to calculate the acceleration parameter it is helpful to first consider circular motion with
constant speed, called uniform circular motion. Let there be a particle moving along a circle of radius r with a velocity
shown in figure given below, such that

, as

= v = constant. For this particle, it is our aim to calculate the magnitude and direction of

its acceleration. We know that,

Now, we have to find an expression for


points A and B. Displacing
=

in terms of known quantities. For this, consider the particle velocity vector at two

, parallel to itself and placing it back to back with

Consider AOB, angle between OA and OB is same as angle between


and

, as shown in figure given below. We have

is also perpendicular to vector OB.

OB = OA = r and

So, AOB is similar to the triangle formed by

and

and

because

is perpendicular to vector OA

Thus,From geometry we have, v/v = AB/r


Now AB is approximately equal to vt.

In the limit t 0 the above relation becomes exact, we have

This is the magnitude of the acceleration. The direction


because of this

is instantaneously along a radius inward towards the centre of the circle,

is called radial or centripetal acceleration. The acceleration vector of a particle in uniform circular motion

averaged over one cycle is a null vector.


Problem 3:
The moon revolved about the earth making a complete revolution in 2.36 mega second. Assume that the orbit is circular and has a
radius of 385 mega meter. What is the magnitude of the acceleration of the moon towards the earth?
Solution:Here first of all we calculate speed v of the moon which is given by v = 2R/T
where R = 385 mega meter = 385 106 m
and T = 2.36 mega second = 2.36 106 sec.
Hence v = 1020 m/sec.
The magnitude of centripetal acceleration is
a = v2/R = 0.00273 m/sec2.
From the above observation we conclude that, the magnitude of the acceleration of the moon towards the earth would be 0.00273
m/sec2.
In the previous enquiry we have discussed the uniform circular motion in which the particle has constant speed. If the particle's
speed varies with time then the motion will be no more uniform but a non-uniform circular motion. Let us discuss about this motion
using the concept of vectors.

Simulation for Car and Curves:-

This animation is used to explain why a passenger slides to the "outside" of a curve while riding
inside a car is NOT an example of centrifugal forces. Instead is is a combination of centripetal
force and inertia. It emphasize that when an object moves to the outside of a circle it is
because of a lack of enough centripetal force and inertia keeps it moving in a straight line.
Non uniform circular motion:-

Let us use the vector method to discuss non-uniform circular motion.


In the side figure,

and

are unit vectors along radius and tangent vector respectively. In terms of er and e the motion of a

particle moving counter clockwise in a circle about the origin in figure 2.30 can be described be the vector equation.

In this case, not only

=(d

)/dt =

but v also varies with time. We can obtain instantaneous acceleration as,

d/dt + v(d

)/dt

Again,

Here, aT = dv/dt and aR = v2/r


The first term,

is the vector component of

that is tangential to the path of the particle and arises from a change in the

magnitude of the velocity in circular motion, called tangential acceleration whereas aR centripetal acceleration.
The magnitude of

Problem 4:

is

Point A travels along an arc of a circle of radius r as shown in figure given below. Its velocity
depends on the arc coordinates l as v = A l where A is a constant. Let us calculate the angle between the vectors of the total
acceleration and of the velocity of the point as a function of the coordinate l.
Solution:It is seen from figure shown above that the angle can be found by means of the
formula tan = aR/aT. Let us find aR and aT.
aR = v2/r=(A2 l)/r; aT= dv/dt = dv/dl = A/(2l)Al = A2/2
Hence tan = 2l/r.
Problem 5 (JEE Main):
Position vector of a particle moving in x-y palne at time t is
.
What will be the path of the particle?
Solution:x = a (1-cos t)
y = a sint
From first equation, we get
acos t = a-X

... (1)

From second equations, we get,


a sin t = y
Squaring and adding Eqs. (1) and (2), we get,
(X-a)2 + y2 = a2
This is an equation of a circle of radius a and ceter at (a,0).

Centripetal and centrifugal forces are equal in magnitude and opposite in direction.

Centripetal and centrifugal forces cannot be termed as action and reaction since action and reaction never act on same body.

Basic requirement for a body to complete motion in a vertical circle, under limiting conditions, is that the tension in the string must not vanish
before it reaches the highest point. If it vanishes earlier, it will be devoid of the necessary centripetal force required to keep the body moving in
a circle.

Cetrifugal force is the fictitious force which acts on a body, rotating with uniform velocity in a circle, along the radius away from the centre.

Relative Motion
Relative Velocity

In spite of our best efforts we could not find a fixed point with respect to which we should study absolute rest and absolute motion.
So, the study of relative rest and relative motion assumes importance.
Consider two cars A and B running parallel to each other on same road with same velocities. No doubt the speedo meter of each
indicates motion but the two drivers will always find themselves facing each other.
Relative velocity of a body A with respect to another body B, when both are in motion, is the velocity with which A appears
to move to B.
The position, velocity and acceleration of a particle depend on the reference frame chosen.
A particle P is moving and is observed from two frames S and S'. The frame S is stationary and the frame S' is in motion.
Let at any time position vector of the particle P with respect to S is,

Position vector of the origin of S' with respect to S is,

From vector triangle OO'P, we get,

Physical Significance of Relative Velocity

Let two cars move unidirectional. Two persons A and B are sitting in the vehicles as shown in figure. Assume, VA = 10 m/s & VB = 4
m/s. The person A notices person B to be moving towards him with a speed of (10-4) m/s = 6 m/sec. That is the velocity of B with
respect to (or relative to) A. That means

is directed from B to A.

Similarly A seems to move towards B with a speed 6 m/sec. That means the velocity of A relative to B (
m/sec & directed from A to B as shown in the figure.

Therefore,

In general,
So,

vAB = vA2 + vB2 - 2vAvB cos


and = tan-1{(vBsin)/(vA vBcos)}

) has the magnitude 6

Relative Motion between Rain and Man

We know that, vr = vrg = velocity of rain w.r.t. ground, vm vmg.


Velocity of man w.r.t. ground and

velocity of rain w.r.t. man.


So,
That means the vector addition of the velocity of rain with respect of man (
actual velocity of rain
2

. The magnitude and direction of

) and the velocity of man (vehicle) (

) yield the

can be given as,

vr = ((vrm) +(vm) + 2vrm vm cos)


= tan-1((vrm sin)/(vrm cos+ vm )) with horizontal

Illustration (JEE Main):


Four particles A, B, C and D are in motion. The velocities of one with respect to other are given as
north,

is 20 m/s towards east and

(a) 20 m/s towards north


(b) 20 m/s towards south
(c) 20 m/s towards east
(d) 20 m/s towards west
Solution:
From the question, we know that,

is 20 m/s towards south. Then

is

is 20 m/s towards

... (1)
... (2)
... (3)
Equation (1) (2) + (3) gives:

or
That is,

is 20 m/s towards west.

Thus from the above observation we conclude that, option (d) is correct.
Illustration:
A stationary person observes that rain is falling vertically down at 30 km/hr. A cyclist is moving on the level road, at 10 km/hr. In
which direction should the cyclist hold his umbrella to project himself from rain?
Solution:
Relative to stationary frame, velocity of rain is 30 km/hr downward. Take horizontal axis as x-axis and vertical axis as y-axis and i,j
are the unit vectors along x and y axes respectively.

= 0-30,

= 10

= -30j - 10i = -10i - 30j


If angle between horizontal and

is , then,

tan = -30/-10 = 3
=>

tan-1 3

=>

=72

Therefore, to protect himself from rain the cyclist should hold the umbrella at an angle of 72 from horizontal.
Illustration:
A man walking eastward at 5 m/s observes that wind is blowing from the north. On doubling his speed eastward, he observes that
wind is blowing from north-east. Find the velocity of the wind.
Solution:
Let velocity of the wind is,
vw=(v1i+v2j)m/s
And velocity of the man is,

vm=5i
So, vwm = vw- vm=(v1-5)i + v2j
In first case,
v1- 5 = 0 => v1= 5 m/s.
In the second case, tan 45o = v2/(v1- 10)
=> v2= v1 - 10 = -5 m/s.
=> vw= (5i - 5j) m/s.
Illustration:
From a lift moving upward with a uniform acceleration 'a', a man throws a ball vertically upwards with a velocity v relative to the lift.
The ball comes back to the man after a time t.
Show that a + g = 2 v/t
Solution:
Let us consider all the motion from lift frame. Then the acceleration, displacement and velocity everything will be considered from
the lift frame itself. As the ball comes back to the man, therefore displacement from the lift frame is zero. Again, the velocity with
respect to the lift frame is v.
g - (-a) = a + g ()
Now, s = ut + 1/2at

downwards
2

=>

0 = vt - 1/2 (a+g)t2

or a + g = 2(v/t) .
Relative Motion of a Swimmer in Flowing Water
Take

= velocity of man
= velocity of flow of river,

= velocity of swimmer w.r.t. river


can be found by the velocity addition of

and

Crossing of the River with Minimum Drift


Case 1: vmw > w
A man intends to reach the opposite bank at the point directly opposite to the stationary point. He has to swim at angle with a
given speed
of water

w.r.t. water, such that his actual velocity


).

will direct along AB, that is perpendicular to the bank (or velocity

=> For minimum drift,

You can realize the situation by a simple example. If you want to reach the directly opposite point or cross the river perpendicularly,
a man, that is to say, Hari, must report you that, you are moving perpendicular to the shore. What does this report signify? Since
Hari observes your actual velocity (

) to be perpendicular to the bank

is perpendicular to

Observing the vector-triangle vw = vmw sin & vm = vmw cos


=> = sin-1 (vw/vmw ) & vm = ((vmw )2- (vw)2)
=> The time of crossing, t = d/vm
=> t = d/(vmw )2 - (vw)2)
Case 2 : vw > vmw
Let the man swim at an angle with normal to the bank for minimum drift. Suppose the drift is equal to zero. For zero drift, the
velocity of the man along the bank must be zero.
=> vm= vw- vmw sin ' = 0
This gives, sin' = vw / vmw, since vw > vmw, sin' > 1 which is impossible. Therefore, the drift cannot be zero.
Now, let the man swim at an angle with the normal to the bank to experience minimum drift. Suppose that the drifting of the man
during

time

when

the

BC = x
x = (vm)x (t)
where t = AB/((vm )y cos) = d/(vmw cos)
and (vm)x = vw vmw sin
Using (1), (2) & (3), we obtain
x = (vw- vmw sin d/(vmw cos))

(1)
(2)
(3)

reaches

the

opposite

bank

is,

= (vw/vmw sec -tan)d


x = (vw/vmw sec -tan)d

(4)

For x to be minimum,
dx/d = (vw/vmw sec - tan - sec2)d = 0
vw/vmw tan = (sec ) => sin = vmw/vw
= sin-1(vmw/vw)
Substituting the value of in (4), we obtain,
x = [(vw2 vmw2)/vmw] d
Crossing of the River in Minimum Time
Case 1: To reach the opposite bank for a given vmw
Let the man swim at an angle with AB. We know that the component of the velocity of man along shore is not responsible for its
crossing the river. Only the component of velocity of man (vm) along AB is responsible for its crossing along AB.
The time of crossing = t = AB/(vmw cos)
Time is minimum when cos is maximum.

The maximum value of cos is 1 for = 0.


That means the man should swim perpendicular to the shore.
=>

mw

=> Then tmin = d/(vmw cos)|(=0) = d/vmw => tmin= d/vmw


Case 2:
To reach directly opposite point on the other bank for a given vmw & velocity v of walking along the shore.
To attain the direct opposite point B in the minimum time, let the man swim at an angle with the direction AB. The total time of
journey t = the time taken from A to C and the time taken from C to B.
=> t = tAC + tCB

where tAC = AB/vmvcos & tCB = BC/v where v = walking speed of the man from C to B.
=> t = AB/vmvcos + BC/v
Again BC = (vm)xt
=> BC = (vw - vmwsin) (AB/vmwcos)
Using (1) & (2) we obtain,

t = AB/vmwcos + ((vw - vmwsin)/v(vmvcos))


=> t = AB[(1+vw/v)sec/vmv - tan/v]
=> t = d/vmv[(1+vw/v)sec/vmv - tan/v]
Putting dt/d = 0, For minimum t we get,
dt/d = d/d[d/vmv (1+vw/v) sec/vmv - tan/v]
= [sec/vmv - tan/v (1+vw/v) (sec2)/v] = 0
=>tan/vmv (1+vw/v) sec/v
=> sin = (vmw/v+vw)
=> = sin-1(vmw/v+vw)
This expression is obviously true when vmw < v + vw.
Velocity of Separation/Approach, Relative Angular Velocity
Let thane be two particles A and B with velocity

and

at any instant as visualized from ground frame.

If we visualize the motion of B from frame of A the velocity of particle B would be

If , be the angle made with line AB,


then, VB cos - VA cos is relatively velocity of B w.r.t. A along line AB.

If VB cos - VA cos > 0; it is called as velocity of separation.

If VB cos - VA cos < 0; it is called as velocity of approach.

VB sin - VA sin is relative velocity of B w.r.t. A along direction perpendicular to AB. If length of AB is,
then, angular velocity B w.r.t. A is (VB sin - VA sin)/l

and relative angular velocity = (VB sin - VA sin)/l.

The analytical method for determination of relative velocity of one body with respect to another which is discussed here is to general nature.
This treatment is valid for motion in two dimensions and in three dimensions also.

Relative velocity of a body A with respect to another body B, when both are in motion, is the velocity with which A appears to move to B.

Relative velocity of a body B w.r.t. a body A, when both are in motion, is the velocity with which B appears to move to A.

Solved Examples on Mechanics:Example 1:A 5.1-kg block is pulled along a frictionless floor by a cord that exerts a force P = 12 N at angle =25 above the horizontal as shown
in below figure. (a) What is the acceleration of the block? (b) The force P is slowly increased. What is the value of P just before the
block is lifted off the floor? (c) What is the acceleration of the block just before it is lifted off the floor?

Concept:The figure below shows the forces acting on the block with mass m:
Assume that the weight of the Block is given by W such that W = mg, the horizontal component of force
the vertical component of force is Py = Psin ( is the angle at which the force

is Px = Pcos whereas

acts).

(a) When the block does not lift off from the ground there is no acceleration in the vertical direction. However, the acceleration does
exist on the horizontal direction and is given as:
ax = Px/m
Substitute Px = Pcos,

ax = Pcos/m
Substitute the values of P, m and to get the desired result.
(b) When the force

is increased, the magnitude of vertical component of force Py = Psin will also increase. The moment at

which the block is about to lift, the magnitude of the vertical component of force will be equal to the weight W, that is,
Py = W
Substitute Py = Psin,
Psin = W
P = W/sin
Substitute the values of W and to get the desired result.
(c) Just before the block is lifted there in no acceleration in the vertical direction however the acceleration exists in the horizontal
direction, and is given as:
ax = Px/m
Substitute Px = Pcos,
ax = Pcos/m
Substitute the values of P, m and to get the desired result.
Solution:(a) Substitute 12 N for P, 5.1 kg for m and 25 for in equation ax = Pcos/m,
ax = Pcos/m
= 12 N cos (25) /5.1 kg
= (10.87 N) [(1 kg.m/s2) / 1 N] / 5.1 kg
= 2.2 m/s2
Therefore, the horizontal acceleration of the block is 2.2 m/s2.
(b) The weight of the block is:
W = mg
Substitute 5.1 kg for m and 9.8 m/s2 for g,
W = (5.1 kg) (9.8 m/s2)
= (49.9 kg.m/s2) (1 N/1 kg.m/s2) = 49.9 N
Substitute 49.9 N for W and 25 for in equation P = W/sin,
P = 49.9 N/sin ( 25) = 118.26 N
Round off to three significant figures,
P = 119 N
Therefore the magnitude of net force on the block when it is about to lift is 119 N.
(c) Substitute 119 N for P, 5.1 kg for m and 25 for in equation ax = Pcos/m,
ax= 119 N (cos25) / 5 kg
= (107.8 N) [(1 kg.m/s2)/1 N] / 5.1 kg = 21.1 m/s2
Round off to two significant figures,
ax = 21 m/s2
Therefore, the acceleration of the block when it is about to lift is 21 m/s2.
_____________________________________________________________________________________________________

Example 2:A worker drags a crate across a factory floor by pulling on a rope tied to the crate. The rope, which is inclined at 38.0 above the
horizontal, exerts a force of 450 N on the crate. The floor exerts a horizontal resistive force of 125 N, as shown in the below figure.
Calculate the acceleration of the crate (a) if its mass is 96.0 kg, and (b) if its weight is 96.0 N.

Concept:The figure below shows the forces acting on the crate of mass M, when it is pulled by a worker.
The force experienced by the crate is
components as

, and can be resolved into horizontal and vertical

and

respectively.

Also assume that the resistive force from the ground is represented by
the horizontal direction is given by

(refer figure above), and the acceleration of the crate in

and the acceleration in the vertical direction is

(a) Using Newtons second law of motion for the forces acting in the horizontal direction, one can write
F cos (38.0) f = Max
ax = [F cos (38.0) f]/ M

(1)

Therefore, the acceleration of the crate in the horizontal direction is ax = [F cos (38.0) f] / M.
Similarly, apply Newtons second law in the vertical direction,
F sin (38.0) Mg = May
ay = F sin (38.0) Mg / M
It can be seen from the equation above that if the term F sin (38.0) is smaller than the weight Mg, the crate will rest on the ground.
(b) If the weight of the crate is W, then its mass is W/g, and the acceleration given by equation (1) can be written as:
a = [F cos (38.0) f]/ (W/g)
Therefore, the acceleration of the crate of weight W can be calculated using relation a = [F cos (38.0) f]/ (W/g).
Similarly, apply Newtons second law in the vertical direction,
F sin (38.0) W = (W/g) ay
ay = [F sin (38.0) W] / [W/g]
Therefore the acceleration of the crate in the vertical direction is ay = [F sin (38.0) W] / [W/g].
Solution:(a) Substitute 450 N for F, 125 N for f and 96.0 kg for M in equation a = [F cos (38.0) f]/ M,

a = [F cos (38.0) f]/ M


= [(450 N) cos (38.0) 125 N]/[96.0 kg]
= (229.6 N) [(1 kg.m/s2)/1 N] / 96.0 kg
= 2.40 m/s2
Therefore, the horizontal acceleration of the crate of mass 96.0 kg is 2.40 m/s2.
Substitute 450 N for F in expression F sin (38.0),
F sin (38.0) = (450 N) sin (38.0) = 277 N
Substitute 9.8 m/s2 for g and 96.0 kg for M in expression Mg,
Mg = (96.0 kg) (9.8 m/s2)
= (940.8 kg.m/s2) [1 N/(1 kg.m/s2)] = 940.8 N
The weight of the crate is larger than the upward force F sin (38.0), therefore the crate will rest on the ground.
(b) Substitute 450 N for F, 125 N for f, 9.8 m/s2 for g and 96.0 N for W in equation a = [F cos (38.0) f]/ (W/g),
a = [(450 N) cos (38.0) 125 N]/(96.0 N/9.8 m/s2)
= (229.6 N) [(1 kg.m/s2)/1 N] / 9.79 kg = 23.4 m/s2
Therefore, the horizontal acceleration of the crate of weight 96.0 N is 23.4 m/s2.
Substitute 450 N for F, 125 N for f, 9.8 m/s2 for g and 96.0 N for W in equation ay = [F sin (38.0) W] / [W/g],
ay = [F sin (38.0) W] / [W/g]
= (450 N) sin (38.0) (96.0 N)/(96.0 N/9.8 m/s2)
= (181 N) [(1 kg.m/s2)/1 N] / 9.79 kg
= 18.4 m/s2
Therefore, the crate has an upward acceleration of 18.4 m/s2.
____________________________________________________________________________________________________
Example 3:A 110-kg crate is pushed at constant speed up the frictionless 34 ramp as shown in the below figure. What horizontal force F is
required? (Hint: Resolve forces into components parallel to the ramp.)

Concept:The figure below shows the component of forces

on the block, moving with a constant speed on a frictionless 34 ramp.

Assume that the x axis is along the length of the ramp, and y axis perpendicular to the ramp such that the component of force along
the x axis is Fx = F cos (34) whereas the component of force along the y axis is Fy = Fsin (34).
The block moves up the ramp at constant speed, and therefore has no acceleration in the x direction.
The Newtons third law of motion for the forces along the x axis gives,
Mg sin (34) F cos (34) = 0
Here M is the mass of the block and g is the acceleration due to gravity.
Mg sin (34) = F cos (34)
F = Mg sin (34) / cos (34)
= Mg tan (34)
Therefore the force required to move the block with constant speed on a frictionless 34 ramp is given by relation F = Mg tan (34).
Solution:Substitute 110 kg for M and 9.8 m/s2 for g in equation F = Mg tan (34),
F = Mg tan (34)
= (110 kg) (9.8 m/s2) tan (34)
= (727 kg.m/s2) [1 N/(1 kg.m/s2)]
= 727 N
Therefore, the magnitude of force required to move the block with constant speed on a frictionless 34 ramp is 727 N.
_________________________________________________________________________________________________
Example 4:A jet fighter takes off an angle of 27.0 with the horizontal, accelerating at 2.62 m/s2. The weight of the plane is 79,300 N. Find (a)
the thrust T of the engine on the plane and (b) the lift force L exerted by the air perpendicular to the wings; see below figure. Ignore
air resistance.

Concept:The figure below shows the forces acting on the jet fighter:
Assume that the x direction is in the direction of motion of plane during its take off and the y direction perpendicular to it such that
the weight W can be resolved into components along the x and y direction as Wx = W sin and Wy = W cos ( is the angle between
the jet and the ground).
(a) If ax is the acceleration of the jet in x direction, then the net force acting on the jet is F = (W/g) ax (g is the free fall acceleration).
Apply Newtons second Law of motion along the x direction,
T Wx = F
Substitute Wx = W sin and F = (W/g) ax,
T Wsin = (W/g) ax
T = (W/g) ax + Wsin
Substitute the value of ax, W and to calculate the desired result.
(b) The jet does not have any acceleration in the y direction, therefore the sum of the forces acting on it in that direction must be
equal to zero, that is
L Wy = 0
L = Wy
Substitute Wy = W cos,
L = W cos
Solution:(a)Substitute 79,300 N for W, 2.62 m/s2 for ax, 9.8 m/s2 for g and 27.0 for in equation T = (W/g) ax + Wsin,
T = [(79,300 N)/(9.8 m/s2)] (2.62 m/s2) + (79,300 N) sin (27.0)
= 21,200.6 N + 36001.4 N
= 57,202 N
Round off to three significant figures,
T = 57,000 N
Therefore, the magnitude of thrust T is 57,000 N.
(b) Substitute 79,300 N for W and 27.0 for in equation L = W cos,
L = W cos

= (79,300 N) cos (27.0)


= 70656.8 N
Round off to three significant figures,
L = 70,600 N
Therefore the magnitude of the force L is 70,600 N.
___________________________________________________________________________________________________
Example 5:A certain airplane has a speed of 180 mi/h and is diving at an angle of 27 below the horizontal when a radar decoy is released. The
horizontal distance between the release point and the point where the decoy strikes the ground is 2300 ft. (a) How long was the
decoy in the air? (b) How high was the plane when the decoy was released? See below figure.

Concept:The figure below shows the plane, decoy and the initial velocity v0 of the decoy when it was
released.
It can be seen from the figure that the horizontal component of velocity of the decoy when it was released is v0 cos (-) while the
vertical component of the decoy is v0 sin (-). It is important to note that the negative sign in is due to the fact that the angle is
measured in clockwise direction with respect to horizontal.
(a) The time of flight of decoy can be calculated using relation,
t = x/v0cos
Here x is the range of the decoy, v0 is the initial velocity and is the angle between the decoy with respect to horizontal (refer figure
above).
(b) The vertical displacement of the decoy during its time of flight can be calculated using relation,
y = (v0 sin) t gt2

Here g is the acceleration due to gravity and t is the time of flight calculated in part (a).
Solution:(a) Substitute -27 for , 180 mi/h for v0 and 2300 ft for x in equation t = x/v0cos,
t = [2300 ft] / [(180 mi/h) (5280 ft/ 1 mi) (1 h/3600 s) cos (27)]
= (2300 ft)/(235.2 ft/s)
= 9.77 s
Therefore, it takes 9.77 s for decoy to reach the target.
(b) Substitute 9.77 s for t, 32 ft/s2 for g, -27 for and 180 mi/h for v0 in equation y = (v0 sin) t gt2 ,
y = ((180 mi/h) sin (27)) (9.77 s) (32 ft/s2) (9.77 s)2
= [((180 mi/h) (5280 ft/1 mi) (1 h/3600 s) sin (-27)) (9.77 s)] (32 ft/s)2 (9.77 s)2
= -1170.9 ft 15272 ft
= -2698 ft
The negative sign accounts for the fact that the decoy moves closer to ground.
Therefore, the vertical displacement of decoy is 2698 ft.
___________________________________________________________________________________________________
Example 6:In earlier days, horses pulled barges down canals in the manner shown in below figure. Suppose that the horse pulls a rope that
exerts a horizontal force of 7900 N at an angle of 18 to the direction of motion of the barge, which is headed straight along the
canal. The mass of the barge, which is headed straight along the canal. The mass of the barge is 9500 kg and its acceleration is
0.12 m/s2. Calculate the horizontal force exerted by the water on the barge.

Concept:The force say

acting on the barges of mass m is show in the figure below:

The horizontal component of force on the barge is Fx = F cos whereas the vertical component of force is Fy = F sin ( is the angle
made by force F relative to horizontal axis).
Assume that the horizontal force exerted by the water on the barge is Px, then from Newtons second law one can write,

Fx Px = max
Px = Fx - max
Here, ax is the acceleration of the barges in the horizontal direction.
Substitute
Fx = F cos,
Px = F cos - max
Substitute the value of F, m and a to get the desired result.
Solution:Substitute 0.12 m/s2 for a, 9500 kg for m, 7900 N for F and 18 for in equation Px = F cos - max,
Px = F cos - max
= (7900 N) cos(18) (9500 kg) (0.12 m/s2)
= 7513.3 N (1140 kg.m/s2) [1 N/(1 kg.m/s2)]
= 6373. 3 N
Round off to four significant figures,
Px = 6373 N
Therefore the magnitude of horizontal force exerted by the water is 6373 N and it acts in a direction opposite to the horizontal
motion of barges.
__________________________________________________________________________________________
Example 7:A dart is thrown horizontally toward the bulls eye, point P on the dart board, with an initial speed of 10 m/s. It hits at point Q on the
rim, vertically below P, 0.19 s later; as shown in below figure. (a) What is the distance PQ? (b) How far away from the dart board did
the player stand?

Concept:(a) The distance PQ is the vertical distance travelled by the dart in time t under the action of gravitational force, and can be given as:
y = - gt2
The negative sign is due to the convention that the vectors pointing downwards are taken as negative.
Here g is the acceleration due to gravity and t is the elapsed time.
(b) The horizontal distance between the dart board and the player can be calculated using
relation,
x = v0xt
Here v0x is the initial speed of the dart and t is the elapsed time.
Solution:(a)Substitute 9.81 m/s2 for g and 0.19 s for t in equation y = - gt2,
y = - (9.81 m/s2) (0.19 s)2
= - 0.17 m
The negative sign indicates the fact that the displacement is in the negative y direction.
Therefore the distance PQ is -0.17 m.
(b) Substitute 0.19 s for t and 10 m/s for v0x in equation x = v0xt,
x = (10 m/s) (0.19 s)
= 1.9 m
Therefore, the horizontal separation between the dart board and the player is 1.9 m.
_________________________________________________________________________________________

Example 8:In a baseball game, a batter hits the ball at a height of 4.60 ft above the ground so that its angle of projection is 52.0 to the
horizontal. The ball lands in the grandstand, 39.0 ft up from the bottom; see in below figure. The grandstand seats slope upward at
28.0 with the bottom seats 358 ft from home plate. Calculate the speed with which the ball left the bat. (Ignore air resistance.)

Concept:The figure below shows the trajectory of the ball:


It can be seen from the figure above that the total horizontal displacement say x of the ball is x = 358 ft + 39 ft cos (28) whereas the
vertical displacement say y of the ball is y =39 ft sin (28) 4.6 ft .
Assume that the ball left the ball with initial velocity v0 at an angle such that the horizontal component of initial velocity is v0x =
v0 cos whereas the vertical component of initial velocity is v0y = v0 sin.
The ball does not accelerate in the horizontal direction, therefore the horizontal distance travelled by the ball is:
x = v0xt
Substitute x = 358 ft + 39 ft cos (28) and vox = v0cos,
358 ft + 39 ft cos (28) = (v0cos) t
t = [358 ft + 39 ft cos (28)] / (v0cos)
Similarly, the vertical displacement of the ball can be written as:
y = v0yt gt2
Substitute v0y = v0 sin and y =39 ft sin (28) 4.6 ft,
[39 ft sin (28) 4.6 ft] = (v0 sin)t gt2
Substitute t = [358 ft + 39 ft cos (28)] / (v0cos) ,
[39 ft sin (28) 4.6 ft] = (v0 sin)[[358 ft + 39 ft cos (28)] / (v0cos)] g{[358 ft + 39 ft cos (28)] / (v0cos)}2
13.7 ft = (392.4 ft) tan g (392.4 ft/v0cos)2
Solve the equation for v0 to get the desired result.
Solution:Substitute 52 for and 32.2 ft/s2 for g in equation 13.7 ft = (392.4 ft) tan g (392.4 ft/v0cos)2,
13.7 ft = (392.4 ft) tan (52) g (392.4 ft/v0cos(52))2,
(32.2 ft/s2) [392.4 ft/v0 cos (52)]2 = 488.5 ft
Round of to three significant figures,
v0 = 115 ft/s

Therefore, the speed with which the ball leaves the bat is 115 ft/s.
_____________________________________________________________________________________________
Example 9:You throw a ball with a speed of 25.3 m/s at an angle of 42.0 above the horizontal directly toward a a wall as shown in below figure.
The wall is 21.8 m from the release point of the ball. (a) How long is the ball in the air before it hits the wall? (b) How far above the
release point does the ball hit the wall? (c) What are the horizontal and vertical components of its velocity as it hits the wall? (d) Has
it passed the heighest point on its trajectory when it hits?
Concept:The figure below shows the velocity of the ball and its components along the horizontal and vertical directions respectively:

The horizontal component of initial velocity of ball is:


vx = v0 cos
Here, v0 is the magnitude of initial velocity of the ball, and is the angle at which the ball is projected relative to horizontal.
The vertical component of initial velocity of the ball is:
vy = v0 sin
Here, v0 is the magnitude of initial velocity of the ball, and is the angle at which the ball is projected relative to horizontal.
(a) The time of flight of the ball is given as:
t = x/vx
Here, x is the range of the projectile.
Substitute vx = v0 cos,
t = x/v0 cos
Substitute the value of x, v0 and to get the desired result.
(b) The vertical displacement of the ball corresponding to time t is:
y = vyt gt2
Substitute vy = v0sin,
y = (v0sin) t gt2
Substitute the calculated value of t, and the given values of v0, to obtain the desired result.

(c) The horizontal component of velocity remains the same as the ball hits the wall because the ball has no acceleration in the
horizontal direction. However, the ball acceleration in the vertical direction and therefore the magnitude of its final vertical velocity
changes to,
vy = vy gt
Substitute vy = v0 sin ,
vy = v0 sin gt
Substitute the calculated value of t, and the given values of v0, and g to obtain the desired result.
(d) If the magnitude of vy has a positive sign, the ball is still heading up and has not reached its maximum height yet. However, if the
sign is negative, the velocity vector of the ball has reversed its direction and the ball has started falling downwards depicting that the
maximum height is being crossed.
Solution:(a) Substitute 25.3 m/s for v0, 42.0 for and 21.8 m for x in equation t = x/v0 cos,
t = [21.8 m]/[(25.3 m/s) cos (42.0)] = 1.15 s
Therefore, the time of flight of the ball is 1.15 s.
(b) Substitute 25.3 m/s for v0, 42.0 for , 9.8 m/s2 for g, and 1.15 s for t in equation y = (v0sin) t gt2,
y = (v0sin) t gt2
= ((25.3 m/s) sin (42.0)) (1.15 s) (9.8 m/s)2 (1.15 s)2 = 12.9 m
Round off to two significant figures,
y = 13 m
Therefore, the vertical distance travelled by the ball in 1.15 s is 13 m.
(c) Substitute 25.3 m/s for v0, 42.0 for , 9.8 m/s2 for g, and 1.15 s for t in equation vy = v0 sin gt,
vy = v0 sin gt
= (25.3 m/s) sin (42.0) (9.8 m/s2) (1.15 s)
= 5.65 m/s
Therefore, the vertical component of final velocity of the ball before hitting the wall is 5.65 m/s.
The horizontal component of velocity remains unchanged during its flight,
vx = vx = v0 cos
Substitute 25.3 m/s for v0 and 42.0 for ,
vx = (25.3 m/s) cos (42.0) = 18.8 m/s
Therefore, the horizontal component of final velocity of ball, before hitting the wall is 18.8 m/s.
(d) From part (c) it is clear that vertical component of final velocity of ball is 5.65 m/s, and is positive. Therefore the ball is yet to
reach its maximum height but collided with the wall before doing the same.
_____________________________________________________________________________________________
Example 10:(a) In Galileos Two New Sciences, the author states that for elevations [ angles of projection] which exceed or fall short of 45 by
equal amounts, the ranges are equal. Prove this statement. See below
figure. (b) For an initial speed of 30.0 m/s and a range of 20.0 m, find the two
possible elevation

Concept:Assume the angle by which one of the projectiles falls shorter of 45 is whereas the other one exceeds 45 by the same account.
The range of the projectile with angle of projection 45 is given as:
R1 = v02 sin ( 45 ) / g
Here, v0 is the initial speed of the projectile, and g is the acceleration due to gravity.
Similarly, the range of the projectile with angle of projection 45 + is:
R2 = v02 sin 2 ( 45 + ) / g
Here, v0 is the initial speed of the projectile, and g is the acceleration due to gravity.
The term v02/ g remains the same in R1 and R2, therefore if the values sin 2 (45 - ) is equal to sin 2 (45 - ), then the range R1 and
R2 is equal.
(b) Assume the angle of projection of one of the projectile is , then its value can be calculated from expression for R as
R = v02 sin 2 / g
= sin-1 (Rg/v02)
The angle of projection for the other projectile is 90 - .
Solution:(a) The quantities sin 2 (45 - ) and sin 2 (45 + ) can be written as sin 2 (45 ).
Also,
sin 2 (45 ) = sin (90 2)
= cos ( 2)
= cos (2)
It is important to note that cos ( 2) = cos (-2) = cos (), and therefore the terms sin 2 (45 - ) and sin 2 (45 + ) are equal. This
proves the range R1 is equal to range R2.
(b) Substitute 30 m/s for v0, 20.0 m for R and 9.8 m/s2 for g in equation = sin-1 (Rg/v02),
= sin-1[(20.0 m) (9.8 m/s2) / (30 m/s)2] = 6.28
Round off to two significant figures, = 6.3
The other possible elevation angle of projectile is:
= 90
Substitute = 6.3,
= 90 6.3
= 83.7
Therefore, the first possible elevation angle of projectile is 6.3, whereas the other possible elevation angle of projectile is 83.7.

Newtons Laws of Motion:Newton's laws of motion are of fundamental importance in classical physics. Newton gave three laws connected with motion and
are, popularly, known as Newtons laws of motion.
(a) First Law (Law of Inertia):-

Everybody continues in its state of rest or of uniform motion in a straight line unless it is compelled by some external force to change
that state.

(b) Second Law:-

The rate of change of momentum of a body is directly proportional to the impressed force and takes place in the direction of the
force.
So, F= dp/dt
This results force acting a body F is equal to the mass of the body m times acceleration of the body a.
So, F = ma

(c)Third Law:-

To Every action there is an equal and opposite reaction.


FAB = - FBA
The first two laws relate to the type of motion of a system that results from a given set of forces. The third law studies the basic
nature of all forces i.e. they act in action - reaction pair.
"Newton's laws of Motion" is one of the easiest and important chapters of Mechanics in the Physics syllabus of IIT JEE, AIEEE
and other engineering examinations. These laws are not new to any aspirant as he is using these laws even in previous classes.
The examples based on this are very easy and can be seen even in day to day life.
The chapter is important not only because it fetches 2-3 questions in most of the engineering examination but also because it is
prerequisite to the other chapters of Mechanics.
Types of Forces:-

If we want to produce motion in a body, already at rest, or if we want to destroy the motion of a moving body, we have to apply
effort known as force in the form of a pull or push.
Force:-

Force is defined as that pull or push which produces or tends to produce, destroys or tends to destroy motion in a body, increases or
decreases the speed of the body or changes its direction ofmotion. Therefore, Force" is an external or internal agent present to
"influence" the natural state of motion of an object. Thus, this is an influence (force) needed to change the natural state of body; that
is of rest or of uniform motion.
Force is the basic cause of motion.

Classification of Forces:There are different types of forces in our universe. Based on the nature of the interaction between two bodies, forces can be
classified into two main categories.

(a)

Contact Forces:-

These are the forces which come into play due to actual contact between the sources and the object. Basically, it is the force that
act between the bodies in contact with each other. For example, Normal Reaction, Friction etc.
(i) Tensional Force (T):When a string, thread or wire is held taut, the ends of the string or thread (or wire) pull on whatever bodies are attached to them in
the direction of the string. This force is known as Tension.
If the string is massless, then the tension T has the same magnitude at all points throughout the string.
The direction of tension is always from the point of attachment to the body.

(ii) Spring Force:Force in an extended (or compressed) spring is proportional to the magnitude of extension (or compression).
i.e. F x, in magnitude, but opposite in direction.
So, F = -kx, where k is a positive constant, also known as the spring constant of the spring; and x is the compression or elongation
from the natural length.
(iii) Normal reaction:-

When a body exerts a force on another, the second provides a reaction which acts perpendicular to the surface of 2nd body.
(iv) Friction:-

It is a force that acts between bodies in contact with each other along the surface of contact and it opposes relative motion (or
tendency of relative motion) between the two bodies.

(v) Air Resistance (Fa):Applicable when motion takes place through air. This force becomes appreciable for bodies moving at high speeds.

(vi) Weight (W):It is a field force, the force with which a body is pulled towards the center of the earth due to its gravity. It has the magnitude mg,
where m is the mass of the body and g is the acceleration due to gravity.

(b)

Non-Contact Forces (Field Forces or Action at a Distance Forces):-

Forces which come into existence without any physical contact between the bodies. These forces are due to some inherent
characteristics of the body.

Forces that act between bodies separated by a distance without any actual contact. For example, Tension, Spring, Weight etc.
Examples:(i) Gravitational Forces (Fg):- This is due to the gravitation attraction between two bodies. If we deal with cases of attraction due to
earth, this force is always directed downwards.
(ii) Electrical Forces (Felec):- These forces are due to the charges present on the two bodies. The direction of these forces depends
upon the type of charges on the two interacting bodies.
(iii) Magnetic Forces (Fmag):- Forces which come into play between two bodies due to their magnetic characteristics. Their
direction also depends upon the nature of magnetic polarity acquired by the body.
In mechanics we shall only deal with gravitational forces from this category.

Newtons First Law of Motion:-

To study Newtons first law of motion, the concept of equilibrium should be clear to us. Whenever a number of forces act on a body
and they neutralize each others effect, the body is said to be in equilibrium. In such a case there is no change in the state of rest or
of motion. If however, the system of forces have a resultant, the state of rest or that of motion undergoes a change. This
is explained by Newtons first law of motion.
It states that, Every body continues in its state of rest or of uniform motion in a straight line unless it is compelled by some
external force to change that state. Therefore, every object persists in its natural state of motion i.e. continues to be at rest
or moves in a straight line with uniform (constant) velocity, in the absence of a net external force acting (impressed) on it.
It can be easily deduced from the statement of change in the state of motion. It is directly related to a frame of reference about
which we have discussed earlier. To mark the point here, we can discover that by viewing objects from different frame of
references the natural state of motion as perceived by different observers will be obviously different (can only be same if the frames
are truly equivalent). Therefore, the change in state will also depend on the choice of reference frame. Finally, the amount of
acceleration produced in a body (or change in velocity) will depend on our choice of reference frames.

Law of Inertia:-

Inertia is the property of all bodies by virtue of which they are unable to change their state of rest or of uniform motion in a
straight line without the help of an external force. In other words inertia can also be termed as a resistance to change the
state of motion of a body.
Inertia can be classified into following three categories.
(a) Inertia of Rest:It is the property of a body by virtue of which it is unable to change its state of rest without the help of an external force.
(b) Inertia of Motion:It is the property of a body by virtue of which it is not able to change its speed without the help of an external force.
(c) Inertia of Direction:It is the property of a body by virtue of which it is unable to change its direction of motion without the help of an external force.
Qualitative definition of force from first law:Newtons first law states that there cannot be any change in the state of rest or that of motion of a body unless some external force
acts upon it. In other words force is an agent which is capable of producing any change in state of rest or that of motion (including
direction). This provides a qualitative definition of force.
Some Conceptual Questions:Question 1:-

A car moving at constant speed is suddenly braked. The occupants, all wearing seat belts, are thrown forward. The instant the car
stops, however, the occupants are all jerked backward. Why? Is it possible to stop an automobile without this jerk?

Solution:Newtons first law states that, without any external force, if a body is at rest, it will remain at rest and if the body is moving with
constant velocity, it will continue to do so. When the car is suddenly braked, due to the inertia, the occupants in the car will tend to
move in the forward direction of car. When the car stops the sit belt in the car will produce backward momentum on the occupants.
Since the all the occupants wearing seat belts, therefore the occupants are all jerked backward.
Yes, it is possible to stop an automobile without this jerk. This can be done by slowing down the car a little longer time.
Question 2:Why do you fall forward when a moving bus decelerates to a stop and fall backward when it accelerates from rest? Subway
standees often find it convenient to face the side of the car when the train is starting or stopping and to face the front or rear when it
is running at constant speed. Why?
Solution:Newtons first law states that, without any external force, if a body is at rest, it will remain at rest and if the body is moving with
constant velocity, it will continue to do so.
When the moving bus decelerates, due to inertia, the body will tend to move in the forward direction of the bus. Therefore, you fall
forward when the moving bus decelerates to a stop. Again, when the bus accelerates from rest, due to inertia, the body will tend to
maintain the rest position. That is why; you fall backward when the bus accelerates from rest.
The tendency of a body to remain at rest or in uniform linear motion is called inertia. The reference frame to which it applies is called
inertial frames. When the train is starting or stopping, there is no net force acting on the observer. So the observer is in the inertial
frame. That is why; it is easy to face the side of the car because both the frame (one is observer in the car and the other one is the
outside of the car) are in rest. But when the car is running at constant speed, the observer in the car does not remain at rest. The
frame is not an inertial frame. That is why; it is easy to face the front or rear when the car is running at constant speed because the
car is in the forward direction.
Question 3:A mass 'M' is lying (figure shown below) on a table which is at rest (w.r.t. the table on which it is kept). Explain its state with the help
of Newton's First Law of motion.

Solution:Since 'M' is lying on a table, there is no external force acting on it (forget about gravity just for the immediate discussion). As per
Newton's first law of motion it will keep on lying at rest with respect to table for infinite time.
Here, comes out a very important, intrinsic (that is inherent) property of a body which is that it retains its state of motionlessness (as
well as of motion, if it is in motion) which is termed as INERTIA of an object. This is present in all materialistic bodies in this
universe.

Newtons Second Law of Motion:Momentum:-

Momentum of a body is defined as the amount of motion contained in a body.


Quantity of motion or the momentum of the body depends upon,
(a) mass of the body.
(b) velocity of the body.
Therefore momentum of a body of mass m and velocity v will be,

Quantitative definition:Momentum of a body is equal to the product of its mass and velocity. Momentum is a vector quantity and possesses the
direction of velocity.
Units:S.I:- kg m s-1
C.G.S:- g cm s-1
Momentum can be put into following two categories.
Dimension:[MLT-1]
(a) Non-relativistic momentum:According to classical physics (or non-relativistic physics) which is based upon the concepts of Newtons laws of motion, mass of a
body is considered to be a constant quantity, independent of the velocity of body. In that case momentum

is given by,

.
Thus, momentum of a body is a linear function of its velocity.
(b) Relativistic momentum:In accordance to Einsteins special theory of relativity, mass of a body depends upon the relative velocity v of the body with respect
to the observer. If m0 is the mass of body observed by an observer at rest with respect to body, its relativistic mass m is given by,

Therefore, momentum of a body according to the concepts of theory of relativity is given by,

Thus, relativistic momentum is not a linear function of v.

Frame of Reference:A system of co-ordinates whose axes can be suitably chosen is said to be a frame of reference. For location of a point P we need
three co-ordinate x, y and z. For complete identification of an event we must know t also, i.e., the time of the occurrence. Hence an
event in characterized by four co-ordinates (x,y,z,t). A reference frame describing an event in these four co-ordinates is known a
space time frame.

Inertial and Non-Inertial Frame of Reference:-

(a) Inertial Frame:A frame of reference either at rest or moving with a uniform velocity (zero acceleration) is known as inertial frame. All the
laws of physics hold good in such a frame.
An inertial frame is endowed with the following characteristics:
(i) All the fundamental laws of physics are valid in inertial frames.
(ii) All the fundamental laws of physics assume the same mathematical shape in all inertial frames.
(iii) They are isotopic with respect to mechanical and optical experiments
(b) Non-Inertial or Accelerated Frame:- It is a frame of reference which is either having a
uniform linear acceleration or is being rotated with uniform speed.
Newtons Second Law:-

The rate of change of momentum of a body is directly proportional to the impressed force and takes place in the direction
of the force.
Newtons first law provides a qualitative definition of the force while second law provides a quantitative definition of the force.
Let

be the instantaneous velocity of the body. Momentum

of the body is given by,

According to second law,


(rate of change of momentum)
Or,

Or,

Or,

Here k is the constant of proportionality. Mass m of a body is considered to be a constant quantity.

or,

The units of force are also selected that k becomes one.


Thus, if a unit force is chosen to be the force which produces a unit acceleration in a unit mass,
i.e., F = 1, m = 1 and a = 1.
Then, k = 1
So, Newtons second law can be written , in mathematical form, as

i.e., Force = (mass) (acceleration)


This provides us a measure of the force.

Here, if F = 0 then we find a = 0. This reminds us of first law of motion. That is, if net external force is absent, then there will be no
change in state of motion, that means its acceleration is zero.
Further we can extend second law of motion, (in fact its decomposition) to three mutually perpendicular directions as per our
coordinate system.
If components in x, y and z direction are Fx, Fy & Fz respectively, the three acceleration produced when Fx, Fy & Fz act
simultaneously) in the body are, Now,

If we add three forces then resultant is called net external force.


Similarly,

is called net acceleration produced in the body.


Unit of Force:S.I:- Newton [kg.m/sec2]
C.G.S:- Dyne [g.cm/sec2]
Dimension:[MLT-2]
Impulse:-

Impulse of a force is defined as the change in momentum produced by the force and it is equal to the product of force and
the time for which it acts. Therefore, a large force acting for a short time to produce a finite change in momentum which is called
impulse of this force and the force acted is called impulsive force or force of impulse.

According to Newtons second law of motion,

or,

So, Impulse of a force = change in momentum.


If the force acts for a small duration of time, the force is called impulsive force.

As force is a variable quantity, thus impulse will be,

The area under F - t curve gives the magnitude of impulse.


Impulse is a vector quantity and its direction is same as the direction of

Unit of Impulse:- The unit in S.I. system is kgm/sec or newton -second.


Dimension:- MLT1

Problem 1:The Sun yacht Diana, designed to negative in the solar system using the pressure of the sunlight, has a sail area of 3.1 km2 and a
mass of 930 kg. Near Earths orbit, the sun could exert a radiation force of 29 N on its sail. (a) What acceleration would such a force
impart to the craft? (b) A small acceleration can produce large effects if it acts steadily for a long enough time. Starting from rest
then, how far would the craft have moved after 1 day under these conditions? (c) What would then be its speed? (See The Wind
from the Sun, a fascinating science fiction account by Arthur C.Clarke of a Sun yacht race.)
Solution:(a)
Given Data:Mass of the yacht Diana, m = 930 kg
Force exerted by the sun light, F = 29 N
Force acting on the body (F) is equal to the product of mas of the body (m) and acceleration of the body (a).
So, F = ma

(1)

From equation (1), the acceleration (a) of the body would be,
a = F/m

(2)

Putting the value of m and a in equation (2), the acceleration such force impart to the craft would be,
a = F/m
= 29 N /930 kg
= (3.110-2 N/kg) (1 kg. m/s2 /1 N)
= 3.110-2 m/s2

(3)

Thus acceleration such force impart to the craft would be, 3.110-2 m/s2.
(b)
Given Data:Time, t = 1 day
= (1day) (24 h/1 day) (60 min/1 h) (60 s/1 min)
= 86400 s
Initial velocity, vi = 0
Acceleration, a = 3.110-2 m/s2
From equation of motion, we know that,
Distance travelled by the body (x) = vi + at2

So, x = vit+ at2

(4)

Putting the value of vi, a and t in equation (4), the distance travelled by the craft will be,
x = vit+ at2
= 0+ (3.110-2 m/s2) (86400 s)2
=1.1571108

(Since, a = 3.110-2 m/s2 and t = 86400 s)

(5)

Rounding off to two significant figures, the distance will be 1.2108 m.


Thus from the above observation we conclude that, the craft have moved1.2108 m after 1 day under these conditions.
(c)
Given data:
Acceleration, a = 3.110-2 m/s2
Time, t = 86400 s
Acceleration of an object is equal to the velocity of the object divided by time.
a = v/t
So, v = at

(6)

Putting the value of a and t in equation (6), velocity would be,


v = at
= (3.110-2 m/s2) (86400 s)
= 2678.4 m/s
Rounding off to two significant figures, speed will be 2700 m/s.
Thus from the above observation we conclude that, speed will be 2700 m/s.
Problem 2:A car travelling at 53 km/h hits a bridge abutment. A passenger in the car moves forward a distance of 65 cm (with respect to the
road) while being brought to rest by an inflated air bag. What force (assumed constant) acts on the passengers upper torso, which
has a mass of 39 kg?
Concept:Force acting (F) on the body is equal to the mass of the body (m) times deceleration of the body (a).
F = ma (1)
Solution:First we have to find out the deceleration (a) of the car.
If v0 is the initial speed of car and v is the final speed of the car, then the average speed (vav) of the car will be,
vav, = (v+ v0)

(2)

To obtain the average speed (vav) while the car is decelerating, substitute 53 km/h for v0 and 0 m/s for v in the equation vav =
(v+ v0),
vav = (v+ v0)
= ((53 km/h)+ (0 m/s))
= ( 53 km/h) (1,000 m/1 km) (1 h/60 min) (1 min/60 s)
= 7.4 m/s

(3)

But average speed (vav) is equal to the rate of change of displacement (x).
vav = x/ t

So, t = x/ vav

(4)

To obtain the time of deceleration t, substitute 0.65 m for x and 7.4 m/s for vav in the equation t = x/ vav,
t = x/ vav
= 0.65 m /7.4 m/s
= 8.810-2 s

(5)

Deceleration (a) is equal to rate of change of velocity.


So, a = v /t
= ((0) - (53 km/h))/ 8.810-2 s
= (-53 km/h)/ 8.810-2 s
= ((-53 km/h) (1,000 m/1 km) (1 h/60 min) (1 min/60 s))/ 8.810-2 s
= (-14.7 m/s)/ (8.810-2 s)
= -1.7102 m/s2

(6)

To obtain the force (F) acting on the passengers upper torso having mass 39 kg, substitute 39 kg for mass mand -1.7102 m/s2 for
deceleration a in the equation, F = ma,
F = ma
= (39 kg) (-1.7102 m/s2)
= -6630 kg. m/s2
= -(6630 kg. m/s2) (1 N/1 kg. m/s2)
= -6630 N

(7)

Rounding off to two significant figures, the magnitude of the force will be 6600 N.
Problem 3:Workers are loading equipment into a freight elevator at the top floor of a building. However, they overload the elevator and the worn
cable snaps. The mass of the loaded elevatorat the time of the accident is 1600 kg. As the elevator falls, the guide rails exert a
constant retarding force of 3700 N on the elevator. At what speed does the elevator hit the bottom of the shaft 72 m below?
Concept:Force acting on the body (F) is equal to the product of mas of the body (m) and acceleration of the body (a).
So, F = ma
From equation F = ma, the acceleration (a) of the body would be,
a = F/m
Weight W of the object is equal to the mass m of the object times of the free fall acceleration g.
W = mg
In accordance to equation of motion, the distance y travelled by the body will be,
y = ut + at2
Here u is the initial velocity, t is the time, and a is the acceleration.
When the elevator falls, the initial velocity u will be equal to zero.
So, u = 0
Substitute 0 for u in the equation y = ut + at2,
y = ut + at2
= 0t + at2

= at2
So the time t will be,
t = 2y/a
Speed v is equal to the product of acceleration a of the body and time t.
So, v = at
Solution:To obtain the weight of the elevator W, substitute 1600 kg for mass of the elevator m and 9.81 m/s2 for free fall acceleration g in the
equation W = (m) (g),
W = (m) (g)
= (1600 kg) (9.81 m/s2)
=15680 kg,m/s2
= (15680 kg,m/s2) (1 N/1 kg,m/s2)
= 15680 N
The magnitude of the net force F will be,
F = W-R
To obtain the magnitude of the net force F, substitute 15680 N for W and 3700 N for retarding force R in the equation F = W-R,
F = W-R
= (15680 N) (3700 N)
=11980 N
Rounding off to two significant figures, the magnitude of the net force F will be 12000 N.
To obtain the acceleration a, substitute 12000 N for F and 1600 kg for m in the equation a = F/m, we get,
a = F/m
= 12000 N/1600 kg
= (7.5 N/kg) (1 kg.m/s2/1 N)
= 7.5 m/s2
To obtain the time t to fall, substitute -72 m for y and -7.5 m/s2 for a in the equation t = 2y/a,
t = 2y/a
= 2(-72 m) /(-7.5 m/s2)
= 4.4 s
To obtain the final speed v at which the elevator hits the bottom of the shaft 72 m below, substitute 7.5 m/s2(only magnitude of a)
for a and 4.4 s for t in the equation v = at, we get,
v = at
= (7.5 m/s2) (4.4 s)
= 33 m/s
From the above observation we conclude that, the speed at which the elevator hits the bottom of the shaft 72 m below would be 33
m/s.

Newtons Third Law of Motion:It states that,

To every action there is an equal and opposite reaction.

Whenever one force acts on a body, it gives rise to another force calledreaction. A single isolated force is an impossibility. The two
forces involved in any interaction between two bodies are called action and reaction. But this does not imply any difference in
their nature, or that one force is the cause and the other is the effect. Either force may be considered as action and the other
reaction to it.
It may be noted that action and reaction never act on same body.
Note: The most important fact to notice here is that these oppositely directed equal action and reaction can never balance or cancel
each other because they always act, on two different point (broadly on two different objects) For balancing any two forces the first
requirement is that they should act one and the same object. (or point, if object can be treated as a point mass, which is a common
practice)
Few examples on Newtons third law of motion:
(a)

Book kept on a table:-

A book lying on a table exerts a force on the table which is equal to the weight of the book. This is the force of action. The table
supports the book, by exerting an equal force on the book. This is the force of reaction, as shown in the below figure. As the system
is at rest, net force on it is zero. Therefore, forces of action and reaction must be equal and opposite.

(b) Walking on the ground:-

While walking a person presses the ground in the backward direction (action) by his feet. The ground pushes the person in forward
direction with an equal force (reaction). The component of reaction in the horizontal direction makes the person move forward.
(c) Process of Swimming:A swimmer pushes the water backwards (action). The water pushed the swimmer forward (reaction) with the same force. Hence the
swimmer swims.
(d) Firing from a gun:When a gun is fired, the bullet moves forward (action). The gun recoils backwards (reaction).

(e) Fight of jet planes and rockets:The burnt fuel which appears in the form of hot and highly compressed gases escapes through the nozzle (action) in the backward
direction. The escaping gases push the jet plane or rocket forward (reaction) with the same force, hence, the jet or rocket moves.
(f) Rubber ball re-bounds from a wall:When a rubber ball is struck against a wall or floor it exerts a force on a wall (action). The ball rebounds with an equal force
(reaction) exerted by the wall or floor on the ball.
(g) It is difficult to walk on sand or ice:This is because on pushing, sand gets displaced and reaction from sandy ground is very little. In case of ice, force of reaction is
again small because friction between feet and ice is very small.
(h) Driving a nail in to a wooden block without holding the block is difficult:This is because when the wooden block is not resting against a support, the block and nails both move forward on being hit with a
hammer. However, when the block is held firmly against a support, and the nail is hit, an equal reaction of the support drives the nail
into the block.
(i) A tea cup breaks on falling on the ground:Tea cup exerts certain force (action) on ground while the ground exerts an equal and opposite reaction on the cup. Ground is able to
withstand the action of cup, but the cup being relatively more delicate breaks due to reaction.

Problem 1:Two blocks, with masses m1 = 4.6 kg and m2 = 3.8 kg, are connected by a light spring on a horizontal frictionless table. At a certain
instant, when m2 has an acceleration a2 = 2.6 m/s2, (a) what is the force on m2 and (b) what is the acceleration of m1?
Concept:Force acting on the body (F) is equal to the product of mas of the body (m) and acceleration of the body (a).
So, F = ma
From equation F = ma, the acceleration (a) of the body would be,
a = F/m
Solution:(a) The net force Fx on the second box having mass m2 will be,
Fx = m2a2x
Here a2x is the acceleration of the second block.
To obtain the net force Fx on the second box having mass m2, substitute 3.8 kg mass m2 and 2.6 m/s2 fora2x in the equation
Fx = m2a2x,
Fx = m2a2x
= (3.8 kg) (2.6 m/s2)= 9.9 kg .m/s2
= (9.9 kg .m/s2) (1 N/ 1 kg .m/s2)= 9.9 N
From the above observation we conclude that, the net force Fx on the second box having mass m2 would be 9.9 N. There is only
one (relevant) force on the block, the force of block 1 on block 2.
(b) There is only one (relevant) force on block 1, the force of block 2 on block 1. By Newtons third law this force has a magnitude of
9.9 N.
So the Newtons second law gives,
Fx = m1a1x = -9.9 N
But, m1a1x = (4.6 kg) (a1x)

(Since, m1 = 4.6 kg)

(4.6 kg) (a1x) = -9.9 N


So, a1x = -9.9 N/4.6 kg
= (- 2.2 N/kg) (1 kg.m/s2 / 1 N) = -2.2 m/s2
From the above observation we conclude that, the acceleration of m1 will be -2.2 m/s2.
_______________________________________________________________________________________________
Problem 2:A meteor of mass 0.25 kg is falling vertically through Earths atmosphere with an acceleration of 9.2 m/s2. In addition to gravity, a
vertical retarding force (due to frictional drag of the atmosphere) acts on the meteor as shown in the below figure. What is the
magnitude of this retarding force?

Solution:Given Data:
Mass of the meteor, m = 0.25 kg
Acceleration of the meteor, a = 9.2 m/s2
The net force exerted (Fnet) on the meteor will be,
Fnet = ma
= (0.25 kg) (9.2 m/s2) = (2.30 kg. m/s2) (1 N/ 1 kg. m/s2) = 2.30 N

(1)

If g (g = 9.80 m/s2) is the free fall acceleration of meteor, then the weight of the meteor (W) will be,
W = mg = (0.25 kg) (9.80 m/s2)
= (2.45 kg. m/s2) (1 N/ 1 kg. m/s2) = 2.45 N

(2)

The vertical retarding force would be equal to the net force exerted on the meteor (Fnet) minus weight of the meteor (W).
So, vertical retarding force = Fnet W

(3)

Putting the value of Fnet and W in equation (3), the vertical retarding force will be,
Vertical retarding force = Fnet W = 2.30 N -2.45 N = -0.15 N

. (4)

From equation (4) we observed that, magnitude of the vertical retarding force would be, -0.15 N.
_______________________________________________________________________________________________
Problem 3:Suppose in figure shown above we put one more block of 5 kg mass adjacent to 10 kg and a force of
150 N acts as shown in the figure below, then find the forces acting on the interface.

Solution:The

combined

acceleration

a = F/((10+5))=150/15=10/sec

of

the

two

bodies

when

treated

as

one

is

So each one moves with a = 10m/sec2 keeping their contact established.


Here you can feel that due to 150N force the body of 5 kg feels as if it is being pushed by the 10 kg mass. There is force acting on
5kg called R1, to oppose it by third law this body exerts a force R2 on 10kg. The interface is as shown in Figure given below.

Also, third law tells us that R1 = R2 in magnitude and is opposite in direction.

R1 = R2 = R
Here since 150 N force acts on the 10kg mass and only r acts on the 5kg mass. For motion in 5kg only R is responsible. We can
write the initial equation as:
F = 150 = (10 + 5) a
150 = 10a + 5a
Here 10a is force experienced by 10kg mass. And 5a is experienced by 5kg mass.
R = 5a a = 10m/sec2
So,R = 50N
Thus,Net force experienced by 10kg block is (150-R) = 10a 150-R = 1010 = 100 N
Therefore, R = 50
Therefore we get R = 50N for both blocks. Hence we find "action and reaction are equal and opposite". Now net force on the body of
10kg mass is 100N & Net force on the body of 5kg mass is 50N and on the interface action and reaction are both equal and also are
equal to force experienced by second body.
_________________________________________________________________________________________________
Problem 4:An object is hung from a spring scale attached to the ceiling of an elevator. The scale reads 65 N when the elevator is standing still.
(a) What is the reading when the elevator is moving upward with constant speed of 7.6 m/s? (b) What is the reading of the scale
when the elevator is moving upward with a speed of 7.6 m/s and decelerating at 2.4 m/s2?
Solution:Weight of the object (W) when the elevator is standing, W = 65 N
(a) We have to find out the scale reading when the elevator is moving upward with a constant speed of 7.6 m/s.
Since the elevator is moving upward with a constant speed, therefore there is no acceleration of the system resulting there is no
force. Thus the scale reading must be equal to the weight of the object and that will be 65 N.
(b) We have to find out the scale reading when the elevator is moving upward with a speed of 7.6
m/s and decelerating at 2.4 m/s2.

The force exerted on the object due to the deceleration at the rate 2.4 m/s2 (a = - 2.4 m/s2) will be,
F = ma
= (W/g) a

(1)

Where W is the weight of the object (W = 65 N) when the elevator is at rest and g is the free fall acceleration of the object (g=9.80
m/s2).
Putting the value of W, g and a in equation (1) the force exerted on the object will be,
F = (W/g) a
= (65 N/9.80 m/s2) (-2.4 m/s2)
= -15.92 N
(2) (Rounding off to two significant figure)

= -16 N

When the elevator is moving upward with a speed of 7.6 m/s and decelerating at 2.4 m/s2, the force would be,
= F- (-W)
= -16 N (-65 N)
= -16 N+65 N
= 49 N

(3)

From the above observation we conclude that, the scale reading when the elevator is moving upward with a speed of 7.6 m/s and
decelerating at 2.4 m/s2 would be 49 N.
Frame of Reference:A system of co-ordinates whose axes can be suitably chosen is said to be a frame of reference. For location of a point P we need
three co-ordinate x, y and z. For complete identification of an event we must know t also, i.e., the time of the occurrence. Hence an
event in characterized by four co-ordinates (x,y,z,t). A reference frame describing an event in these four co-ordinates is known a
space time frame.

Inertial and Non-Inertial Frame of Reference:In general we solve the problem of mechanics using inertial frame, which was discussed in chapter two, but as the same time it is
possible to solve the same problem using a non-inertial frame. Let us discuss about the difference between these frames.
When Newton stated his first law he made a very important distinction. He decreed the absolute equivalence between a state of rest
and one f uniform motion and distinguished it specifically and absolutely from that of an accelerated motion. If the environment is
completely symmetric then no direction is preferred over another and therefore if a body possesses a initial velocity (which might be
zero) it will persist with that velocity. If suppose we say that the velocity will change then we will have to concede that the velocity

changes in a particular direction. But why should it change in one direction and not in the other, since all direction are
equally favored. So the only way it can change is to change in all directions. But this is impossible so it will not change at all, i.e. if
environment is really symmetric. Therefore if we grant a change in velocity we will also have to grant an irregularity in the
environment in the same direction as the change in velocity. The acceleration, he said to be understood as an irregularity and he
expressed force as that basic asymmetry in the environment which produces this irregularity. The most important aspect in all this is
that force is theoretical construction to explain away the irregularities in motion and is not to be understood as a tangible entity.

Now, for constant mass system


If force is a tangible entity then the force in all systems on the same body should be same. Let us see if this is true.
Consider a body of mass m. We will observe its motion from three different frames.
Simulation for Frame of Reference:(i)

Reference frame is at rest:-

The acceleration of the mass will be, say,


Therefore the force on it will be

We will reason that

(ii) Reference frame starts moving with constant velocity vector-v :The acceleration of frame =
Thus, acceleration of mass m relative to frame is given by

Force on it will be

inertial and we will reason that

(iii) Reference frame moves with constant acceleration:Let the acceleration of frame be

Thus, acceleration of mass relative to frame will be

Let there be force

frame on mass we will reason, that

We see that the force is not the same as that in the inertial frames.
Therefore we postulate that under observation from an accelerated reference frame we substitute the inertial forces on the body with
those same initial forces plus an additional force which numerically equal to the mass of the body under observation times the
acceleration of the frame taken in the opposite direction. This force we call as pseudo force.
Now, we can work on a problem from an accelerated reference frame by just adding a pseudo force and pretending that nothing has
changed.
Let us illustrate our point.
(a) Inertial Frame:A frame of reference either at rest or moving with a uniform velocity (zero acceleration) is known as inertial frame. All the
laws of physics hold good in such a frame.

(b) Non-Inertial or Accelerated Frame:It is a frame of reference which is either having a uniform linear acceleration or is being rotated with uniform speed.

An inertial frame is endowed with the following characteristics:

All the fundamental laws of physics are valid in inertial frames.


All the fundamental laws of physics assume the same mathematical shape in all inertial frames.
Inertial frames are isotopic with respect to mechanical and optical experiments
The optical experiments performed in an inertial frame in any direction will always yield the same results.

Is earth an inertial frame of reference?

Earth rotates around its axis as also revolves around the sun. In both these motion, centripetal acceleration is present. Therefore,
strictly speaking earth or any frame of reference fixed on earth cannot be taken as an inertial fame. However, as we are dealing with
speeds x 108 ms-1 (speed of light) and speed of earth is only about 3 x 104 m/s, therefore when small time intervals are involved
effect of rotation and revolution of earth can be ignored. Furthermore, this speed of earth can be assumed to be constant. Hence
earth or any other frame of reference set up on earth can be taken as an approximately inertial frame of reference.
On the contrary, a frame of reference which is accelerated or decelerated is a non-inertial frame.
Other examples of inertial frames of reference are:
(i)

A frame of reference remaining fixed w.r.t. stars.

(ii)

A space ship moving in outer space, without spinning and with its engine cut off.

Apparent weight of a man inside a lift:-

(a) The lift possesses zero acceleration (fig-1): W = mg


(b) The lift moving upward with an acceleration a (fig-2):
W = mg + ma
= mg + mg
= 2 mg
(c) The lift moving downward with an acceleration a (fig-3):
W = mg ma
= mg mg
=0
Conceptual Problem:Problem 1:Suppose that you are standing on the balcony of a tall tower, facing east. You drop an object so that it falls to the ground below; see
below figure. Suppose also that you can locate the impact point very precisely. Will the object strike the ground at a, vertically below
the release point, at b to the east, or at c to the west? The object was released from rest; the Earth rotates from west to east.

Solution:If one assumes that the factor by which the Earth rotates is negligible during the time the object takes to reach the ground, then the
object will hit the ground at point a and one will not have to concern about theCarioles Effect.
In case the factor is non-negligible, and the Earth moves from West to East, the object will hit the ground at point c. The situation is
similar to the perceived leftward displacement of the air moving to a low pressure point from north to south. To the person standing
on the balcony of a tall tower, a psuedo force has acted on the object in a direction from east to west.
____________________________________________________________________________________________________
Problem 2:What is the distinction between inertial reference frames and those differing only by a translation or rotation of the axes?
Solution:One can distinguish the inertial frame of reference against the translation of the axes when the translation or rotation occurs nonuniformly.
If the translation of one frame of reference relative to the other is such that observer in one frame measures some acceleration of
the other, then the observer can draw a distinction between his frame and the inertial frame of reference.
It is important to note that the rotating frame will always be non-inertial because to account for the rotation there must be a change
in velocity vector. Therefore the inertial reference frame will always be distinct from the rotation frame.
However the translation can be distinguished from the inertial frame of reference only when the translation occurs at a uniform
velocity.
_____________________________________________________________________________________________________
Problem 3:-

A passenger in the front seat of a car finds himself sliding toward the door as the driver makes a sudden left turn. Describe the
forces on the passenger and on the car at this instant if the motion is viewed from a reference frame (a) attached to the Earth and
(b) attached to the car.
Solution:(a) From the reference frame attached to the Earth, when the car takes a left turn, the inertia of the passenger would maintain the
state of motion of the passenger (in accordance with Newtons first law of motion). This causes the passenger to move in the
direction of velocity vector of car before car took a turn.
Therefore, to the observer at Earth frame, the passenger has the initial velocity equal to the velocity of the car before it takes the
turn. The observer also see the car taking a turn, and accounts for an acceleration for the same. Thus, the observer would expect
frictional forces to exist between the passenger and the seat.

Given that the passenger has moved towards the door when the car took a left turn, it is obvious that the passenger moves to the
right. Therefore the frictional force would be in a direction opposite to the motion of the passenger, that is, towards the left.
If you assume that the frictional force between the passenger and seat is given by f , mass of passenger by m whereas the
acceleration by a. Then the equation depicting the motion of passenger relative to the observer in Earths frame is:
f = ma
Therefore the magnitude of frictional force between the passenger and the seat is equal to the magnitude of the deceleration of the
passenger.
(b) According to the observer in cars frame, the car experiences a centripetal acceleration when it turns to left. Therefore a psuedo
force will act on the passenger, accelerating him in the direction of the seat. But there is a friction between the seat of the car and
the passenger, which decelerate the passenger in a direction opposite to the direction of its motion.
Thus the magnitude of net force acting on the passenger is the difference between magnitude the centrifugal force and magnitude
the frictional force.
If you assume that the centrifugal force acting on passenger is F, the frictional force between the passenger and the seat is f , the
mass of passenger is m whereas the acceleration is a. Then the equation depicting the motion of passenger relative to the observer
in cars frame is:
F f = ma
Therefore one can see that the perceived acceleration for the observer in car differs from that of the observer on Earth.
__________________________________________________________________________________________________

Problem 4:Do you have to be concerned with the carioles effect when playing tennis or golf? If not, why not?

Solution:No, one does not have to concern with the Carioles Effect when playing tennis or golf because the factor by which the Earth rotates
during the time the ball goes from its source to the destination, is very small and can be neglected. Therefore the player can take
his/her shot as if the destination of the ball would be at the same position as it was at the time the shot was taken.

Friction:-

Whenever the surface of a body slides over another, each body experiences a contact force which always opposes the
relative motion between the surfaces. This contact force is called frictional force. Intermolecular interaction arising due to
elastic properties of matter is the cause of frictional force.This force acts tangentially to the interface of two bodies.
Cause of friction:Old view:Earlier it was thought that roughness of the two surfaces causes friction in the figure because it can be easily seen that smoother
the surfaces, lesser is the friction. Interlocking of irregularities of the two surfaces causes hindrance to sliding. This, however, is not
the current view.
Current view:The current view is a slight deviation from the old view. Earlier we thought that interlocking of irregularities of surfaces was causing
friction. Now, it is though that due to irregularities, the common surface area which is in actual contact of the two surfaces, is much
less than the total overall area in contact. In one experiment, it came out to be 1/10,000th of the apparent area.
Thus, while the total interactive (action and reaction) forces between the two surfaces remain the same, the pressures at the points
of contact are extremely high and cause the humps to flatten out (undergoing plastic deformation) until the increased area of contact

enables the upper solid to be supported. It is thought that at the points of contact, small, cold-welded joints are formed by the strong
adhesive forces between molecules which are very close together. These have to be broken away before one surface can move
over the other. Thus the force of friction is found to depend upon the following factors.

(i) The nature of two surfaces with the surfaces are being pressed together.
(ii) Normal force with the surfaces are being pressed together.
(iii) Actual area of contact
Types of Friction:There are four types of friction.
(a) Static Friction (b) Kinetic Friction

(c) Rolling Friction

(d) Fluid Friction

(a)Static friction:-

Static friction is the force of friction between two surfaces so long as there is no relative motion between them. It is always equal to
the applied force. The static frictional forces are incorporated in the following inequality.

The magnitude of static friction fs (static frictional force) has a maximum value fs,max that is given by,
fs,max = sN

Here s is the coefficient of kinetic friction and N is the normal force.


So, coefficient of static friction, s= fs,max /N
Static friction is always equal to the apllied force. It will be observe that value of static friction increases to certain maximum value,
beyond which if the applied force is increased body starts moving. This maximum value of force of friction is called limiting friction.
Limiting friction is the maximum value of force of friction between two surfaces so long as there is no relative motion
between them.
(b) Kinetic friction:Kinetic friction is the force of friction which comes into play between two surfaces when there is some relative motion between them.
The magnitude of force of kinetic friction fk (kinetic frictional force) is proportional to the normal force N.

So,

Here k is the coefficient of kinetic friction.


Thus, coefficient of kinetic friction, k = fk/N
Laws of limiting friction:(a) The direction of force of friction is always opposite to the direction of motion.
(b) The force of limiting friction depends upon the nature and state of polish of the surfaces in contact and acts tangentially to the
interface between the two surfaces.
(c) The magnitude of limiting friction F is directly proportional to the magnitude of the normal reaction R between the two surfaces
in contact, i.e.,
FR
(d) The magnitude of the limiting friction between two surfaces is independent of the area and shape of the surfaces in contact so
long as the normal reaction remains the same.
(c) Rolling friction:Force of friction which comes into play, between two surfaces, while one is rolling over the other is called rolling friction. Rolling
friction is similar to kinetic friction.
So,

Here r is the coefficient of rolling friction and N is the normal force.


(d) Fluid friction:Fluid friction is the opposing force which comes into play when a body moves through a fluid.
Cause and direction of rolling friction:-

A wheel of radius R rolling without sliding on a flat surface will experience a resistance due to the very small local deformation that
takes place, which is sometimes elastic sometimes inelastic, i.e., a kind of ridge is formed in front of the wheel as shown
exaggerated in the figure. This gives rise to the force FR, whose line of action passes through the center C of the wheel and P the
horizontal force necessary to force the wheel to topple over the point M, the total clockwise torque acting on the wheel about M must
be more than or marginally more than the total anticlockwise torque about M.
F = Frictional force
Therefore,
P R cos > mg R sin
P > mg tan
The value of "tan " is called the coefficient of rolling friction (g). This value does not depend upon R. If the two surfaces are
absolutely rigid, then no ridge will be formed and q will be zero i.e., coefficient of rolling friction will be zero.
Typical values are R = 0.006 for steel and 0.02 - 0.04 for rubber tiers on concrete surfaces. Rolling friction is very small compared
to the sliding friction. In case of pure rolling R = 0.

The value of angle of friction and that of angle of repose are same and the tangents of both of them is equal to the co-efficient of friction.

When a body slides down an inclined plane, whose angle of inclination with the horizontal is equal to angle of repose, it moves with uniform
velocity.

Sliding friction and rolling friction are independent of velocity.

Fluid friction depends upon velocity. It increases with an increase in velocity.

Force of limiting friction does not depend upon the size and shape of surface in contact.

Coefficient of kinetic friction is less than coefficient of static friction i.e.,k ?s


Co-efficient of Friction:According to the law of limiting friction,
F R
Or F = R

............ (1)

where is a constant of proportionality and is called the coefficient of limiting friction between the two surfaces in contact.
From (1),

Hence coefficient of limiting friction between any two surfaces in contact is defined as the ratio of the force of limiting friction and
normal reaction between them. The value of depends on
(i) nature of the surfaces in contact i.e., whether dry or wet; rough or smooth; polished or not polished.
(ii) material of the surfaces in contact.
For example, when two polished metal surfaces are in contact, 0.2, when these surfaces are lubricated, 0.1. Between two
smooth wooden surfaces, varies between 0.2 and 0.5. Obviously, has no units.
When a body is actually moving over the surface of another body, we place F by Fx, the kinetic friction, and and k.
Therefore,

k is then called the coefficient of kinetic or dynamic friction. As Fk < F, therefore, k is always less than i.e. coefficient of kinetic or
dynamic friction is always less than the coefficient of limiting friction.
Table gives the values of coefficient of limiting/kinetic friction between some pairs of materials:
S.No.

Surface in contact

Coefficient of limiting friction

Coefficient of kinetic friction

1.

Wood on wood

0.70

0.40

2.

Wood on leather

0.50

0.40

3.

Steel on Steel (mild)

0.74

0.57

4.

Steel on Steel (hard)

0.78

0.42

5.

Steel on Steel (greased)

0.10

0.05

Angle of Friction:-

The angle made by the resultant reaction force with the vertical (normal reaction) is known as the angle of the friction.
Now, in the triangle OAB,
AB/OB = cot
So, OB = AB/ cot
= AB tan
Or, tan = OB/AB
=f/N
So, tan = f / N = s
Angle of Repose:It is the angle which an inclined plane makes with the horizontal so that a body placed over it just begins to slide of its own accord.

Consider a body of mass m resting on an inclined plane of inclination . The forces


acting on the body are shown Ff being the force of friction. If friction is large enough, the body will not slide down.
Along x: mg sin f = 0
Along y: N mg cos = 0
i.e. N = mg cos and f = mg sin
Thus, f sN gives,
mg sin s mg cos
So, tan s. This signifies, the coefficient of static friction between the two surfaces, in order that the body doesnt slide down.
When is increased, then tan > . Thus sliding begins, and the angle r = tan-1. This angle is known as the angle of repose.
Methods of Reducing Friction:Friction can be reduced if we try to remove the cause of friction.
(a) By rubbing and polishing
(b) By lubricants
(c) By converting sliding into rolling friction
(d) By streamlining
Problem:A horizontal bar is used to support a 75-kg object between two walls, as shown in the below figure. The equal forces F exerted by
the bar against the walls can be varied by adjusting the length of the bar. Only friction between the ends of the bar and the walls
supports the system. The coefficient of static friction between bar and walls is 0.41. Find the minimum value of the forces F for the
system to remain at rest.

Concept:-

The diagram below shows the forces involved in the system:


The magnitude of frictional force between the walls and the bar is,
f = sN
Here s is the coefficient of static friction between the walls and the bar , and N is the normal force exerted by the wall on the bar.
If the bar is at rest, the horizontal equilibrium is maintained and the sum of the horizontal forces on the bar must be zero, that is

FN=0
F=N
Substitute F = N in equation f = sN ,
f = sN
=s F
To account for the vertical equilibrium of the block, the sum of the vertical forces must be zero,

f+fW =0
f+f=W
2f = W
It is important to note that the term on the left hand side of the above equation account for the fact that the frictional force exist at
both the ends of the bar and acts in a direction opposite to the direction of weight.
The weight of the block is calculated by multiplying the mass of the block with acceleration due to gravity,
W = mg
Substitute W = mg and f = sF in equation 2f = W,
2f = W
2(sF) = W
F = mg/2s
This equation can be used to calculate the magnitude of force exerted by the bar on the wall that will balance the system.
Solution:To calculate the magnitude of force F, substitute 75 kg for m , 9.81 m/s2 for g and 0.41 for s in equation F = mg/2s ,
F = mg/2s
= (75 kg) (9.81 m/s2)/2(0.41)
= (897.2 kg.m/s2) (1 N/1 kg.m/s2)
= 897.2 N
Round off to two significant figures,
F= 897.2 N
Therefore, the magnitude of the force exerted by the bar on the wall, to balance the system is 897.2 N.

Free body diagram


It is to be noticed here that during previous examples, we were using a concept called FBD implicitly which can now be brought to
you conscious attention.

It is like this. Whenever one attempts a problem involving forces and acceleration (say of dynamics or statics) one must show all
forces and acceleration (possible acceleration may be unknown also) on each part of the system treating that part separately (it is
called dividing system into possible subsystems). By all forces we mean external as well as internal forces (internal forces refer to
mutual reactions). Now each system is ready to get treatment of laws of motion e.g. acceleration, velocity.
Coming to actual situation one should first identify all the component involved in the system, say mass m1, mass m2 .........., pulley 1,
pulley 2, etc. Now separate them from others by cutting the string contacts (sort of imaginary separation) In effect, make them free
(that's where comes the name free body) from other components and at the same time show all the forces acting on it, external as
well as internal, arising due to separation from other parts. These are called mutual interaction forces (one of this type we have seen
in the illustration of IIIrd law) and show all possible acceleration. The diagram thus obtained is called a free body diagram
Illustration :
Let us draw FBD for various given systems
[Here we are assuming that all the surfaces strings and pulleys are ideal that is we are neglecting their masses & any friction
present]
FBD (1) : Block of mass M is resting on a frictionless rigid surface

There are only two forces in the system in figure above. mg which is the weight of the block, and is acting on the surface. R2 = mg
through its centre (since the body is symmetric). So, here itself from FBD we can see that net external force on the block is zero.
That is why, it is stationary on the surface.

FBD (2) : Draw the free body diagram of the block shown in figure 1.12.

Where R is reaction from the surface (R = Mg)


FBD (3) : Draw free body diagrams of both the blocks (figure 1.13 a), Assuming a reaction of magnitude 'R' is present at the
interface.

Now a question comes Can 'R' have a direction opposite to what is shown here? Answer is, of course it can have,

[before we proceed further let me point out that once you make calculations you will find that values of R will automatically come
negative, which tells us that earlier direction were the correct ones].
FBD (4) : Suppose situations is as shown in figure above that a light inextensible string pulls a block of mass M on a frictionless
rigid surface.

Here string is acting as a force transmitting element


It will experience a tension T in it. Let us cut it (imagine) at then the situation is as shown in figure shown below.

(We join it again then net force should become zero at that point. Since t, and T are oppositely directed their sum will come out to be
zero).
So tension t is responsible for dragging mass of block.
Therefore free body representation is

Caution : What happens if rope is not massless?


FBD (5) : Draw free body in case of system shown (figure shown below).

Suppose T is tension in the string, then by cutting it at 1-1' and 2-2' we can draw FBD's as shown in figure shown below.

R1 and R2 are reactions from plane, and from Newton's law they are equal to the weights of respective blocks.

FBD (6) : Draw FBD where one of the block is resting on an inclined plane and rope goes over a frictionless massless pulley
(Figure given below).

FBD becomes

Here the noticeable fact is that R2 is perpendicular to the inclined plane and mg is perpendicular to horizontal plane. Since Mg is
always directed downwards. R2 is due to the component of Mg, which is acting, perpendicular to inclined plane. (Here, the rule is
that reaction force is always normal to the surface, which provides the reaction).
Here for "ease" we can resolve force Mg in two components. One parallel to inclined plane and other on perpendicular to inclined
plane. As shown in figure below.

.. free Body, of the block on the inclined plane can be represented by figure shown below.

Finally at the end of this FBD, I want to point out that T2 will always be equal to T2for a continuous homogeneous massless
inextensible string passing over a massless frictionless pulley.
FBD (7) : Suppose both the block are on inclined planes as shown in figure given below.

For FBD cut the strings at 1-1' and 2-2' and separate the two blocks and pulleys as subsystems.

FBD (8) : If pulley is hanging from a rigid support say roof and masses are connected as shown in figure given below.

FBD's become.

Once we are comfortable in drawing FBD then we can proceed to write equation of motion, using Newton's II Law. But prior to that,
it is necessary to briefly introduce you with the concept of equilibrium of bodies.

Equilibrium:
An object is said to be in equilibrium if the vector sum of all the forces acting on the body (externally applied + forces arising due to
mutual interaction;) is zero.

That is

1+

2 +......+

N=0

But since it is impractical to apply it, as it is, to problems, therefore we resolve all forces in the three directions X, Y & Z equilibrium
and if some of the forces is zero in each direction then the body is said to be in equilibrium. (to be more specific - translator
equilibrium).
F1x + F2x + ............ + Fnx = 0
F1y + F2y + ............ + Fny = 0
F1z + F2z + ............ + Fnz = 0
Enquiry : How to apply equations of motion to any problem?
Writing Down Equations of Motion: Once we have made a free body diagram then we can write equations of motion for each part of
the system, for which we have drawn FBD. To write down equations of motion for a sub-system for which we have already drawn
the FBD the requirement is to choose two direction (if required we will need three directions) in which we shall work to reduce the
complexity say, direction X and Y

which in most of the cases is natural. In other cases, where we are dealing with

inclined planes we can fix our coordinate axis x-y in any desired orientation which reduces our trouble of finding out components of
the forces which are acting in various directions and by experience one knows that it is better to choose X axis parallel to the
inclined plane and Y axis perpendicular to it. For inclined plane

is the right choice of coordinate axis, tilted as per

inclination. Now once can go for writing F = ma equations in two directions for any ith sub-system.
Fx = miax

Fy = miay
Let us write down equation of motion for the FBD's we have already drawn.
Refer to FBD (2):

Here figure is given below.

Fy = R - Mg
It will be zero since we know that on the surface the body can't move upwards or downwards.
.. R - mg = 0

=> R = Mg and Fx = F

And this alone should accelerate it


F = Ma, => a = F/m
Refer to FBD (3): Here Figure is given below.

for the smaller block


Fy = R1 - mg = 0
Fx = F-R
For the Large Block
Fx = R

.. R1 = mg
.. F-R = m a1
Fy = R2 - Mg = 0 .. R2 = Mg
.. R = Ma2

Since both Blocks move as on system, their accelerations will be equal


a1 = a2
& R = Ma

.. F - R = ma
from here we can solve for R & a.

Refer to FBD (4): Here figure is given below.

For block
Fy = R - Mg = 0
.. R = mg
Fx = T = Ma
.. a = T/m
Refer to FBD (5): Here Figure is given below.
Here also, since system is fully connected and we assume that string is not loose, then the acceleration in different parts will be
equal. That is, Blocks m and M as well as the string will move with the same acceleration 'a'.

Equation for smaller Block


Fy = R1 - mg = 0
.. R1 = mg
Fx = T = ma ............ (1)
For Block - 'M'

Fy = R2 - Mg = 0

.. R2 = Mg
Fx = F - T = Ma ......... (ii)
solving (i) & (ii) we get T & a.
Refer to FBD (6): Here Figure is given below.
Since, system is fully connected by light inextensible string therefore acceleration in all the parts will be equal.

For Block 'm'


R1 - mg = 0 .. R1 = mg
T1 = ma.
For Block 'M' (Refer to the diagram made with mg cos & mg sin components.
R2 - Mg cos = 0 (since it is not moving to the plane)
R2 = Mg cos and is parallel to the plane
Mg sin - T2 = Ma

............ (ii)

It is already explained that T1 = T2 = T


.. solving (i) & (ii) we get 'T' & 'a' for the system.
Refer to FBD (7) : Here Figure is given below.
T1 = T2 = T (similar to the analysis done in FBD (6))

For block m1 perpendicular to inclined plane T1 - m1g cos = 0 R1 = m1g cos .


Parallel to the plane.
T-m1g sin = m1a ............. (1)
For block m2 perpendicular to plane
R2 - m2g cos = 0 R2 = m2g cos and m2 g sin - T = m2 a ...... (ii)
Solving (i) & (ii) we get 'a' & 'T.
Refer to FBD (8) : Here Figure is given below.

In Y-direction :
For block m1 T - m1g = m1a
For Block m2 m2g - T = m2 a
From (i) & (ii) we can solve for 'T' & 'a'.
On pulley 2T force is acting downwards. Therefore the pulley experiences a force F = 2T. Now you are well conversant with making
FBD's and writing down equations of motion for the given system. Therefore, now we can write down some definite steps to follow

for any given problem, which can considerably reduce your diversion or confusion or say the possibility of getting trapped into
complexity of a problem. (of course for that you have to use your brain all the time with these steps).
(i)

Draw the fully connected clear diagram.

(ii)

Define subsystems, that is parts of the system on which you will work to get your answers.

(iii)

Draw free body diagrams for all possible subsystems.

(iv)

Resolve forces as well as accelerations in x y & z direction. (or perpendicular and || to the plane whenever require).

(v)

Write, Fx, Fy, Fz equations with the physical constraints appearing in the problem.

(vi)

Eliminate some variables and get the required one(s).

Centripetal Force
If a body is moving with a constant speed in a circle, as seen from an inertial frame, it is continuously towards the centre of rotation
with magnitude vr/r (known as centripetal acceleration), where v is the speed of the particle and r is the radius of the circular path.
According to Newton's second law, this body will experience net force directed towards the centre called the centripetal force.
Therefore, net force acting on the body towards the centre = mv2/r, where m is mass of body.
Centrifugal force is a pseudo force acting on the body from a rotating frame.
Illustration:
A bob of mass m is suspended form a inextensible, massless describe a horizontal uniform circular motion as shown in figure 1.42.
about a vertical. Analyze the dynamics of this system.

Solution:
The path of the system described above is shown in figure 1.42. Let the radius of circular path of bob is r equal to l sin and
tension in string is T. The string makes an angle with the vertical. Consider the bob is at A. We can draw the free body diagram of
bob at a as shown in figure 1.43. The force acting on the bob is it's weight mg and tension T of the string. Tenstion T is resolved in
two components T cos and T sin as shown in figure 1.43. we can write the equation of motion

T cos = mg

T sin = mv2/r

Banking of Roads:-

Perhaps you have noticed that when a road is straight, it is horizontal too. However, when a sharp turn comes, the surface of the
road does not remain horizontal. This is called banking of the roads.

Purpose of banking:-

Case I: If coefficient of friction, = 0:What we really wish is that even if there is no friction between the tyres and the road, yet we should be able to take a round turn. In
the given figure Vertical N cos component of the normal reaction N will be equal to mg and the horizontal N sin component will
provide for the necessary centripetal force. [Please note that as we are assuming to be zero here, the total reaction of the road will
be the normal reaction.] Frictional forces will not act in such a case.

Thus, N cos = mg

.......... (i)

N sin = mv /r

.......... (ii)

Dividing equation (ii) by (i), we get


tan = v2/rg

where is the angle of banking.


Case - II : If coefficient of friction, 0:In the above figure shows a section of the banked road and the view of the vehicle form the rear end.
vehicle-form-the-rear-end
The total forces acting are,
N1 and N2 = normal reactions
f1 and f2 = frictional forces
mg = weight
r = radius
= angle of banking
Let N = Resultant of N1 and N2.
f = Resultant of f1 and f2.
Let us resolve all the forces horizontally and vertically. As the vehicle has Equilibrium in vertical direction.
so, N cos + f sin = mg

............ (i)

The resultant of horizontal components i.e., (f cos + N sin ), however, this becomes the net external force acting on the vehicle in
the radially inward direction of the round-turn. This thus provides for the necessary centripetal force (mv2/r).
Therefore, f cos + N sin = mv2/r

............ (ii)

Further, if is the coefficient of friction, we have


f = N

............ (iii)

These are the three basic equations from which, we can find out whatever we want to find out.'

Putting (iii) in (i) gives


N cos = N sin + mg
=>N(cos - sin ) = mg
=> N = mg/cos - sin

...............(iv)

Putting (iii) and (iv) in (ii) gives


mgcos /(cos - sin ) + mgsin/(cos - sin ) = mv2/r
=>

mgr cos + mgr sin = mv2 cos - mv2 sin

Thus, tan = (v2 - rg)/(rg + v2)

............... (A)

or

v2 = rg(+ tan)/(1-tan)

(A)

Gives the angle of banking for the maximum velocity v and (B) gives the value of the maximum velocity which the vehicles

................ (B)

should be allowed on a road banked at an angle .


Notes : 1. The value of is the minimum value of required. The value of v is the maximum allowable velocity.
Therefore, the best angle of banking so that there is absolutely nil wear and tear due to frictional force for the given values of v
and r can be determined by putting = 0 in this formula.
If we put = 0 in formula (A), we get
tan = v2/rg
Further, for zero frictional wear and tear, the velocity for the given values of and r will be v = rgtan.
Skidding:-

Let us consider the situation in the figure. You are cycling fast on road I. You then want to take a turn to go to road II. However, due
to big leak of mobile oil from some truck, the portions of the roads within the area ACBD have become slippery. You do not know
about it. You are cycling fast. When you reach the line AC, you turn the handle mounted on the front wheel towards road II. What
will happen? Will you be able to take the turn? No, you won't be. Although your front wheel is aligned to go towards road II, you still
continue to go straight to road III. This is called skidding. You will skid.

The tendency to slip transverse (transverse means across) to the intended line of run, is called skidding. Thus as soon as you turn
the handle of your cycle (or the driving wheel in a motor car), skidding will try to occur. If there is enough friction, this start of
skidding brings into action a frictional force between the road and the bottom surface of the wheels of the vehicle. This frictional
force then provides for the necessary centripetal force required to negotiate the turn. If friction is not enough, skidding will start
which will not let you take the turn in a normal way. Skidding will also cause additional friction wear and tear of the tyres of your
vehicle.
How to avoid skidding?
Let us consider the situation given in the figure. Let r is the radius of turn which you have to take. N1 and N2 are normal reactions mg
the weight and F1, F2 the frictional forces on the inside and outside wheels.

So, (N1 + N2) = mg


Thus, F1 + F2 = (N1 + N2) = gm

......... (i)
......... (ii)

This must be greater than or equal to the centripetal force required.


Therefore, mg > mv2/r
or, v < rg
or, Vmax = rg

If the velocity of the vehicle is more than rg, it will skid.


Overturning:You may have seen overturned trucks lying on the road. Such heavily loaded trucks! Who could have overturned them!! Overturning
occurs on the roads when the trucks try to change directions, take sharp turns. Overturning occurs, more after, in case of vehicles
which have greater height or whose center of gravity are much high up from the surface of roads.
Let us first consider why over turning would take place at all. Suppose a heavily loaded truck is going straight. Suddenly it takes a
sharp turn towards its left. Now what actually happens is that while the upper portion of the truck still tends to go straight because of

its inertia (Newton's first law of motion), the lower portion starts going towards left because you have turned the driving wheel
accordingly.
Thus, if the inertial forces on the upper portion are much height, they provide so much torque on the truck at its center of gravity that
overturning takes place.
Thus overturning always takes place by lifting off the inner wheels from the ground on the curved path.
Limiting case when a four wheeler just begins to overturn on a plain horizontal road:Let, Mg = weight
N = Total normal reaction = N1 + N2
F = Total friction force
v = velocity
r = radius of the round
2a = distance between inner and outer wheels.
G = center of gravity
h = height of center of gravity from Earth. Frictional force F will provide for the necessary centripetal force.

Therefore, F = mv2/r

............... (i)

When the vehicle just begins to overturn, the inner wheels will just begin to lift off from the ground. Their pressure on ground will
become zero, so the reaction N1 on the inner wheels will become zero.
Thus, N1 = 0
N1 + N2 = N2 = mg
Let us take moments about G
N2 a = F h
Putting (i) and (iii) in (iv) gives
mg a = mv2/r h, or v
i.e., vmax = rga/h
If speed goes beyond it, the vehicle will overturn.
Minimum required to prevent overturning
We know frictional force is N.
=>

F = N = mg

Putting equation (v) in (i) gives

............ (v)

mg = mv2/r,
Or = min = v2/rg
F = mv2/r = m2r

Centripetal force is not a new kind of force. It is the radial component of the net force acting on the particle moving along a circle.
Centrifugal force is a type of pseudo force used by an observer moving in a circle. Numerically, it is equal to the centripetal force but
is oppositely directed if observer and the body both are moving on same circle as a single unit.

The banking angle is the angle at which the vehicle is inclined about its longitudinal axis with respect to the plane of its curved path.
If the force of friction is not strong enough, the vehicle will skid.
Even if there is very little force of friction the vehicle can still go round the curve with no tendency to skid.
Roads are banked because of the inertia of vehicles driving on the road.
Problem 1:A curve has a radius of 50 meters and a banking angle of 15. What is the ideal, or critical, speed (the speed for which no friction is
required between the car's tires and the surface) for a car on this curve?
Solution:-

Here, radius of curve, r = 50 m


banking angle, = 15
free-fall acceleration, g = 9.8 m/s2
We have to find out the ideal speed v (the speed for which no friction is required between the car's tires and the surface)
From the free-body diagram for the car:Fnet = Fcentripital
mg tan = mv2/r
v2 = rg tan
v = rg tan

= (50 m) (9.8 m/s2) (tan 15) = 11 m/s


If the car has a speed of about 11 m/s, it can negotiate the curve without any friction.
____________________________________________________________________________________________
Problem 2:A 1200 kg automobile rounds a level curve of radius 200 m, on a unbanked road with a velocity of 72 km/hr. What is the
minimum co-efficient of friction between the tyres and road in order that the automobile may not skid. (g = 10 m/s2)
Solution:In a unbanked road, the centripetal force is provided by the frictional force.
So, ffriction = mv2/r But flimiting friction > ffriction
or, mg = ffriction or mg = mv2/r
So, min = v2/gr = (2020)/(10200) = 0.2.
The bank angle is the angle at which the vehicle is inclined about its longitudinal axis with respect to the plane of its curved path.
If the force of friction is not strong enough, the vehicle will skid.
even if there is very little force of friction the vehicle can still go round the curve with no tendency to skid.
Roads are banked because of the inertia of vehicles driving on the road.
________________________________________________________________________________________
Problem 3:A turn of radius 100 m is being designed for a speed of 25 m/s. At what angle should the turn be banked?
Solution:-

Here, radius of turn, r = 100 m


speed of the car, v = 25 m/s
free-fall acceleration, g = 9.8 m/s2
We have to find out the bank angle, .
From the free body diagram of the car,
Fnet = Fcentripetal
mg tan = mv2/r
tan = v2/rg
= tan-1(v2/rg)

= tan-1 [(25 m/s)2/ (100 m) (9.8 m/s2)] = 33


So, the banking angle should be about 33.

PROBLEMS
Problem 1:A 26-ton Navy jet as shown in the below figure requires an air speed of 280 ft/s for lift-off. Its own engine develops a thrust of 24,000
lb. The jet is to take off from an aircraft carrier with a 300-ft flight deck. What force must be exerted by the catapult of the carrier?
Assume that the catapult and the jets engine each exert a constant force over the 300-ft takeoff distance.

Concept:Force acting on the body (F) is equal to the product of mas of the body (m) and acceleration of the body (a).
So, F = ma
Weight W of the object is equal to the mass m of the object times of the free fall acceleration g.
W = mg
So the mass m of the object would be,
m = W/g
To obtain force F in terms of weight W, substitute W/g for m in the equation F = ma,
F = ma = (W/g) (a)
Time t taken by a body to travel a distance x with average velocity vav will be,
t = x/ vav
The deceleration a is equal to the rate of change of velocity,
a = v/t
Solution:To find the time t for the plane to travel 300 ft, substitute 300 ft for x and 140 ft /s for vav in the equation t = x/vav,
t = x/ vav = (300 ft)/(140 ft/s) = 2.14 s
To obtain the acceleration a, substitute 280 ft/s for v and 2.14 s for t in the equation
a = v/t,

a = v/t = (280 ft/s)/(2.14 s) = 130 ft/s2


To find out the net force F on the plane, substitute 52,000 lb for W, 130 ft/s2 for a and 32 ft/s2 for free fall acceleration g in the
equation F =(W/g) (a),
F =(W/g) (a) = (52000 lb)( 130 ft/s2)/( 32 ft/s2) = 2.1105 lb
The force exerted by the catapult Fc is equal to the difference of the net force F on the plane and the thrust develop by the own
engine T.
So, Fc = F-T
To obtain the force exerted by the catapult Fc, substitute 2.1105 lb for F and 24,000 lb for T in the equation Fc= F-T,
Fc = F-T = (2.1105 lb) - (24,000 lb) = (2.1105 lb) - (2.4104 lb) =1.86105 lb
From the above observation we conclude that, the force exerted by the catapult Fc would be 1.86105 lb.
_________________________________________________________________________________________________________
___________
Problem 2:A 77-kg person is parachuting and experiencing a downward acceleration of 2.5 m/s2 shortly after opening the parachute. The mass
of the parachute is 5.2 kg. (a) Find the upward force exerted on the parachute by the air. (b) Calculate the downward force exerted
by the person on the parachute.

Concept:Force acting on the body (F) is equal to the product of mas of the body (m) and acceleration of the body (a).
So, F = ma
From equation F = ma, the acceleration (a) of the body would be,
a = F/m
Weight W of the object is equal to the mass m of the object times of the free fall acceleration g.
W = mg
Solution:(a) The net force Fnet on the system is equal to the sum of force exerted by the person and force exerted by the parachute.
So, Fnet = (mpe+mpa) (a)

Here, mpe is the mass of person, mpa is the mass of parachute and a is the downward acceleration.
To obtain the net force Fnet on the system, substitute 77 kg for mpe , 5.2 kg for mpa and
-2.5 m/s2 for a in the equation Fnet = (mpe+mpa) (a),
Fnet = (mpe+mpa) (a) = (77 kg + 5.2 kg) (-2.5 m/s2)
= (-210 kg,m/s2) (1 N/1 kg,m/s2) = -210 N
The weight W of the system will be,
W = (mpe+mpa) (g)
To obtain the weight W of the system, substitute 77 kg for mpe , 5.2 kg for mpa and 9.81 m/s2 for free fall acceleration g in the
equation W = (mpe+mpa) (g),
W = (mpe+mpa) (g) = (77 kg + 5.2 kg) (9.81 m/s2)
= (810 kg,m/s2) (1 N/1 kg,m/s2) = 810 N
If P is the upward force of the air on the system (parachute) then,
P = Fnet +W = (-210 N)+ (810 N) = 600 N
From the above observation we conclude that, the upward force exerted on the parachute by the air would be 600 N.
(b) The net force Fnet on the parachute will be,
Fnet = mpa a
Here, mpa is the mass of parachute and a is the downward acceleration.
To obtain the net force Fnet on the parachute, substitute 5.2 kg for mpa and -2.5 m/s2 for a in the equationFnet = mpa a,
Fnet = mpa a = (5.2 kg)(-2.5 m/s2)
= (-13 kg.m/s2) (1 N/1 kg,m/s2) = -13 N
The weight W of the parachute will be,
W = (mpa) (g)
To obtain the weight W of the system, substitute 5.2 kg for mpa and 9.81 m/s2 for free fall acceleration g in the equation W = (mpa)
(g),
W = (mpa) (g)
= ( 5.2 kg) (9.81 m/s2)= (51 kg,m/s2) (1 N/1 kg,m/s2)
= 51 N
If D is the downward force of the person on the parachute then,
D = - Fnet-W+P
= -(-13 N)-(51 N)+(600 N) = 560 N
_________________________________________________________________________________________________________
_________
Problem 3:A 1400-kg jet engine is fastened to the fuselage of a passenger jet by just three bolts (this is the usual practice). Assume that each
bolt supports one-third of the load. (a) Calculate the force on each bolt as the palne waits in line for clearance to take off. (b) During
flight, the palne encounters turbulence, which suddenly imparts an upward vertical acceleration of 2.60 m/s2 to the palne. Calculate
the force on each bolt now. why are only three bolts used? See below figure.

Concept:Force acting on the automobile (F) is equal to the product of mas of the automobile (m) and acceleration of the auto mobile (a).
So, F = ma
Weight of the automobile (W) is equal to the product of mass of the automobile (m) and acceleration due to gravity on the surface of
the earth (g = 32 ft/s2).
W = mg
Solution:(a) First we have to find out the weight W of the engine.
To obtain the weight of the engine W, substitute 1400 kg for mass m and 9.81 m/s2 for g in the equation W =mg,
W = mg = (1400 kg) (9.81 m/s2)
= 1.37104 kg.m/s2 = (1.37104 kg.m/s2) (1 N/1 kg.m/s2) = 1.37104 N
As each bolt supports 1/3 of this force, thus the force F on a bolt will be,
F = 1.37104 N/3 = 4600 N
From the above observation we conclude that, the force on each bolt would be 4600 N.
(b) To find out the force f on each bolt, first we have to find out the upward force Fup on the bolt. Again to obtain the upward
force Fup on the bolt, we have to obtain the net force Fnet on the engine.
To obtain the net force Fnet on the engine, substitute 1400 kg for mass of the jet engine m and 2.60 m/s2 for acceleration a in the
equation Fnet = ma,
Fnet = ma =(1400 kg) (2.60 m/s2) =3.64103 kg.m/s2
=(3.64103 kg.m/s2) (1 N/1 kg.m/s2) =3.64103 N
So the upward force Fup from the bolts will be equal to the sum of net force Fnet on the engine and weight of the engine W.
Fup = Fnet + W
To obtain the upward force Fup from the bolts, substitute 3.64103 N for Fnet and 1.37104 N for W in the equation Fup = Fnet + W,
Fup = Fnet + W = (3.64103 N) + (1.37104 N) = 1.73104 N
The force per bolt f will be equal to the 1/3 of the upward force Fup from the bolts.

So, f = Fup/3.
To obtain the force per bolt f, substitute 1.73104 N for Fup in the equation f = Fup/3,
f = Fup/3 = (1.73104 N)/3
= 5800 N
From the above observation we conclude that, the force on each bolt would be 5800 N.
_________________________________________________________________________________________________________
_________
Problem 4:A research ballon of total mass M is descending vertically with downward acceleration a as shown in below figure. How much ballast
must be thrown from the car to give the ballon an upward acceleration a, assuming that the upward lift of the air on the ballon does
not change?

Solution:Let us consider initially mass of the system (balloon) is, M.


So the force (f) acting on the system having downward acceleration, a will be,
f = -Ma

(1)

And the weight of the system (W) will be,


W = Mg

(2)

Where, g is the free fall acceleration of the system.


Therefore the upward force acting on the system (F) will be,
F = W + f = Mg+(-Ma)
F = Mg-Ma

(3)

Again let us consider m is the mass of single ballast which is thrown from the balloon.
Now mass of the system is, M-m.
So the force (f1) acting on the system having upward acceleration, a and mass, M-m will be,
f1 = (M-m) a

(4)

And the weight of the system (W1) having mass M-m will be,

W1 = (M-m) g

(5)

Where, g is the free fall acceleration of the system.


To give the balloon an upward acceleration a, the upward force (F) must be equal to the addition of the force (f1) acting on the
system having upward acceleration, a and mass, M-m and the weight of the system (W1) having mass M-m will be.
So the equation will be,
f1+W1 = F

(6)

(M-m) a + (M-m) g = Mg-Ma


Ma-ma +Mg-mg = Mg-Ma
ma + mg = Ma+ Mg Mg + Ma
m(a+g) = 2Ma
m = 2Ma/(a+g)

(7)

From equation (7) we observed that, 2Ma/(a+g) mass of ballast must be thrown from the car to give the balloon an upward
acceleration a.
_________________________________________________________________________________________________________
__________
Problem 5:A childs toy consists of three cars that are pulled in tandem on small frictionless rollers as shown in below figure. The cars have
masses m1 = 3.1 kg, m2 = 2.4 kg, and m3 = 1.2 kg. If they are pulled to the right with a horizontal force P = 6.5 N, find (a) the
acceleration of the system, (b) the force exerted by the second car on the third car, and (c) the force exerted by the first car on the
second car.

Solution:Given Data:
Mass of the first car, m1 = 3.1 kg
Mass of the second car, m2 = 2.4 kg
Mass of the third car, m3 = 1.2 kg
Horizontal force on the car, P = 6.5 N
Acceleration a body is equal to the force exerted on the body divided by mass of the body.
(a) The acceleration of the system is equal to the total horizontal force acting on the system divided by total mass of the system.
So, a = F/(m1+ m2+ m2) = (6.5 N)/(3.1 kg+2.4 kg+1.2 kg)
= (6.5 N)/(6.7 kg) = (0.97 N/kg) (1 kg. m/s2/1N) = 0.97 m/s2

Therefore the acceleration of the system would be 0.97 m/s2.


(b) Force exerted by the second car on the third car would be,
F32 = (mass of third car) (total acceleration of the system) = m3 a = (1.2 kg) (0.97 m/s2)
= (1.164 kg. m/s2) (1 N/ 1 kg. m/s2) = 1.164 N
Rounding off to two significant figures, the force exerted by the second car on the third car would be 1.2 N.
(c) Force exerted by the first car on the second car would be,
F32 = (sum of mass of second car and third car) (total acceleration of the syste) = (m2+ m3) a
= (2.4 kg +1.2 kg) (0.97 m/s2)= (3.492 kg. m/s2) (1 N/ 1 kg. m/s2) = 3.492 N
Rounding off to two significant figures, the force exerted by the first car on the second car would be 3.5 N.
_________________________________________________________________________________________________________
__________
Problem 6:A landing craft approaches the surface of Callisto, one of the satellite (moons) of the planet Jupiter as shown in the below figure. If
an upward thrtust of 3260 N is supplied by the rocket engine, the craft descends with constant speed. Callisto has no atomsphere. If
the upward thrust is 2200 N, the craft accelerates downward at 0.390 m/s2. (a) What is the weight of the landing craft in the vicinity
of Callistos surface? (b) What is the mass of the craft? (c) What is the acceleration due to gravity near the surface of Callisto?

Concept:Force acting on the automobile (F) is equal to the product of mas of the automobile (m) and acceleration of the auto mobile (a).
So, F = ma
So mass m of the body will be,
m = F/a
Weight of the automobile (W) is equal to the product of mass of the automobile (m) and acceleration due to gravity on the surface of
the earth (g = 32 ft/s2).
W = mg
So, g = W/m

Solution:(a) As the craft descends with constant speed, so the net force will be equal to zero. Thus the thrust balances weight. As the upward
thrust of 3260 N is supplied by the rocket engine, therefore the weight of the landing craft in the vicinity of Callistos surface will be
3260 N.
(b) An upward thrust of 3260 N is supplied by the rocket engine; the craft descend with constant speed. If the upward thrust is 2200
N, the craft accelerates downward at 0.390 m/s2.
So the net force F will be,
F = 2200 N 3260 N = -1060 N
To obtain the mass m of the craft, substitute -1060 N for F and -0.390 m/s2 for acceleration a (negative sign due to downward
direction) in the equation m = F/a,
m = F/a = (-1060 N)/(0.390 m/s2) = (2720 N/(m/s2)) (1kg.m/s2 /1 N) = 2720 kg
From the above observation we conclude that, the mass m of the craft will be 2720 kg.
(c) To find out the acceleration due to gravity g near the surface of Callisto, substitute 3260 N for W and 2720 kg for m in the
equation g = W/m,
g = W/m = 3260 N/2720 kg = (1.20 N/kg) (1 kg.m/s2 /1 N) = 1.20 m/s2
From the above observation we conclude that, the acceleration due to gravity g near the surface of Callisto would be 1.20 m/s2.
_________________________________________________________________________________________________________
_______
Problem 7:Two blocks are in contact on a frictionless table. A horizonatl force is applied to one block, as shown in the below figure. (a)If m1 =
2.3 kg, m2 = 1.2 kg, and F = 3.2 N, find the force of contact between the two blocks. (b) Show that If the same force F is applied to
m2 rather than to m1, the force of contact between the blocks is 2.1 N, which is not the same value derived in (a). Explain.

Concept:Force acting on the body (F) is equal to the product of mas of the body (m) and acceleration of the body (a).So, F = ma
From equation F = ma, the acceleration (a) of the body would be,
a = F/m
Solution:(a) The acceleration a of the two block will be,
a = F/(m1+m2)

Here F is the horizontal force, m1 is the mass of the block 1 and m2 is the mass of the block 2.
The net force Fnet on block 2 is from the force of contact, and will be,
Fnet = m2a
= F m2/(m1+m2)

(Since, a = F/(m1+m2))

To obtain the force Fnet of contact between the two blocks, substitute 3.2 N for F,1.2 kg for m2 and 2.3 kg form1 in the
equation Fnet = F m2/(m1+m2),
Fnet = F m2/(m1+m2)
= (3.2 N) (1.2 kg)/(2.3 kg+1.2 kg) = 1.1 N
From the above observation we conclude that, the force Fnet of contact between the two blocks would be 1.1 N.
(b) The acceleration a of the two block will be,
a = F/(m1+m2)
Here F is the horizontal force, m1 is the mass of the block 1 and m2 is the mass of the block 2.
The net force Fnet on block 1 is from the force of contact, and will be,
Fnet = m1a
= F m1/(m1+m2)

(Since, a = F/(m1+m2))

To obtain the net force Fnet on block 1, substitute 3.2 N for F,1.2 kg for m2 and 2.3 kg for m1 in the equationFnet = F m2/(m1+m2),
Fnet = F m1/(m1+m2)
= (3.2 N) (2.3 kg)/(2.3 kg+1.2 kg) = 2.1 N
From the above observation we conclude that, if the same force F is applied to m2 rather than to m1, the force of contact between
the blocks would be 2.1 N.
_________________________________________________________________________________________________________
_________
Problem 8:A block of mass M is pulled along a horizontal frictionless surface by a rope of mass m, as shown in the below figure. A horizontal
force

is applied to one end of the rope. Assuiming that the sag in the rope is negligible, find (a) the acceleration of rope and

block, and (b) the force that the rope exerts on the block.

Concept:Force acting on the body (F) is equal to the product of mas of the body (m) and acceleration of the body (a).
So, F = ma
From equation F = ma, the acceleration (a) of the body would be,
a = F/m
Solution:(a) Treat the system as including both the block and the rope.

Thus the mass of the system will be M+m.


Here M is the mass of the block and m is the mass of the rope.
As there is one horizontal force which is acting to one end of the rope, so,

In accordance to Newtons second law, the horizontal force P will be,


P = (M+m) ax
Here ax is the horizontal acceleration of the rope and block.
From equation P = (M+m)ax, the horizontal acceleration ax of the rope and block will be,
ax = P/(M+m)
From the above observation we conclude that, the horizontal acceleration ax of the rope and block would be P/(M+m).
(b) Now consider only the block. The horizontal force does not act on the block, instead there is
the force of the rope on the block. We will assume that, the magnitude of the force is R, and
this the only relevant force on the block.
Thus the net force on the block will be,

In this case Newtons second law would be written as,


R = Max
Here the horizontal acceleration ax of the block is equal to the acceleration of the block and rope system andM is the mass of the
block.
To obtain the force R that the rope exerts on the block, substitute P/(M+m) for acceleration ax in the equation R = Max we get,
R = Max= m [P/(M+m)] = [M/(M+m)] P
From the above observation we conclude that, the force R that the rope exerts on the block would be [M/(M+m)] P.
_________________________________________________________________________________________________________
_________
Problem 9:A light beam from a satellite-carried laser strikes an object ejected from an accidently launched ballistic missile; see below figure.
The beam exerts a force of 2.710-5 N on the target. If the dwell time of the beam on the target is 2.4 s, by how much is the object
displaced if it is (a) a 280-kg warhead and (b) a 2.1-kg decoy? (These displacements can be measured by observing the reflected
beam.)

Concept:Force acting on the automobile (F) is equal to the product of mas of the automobile (m) and acceleration of the auto mobile (a).
So, F = ma
From the above equation F = ma, the acceleration a will be,
a = F/m
In accordance to equation of motion, the distance y travelled by the body will be,
y = ut + at2
Here u is the initial velocity, t is the time, and ais the acceleration.
Since the object ejected from an accidentally launched ballistic missile, so the initial velocity u will be equal to zero.
So, u = 0
Substitute 0 for u in the equation y = ut + at2,
y = ut + at2 = 0t + at2 = at2
Solution:(a) To find the displacement y which is the object displaced, first we have to find out the acceleration a.
To obtain the acceleration a, substitute 2.710-5 N for F and 280 kg for m in the equation a = F/m,
a = F/m
= (2.710-5 N)/280 kg = (9.6410-8 N/kg) (1 kg.m/s2 /1 N) = 9.6410-8 m/s2
To find the displacement y which is the object displaced from the original trajectory, substitute 9.6410-8m/s2 for a and 2.4 s for t in
the equation y = at2,
y = at2
= (9.6410-8 m/s2) (2.4 s)2 = 2.810-7 m
From the above observation we conclude that, the displacement y which is the object displaced from the original trajectory would be
2.810-7 m.
(b) To find the displacement y which is the object displaced, first we have to find out the acceleration a.

To obtain the acceleration a, substitute 2.710-5 N for F and 2.1 kg for m in the equation a = F/m,
a = F/m
= (2.710-5 N)/2.1 kg = (1.310-5 N/kg) (1 kg.m/s2 /1 N) = 1.310-5 m/s2
To find the displacement y which is the object displaced from the original trajectory, substitute 1.310-5 m/s2for a and 2.4 s for t in
the equation y = at2,
y = at2
= (1.310-5 m/s2) (2.4 s)2 = 3.710-5 m
From the above observation we conclude that, the displacement y which is the object displaced from the original trajectory would
3.710-5 m.
_________________________________________________________________________________________________________
________
Problem 10:What strength fishing line is needed to stop a 19-lb salmon swimming at 9.2 ft/s in a distance of 4.5 in.?
Concept:We have to calculate the force acting on the salmon.
Force (F) acting on the body is equal to the mass of the body (m) times acceleration (a) of the body.
F = ma
In terms of weight (W = mg) the above equation (F = ma) will be,
F = ma
= (ma) (g)/g = (mg)a/g = Wa/g
Solution:Initial speed (vi) of the salmon is, vi = 9.2 ft/s
Final speed (vf) of the salmon is, vf = 0 ft/s
So the average speed of the salmon will be, vav = (vi + vf) /2
= (0 ft/s+9.2 ft/s)/2 =

= 4.6 ft/s

Time required (t) to stop the salmon is equal to the distance travelled (x) by the salmon divided by the average velocity (vav) of the
salmon.
So, t = x/ vav
To obtain the time required (t) to stop the salmon, substitute 4.5 in for x and 4.6 ft/s for vav in the equation t= x/ vav,
t = x/ vav
=(4.5 in)/(4.6 ft/s) =(4.5 in0.0833 ft/1 in)/(4.6 ft/s) = (0.38 ft) /(4.6 ft/s) = 8.310-2 s
The deceleration a of the salmon will be, a = v/t.
To find out the deceleration a of the salmon, substitute 9.2 ft/s for v and 8.310-2 s for t in the equation a= v/t,
a = v/t
= (9.2 ft/s)/(8.310-2 s) =110 ft/s2
To find out the force on the salmon, substitute 19-lb for W, 110 ft/s2 for a, and 32 ft/s2 for g in the equation F= Wa/g,
F = Wa/g
= (19-lb) (110 ft/s2)/ (32 ft/s2) = 65.31 lb
Rounding off to two significant figures, the force will be 65-lb.

From the above observation we conclude that, the force required to stop the 19-lb salmon swimming at 9.2 ft/s in a distance of 4.5 in
would be 65-lb.
_________________________________________________________________________________________________________
_________
Problem 11:A jet plane starts from rest on the runway and accelerates for takeoff at 2.30 m/s2. It has two jet engines, each of which exerts a
thrust of 1.40 105 N. What is the weight of the plane?

Solution:Given Data:
At takeoff time, acceleration of the jet plane, ax = 2.30 m/s2
Thrust of each jet engine, Fx = 1.40105 N
Jet plane consist two jet engines. So the total force exerts by the get plane will be,
Fx = 2(1.40105 N)
= 2.80105 N

(1)

Force exerted on a body is equal to the mass of the body times acceleration of the body.
So, Fx = max

(2)

So from equation (2),


m = Fx /ax

(3)

Putting the value of Fx and ax in equation (3), mass of the jet plane will be,
m = Fx /ax
= 2.80105 N/2.30 m/s2 = (1.22 105 N/m/s2) (1 kg. m/s2 / 1 N) = 1.22 105 kg

. (4)

Weight of a body is equal to the mass of the body times (m) acceleration due to gravity (g) on that surface.
W = mg (5)
We know that that, the value of acceleration due to gravity (g) on the surface of earth is 9.81 m/s2.
So putting the value of m and g in equation (5), weight of the jet plane would be,
W = mg
= (1.22105 kg) (9.81 m/s2) = (11.9682105 kg. m/s2) (1 N/1 kg. m/s2)

= 1.19682106 N

(6)

Rounding off to three significant figures, weight of the jet plane would be 1.20106 N.

Conservation of Linear Momentum:It states that,


In an isolated system (no external force), the algebraic some of the momenta of bodies, along any straight line, remains
constant and is not changed due to their mutual action and reaction on each other.
This can be verified by a following simple experiment.
Consider a body A of mass m1 moving with a velocity

strike against another body B of mass m2,moving with velocity in same

direction as shown in the below figure. Two bodies remain in contact with each other for small time ?t. They get separated and
move with velocities

Let

and

after collision.

be the force exerted by A upon B and

be its reaction. Since the system is isolated, i.e., no external force is

there,

So,
... (1)
This is in accordance with Newtons third law of motion that action and reaction are equal and opposite.
Considering the momenta of the bodies before and after collision.
Body A

Body B

Momentum of A before collision =

Momentum of A before collision =

Momentum of A after collision =

Momentum of B after collision =

Change in momentum of A =

Change in momentum of B =

Time taken for the change of momentum =?t

Time taken for the change of momentum =?t

Rate of change of momentum of A (=Force on A) =

Rate of change of momentum of B (=Force on B)=

So,

So,
Substituting for

and

in equation (1),

Or,
Thus, the total momentum of the system before collision is equal to the total momentum of the system after collision.

This verifies the law of conservation of momentum.


It may be noted that the conservation of momentum is closely connected with the validity of Newtons third law of motion, since we
have used equation (1) [which is nothing but third law] to prove it.
Alternative Method:According to Newtons second law of motion,

Since

(momentum of body),

Incase of an isolated system,

Thus,

or,
Therefore, momentum (in vector form) of an isolated system remains constant. This is in accordance with the law of conservation of
momentum.
IMPORTANT NOTE:-

While applying law of conservation of momentum to a system following consideration must be kept in mind:
(a) The system must be isolated.
(b) While finding the algebraic sum of momenta it must be ensured that all of them are along a particular straight line.
Applications of conservation of momentum:-

Following few examples with illustrate the law of conservation of momentum.


(a) Recoil of gun:-

A gun and a bullet constitute one isolated system. On firing the gun, bullet moves out with a very high velocity
experiences a recoil. It moves in the opposite direction as shown in the below figure. Velocity
calculated by the application of law of conservation of momentum.

. The gun

of the recoil gun can be

Before Firing

After Firing

Momentum of bullet = 0

Momentum of bullet =

Momentum of gun = 0

Momentum of gun =

Total momentum of the system = 0

Total momentum of the system =

Here m and M are the masses of bullet and gun respectively. According to the law of conservation of momentum, momentum
before collision and after collision must be same.

or,
or,

Negative sign indicates that direction of motion of gun is in opposite direction.


(b) Rocket and jet plane:Fuel and oxygen is burnt in the ignition chamber. As hot gases escape from a rear opening, with some momentum, the rocket
moves in the forward direction with the same momentum.

(c)

Explosion of a bomb:-

Momentum of a bomb before explosion is zero. After explosion different fragments fly in various directions. It will be observed that
their momenta, when represented by the slide of a polygon, from a closed polygon, indicating that net momentum after explosion is
also zero. Thus, if the bomb exploded into two fragments, they must move in opposite directions.

(d) A man Jumping from a boat:When a man jumps from the boat to the shore, the boat is pushed backward. It can, exactly, be explained as in the case of recoil of
gun.
Some Conceptual Questions:-

Question 1:Figure below shows a popular carnival device, in which the contestant tries to see how high a weighted marker can be raised by
hitting a target with a sledge hammer. What physical quantity does the device measure? Is it the average force, the maximum force,
the work done, the impulse, the energy transferred, the momentum transferred, or something else? Discuss your answer.

Answer:The device will measure impulse. The impulse of the net force acting on a particle during a given time interval is equal to the change
in momentum of the particle during that interval. Since the contestant is hitting the target with a sledge hammer the change in
momentum is large and the time of collision is small, therefore it signifies that the average impulsive force will relatively large.
Suppose two persons bring the harmer from the same height, but they are hitting with different forces. The person who hits with
greater force for the short time interval the impulse will be more and this results the height of the mark will be more. Thus the device
will measure impulse.
_________________________________________________________________________________________________________
_______
Question 2:Can the impulse of a force be zero, even if the force is not zero? Explain why or why not?
Answer:Yes, the impulse of a force can be zero, even if the force is not zero.

Impulse of a force is defined as the change in momentum produced by the force and it is equal to the product of force and the time
for which it acts. The impulse of a force can be zero, if the net force acting on the particle during that time interval is constant. Since
the force is constant (both magnitude and direction), so change in momentum produced by the force will be zero. Therefore impulse
of the force will be zero.
From the above observation we conclude that, impulse of a force can be zero, even if the force is not zero.
_________________________________________________________________________________________________________
_________
Question 3:Explain how conservation of momentum applies to a handball bouncing off a wall.
Answer:Law of conservation of linear momentum states that, in an isolated system (no external force), the algebraic sum of momenta of
bodies, along any straight line, remains constant and is not changed due to their mutual action and reaction on each other.

The momentum of particle (p) is equal to the mass of particle (m) times the velocity of particle (v).
So,p = mv

(1)

Let us consider m is the mass of the ball and v is the velocity of the ball when the ball is collides with wall.
So using equation (1), the momentum of the ball before collision (p1) will be,
p1= mv

(2)

After collision, when the ball re bounces, the velocity of the ball will be, -v.
So again using equation (1), the momentum of the ball after collision (p2) will be,
p2= -mv

(3)

Conservation of linear momentum states that, the algebraic sum of momenta of bodies, along any straight line, remains constant
and is not changed due to their mutual action and reaction on each other.
p1 + p2 = 0
So, mv + (-mv) = 0

(4)

From equation (4) we observed that, linear momentum of the hand ball is conserved.
_________________________________________________________________________________________________________
Question 4:-

Give a plausible explanation for the breaking of wooden boards or bricks by a karate punch. (See Karate Strikes. by Jearl D.
Walker, American Journal of Physics, October 1975, p.845.)

Answer:In the process, breaking of wooden boards or bricks by a karate punch, the collision between the hand and brick is only for a few
milliseconds. Because the applied external force is large and the time of collision is small therefore the average impulsive force is
relatively large. Thus when you break a wooden board or bricks by a karate punch you have to apply large force for the minimum
time which is impulse. Therefore the impact force on the brick or wooden boards will be high.
Some Solved Problems:-

Problem 1:A 75.2-kg man is riding on a 38.6-kg cart travelling at a speed of 2.33 m/s. He jumps off in such a way as to land on the ground with
zero horizontal speed. Find the resulting change in the speed of the cart.
Concept:Momentum of the body p is equal to the mass of the body m times velocity of the body v.
So, p = mv
In accordance to the principle of conservation of energy, the final momentum of the system is equal to the initial momentum of the
system.
Consider the initial momentum of the man is pi,m, initial momentum of the cart is pi,c, final momentum of the man is pf,m and final
momentum of the cart is pf,c.
We define, pf,m = mmvf,m
pf,c = mcvf,c
pi,m = mmvi,m
pi,c = mcvi,c
Here, mass of the man is mm, mass of the cart is mc, initial velocity of the man is vi,m and cart is vi,c, and final velocity of the man
is vf,m and cart is vf,c.
Solution:So applying conservation of momentum to this system, the sum of the initial momentum of the man and cart will be equal to the sum
of the final momentum of the man and cart.
pf,m + pf,c = pi,m + pi,c
Substitute, mmvf,m for pf,m, mcvf,c for pf,c, mmvi,m for pi,m and mcvi,c for pi,c ijn the equation pf,m + pf,c = pi,m + pi,c,
pf,m + pf,c = pi,m + pi,c
mmvf,m + mcvf,c = mmvi,m + mcvi,c

vf,c- vi,c = (mmvi,m - mmvf,m)/ mc


vc = (mmvi,m - mmvf,m)/ mc
To obtain the resulting change in the speed of the cart vc, substitute 75.2 kg for mm, 2.33 m/s for vi,m and 0 m/s for vf,m in the
equation vc = (mmvi,m - mmvf,m)/ mc,
vc = (mmvi,m - mmvf,m)/ mc
= (75.2 kg) (2.33 m/s) (75.2 kg) (0 m/s)/(38.6 kg)
= 4.54 m/s
As the sign of the change in the speed of the cart vc is positive, this signifies that, the cart speed increases.
From the above observation we conclude that, the resulting change in the speed of the cart vc would be 4.54 m/s.
_________________________________________________________________________________________________________
__
Problem 2:A space vehicle is travelling at 3860 km/h with respect to the Earth when the exhausted rocket motor is disengaged and sent
backward with a speed of 125 km/h with respect to the command module. The mass of the motor is four times the mass of the
module. What is the speed of the command module after the separation?
Concept:Momentum of the body p is equal to the mass of the body m times velocity of the body v.
So, p = mv
In accordance to the principle of conservation of energy, the final momentum of the system is equal to the initial momentum of the
system.
Consider the initial momentum of the motor is pi,m, initial momentum of the command module is pi,c, final momentum of the motor
is pf,m and final momentum of the command module is pf,c.
Initial momentum of the motor is equal to the initial momentum of the command module.
So, initial momentum of the motor will be,
pi,m = mmvi,c
and final momentum of the motor pf,m will be,
pf,m = mm(vf,c vrel)
Here, speed of the motor relative to the command module is vrel.
Again, pf,c = mcvf,c
pi,c = mcvi,c
Here, mass of the motor is mm, mass of the command module is mc, initial velocity of the motor is vi,m and command module is vi,c,
and final velocity of the motor is vf,m and command module is vf,c.
Solution:So applying conservation of momentum to this system, the sum of the initial momentum of the motor and command module will be
equal to the sum of the final momentum of the motor and command module.
pf,m + pf,c = pi,m + pi,c
Substitute, mm(vf,c vrel) for pf,m, mcvf,c for pf,c, mmvi,c for pi,m and mcvi,c for pi,c in the equation pf,m + pf,c = pi,m+ pi,c,
pf,m + pf,c = pi,m + pi,c

mm(vf,c vrel)+ mcvf,c = mmvi,c + mcvi,c


(mm + mc)vf,c - mm vrel = (mm +mc) vi,c
(mm + mc)vf,c = mm vrel + (mm +mc) vi,c
So, vf,c = [mm vrel + (mm +mc) vi,c]/ (mm + mc)
As, mass of the motor is four times the mass of the module, thus,
mm = 4 mc
To obtain the speed of the command module vf,c, substitute 4 mc for mm, 125 km/h for vrel and 3860 km/h forvi,c in the equation vf,c =
[mm vrel + (mm +mc) vi,c]/ (mm + mc),
vf,c = [mm vrel + (mm +mc) vi,c]/ (mm + mc)
= [4 mc(125 km/h) + (4 mc+mc) (3860 km/h)]/ [4 mc+ mc]
= 3960 km/h
From the above observation we conclude that, the speed of the command module vf,c would be 3960 km/h.

Work, Energy and Power:-

Work, Energy and power are some of the more important topics of mechanics in classical physics.
A car moving, a rocket taking off, an aero plane flying, are a few of the events which take place regularly and can give us insight into
the concept of Work and Energy. These are some of the common sense notions of work and energy - which can however be
precisely defined and measured in physics.
These definitions and measurement can be used consistently to describe and predict the behavior of bodies.
Work is related to energy as they are inter-convertible and also the sums of them are conserved in Newtonian Physics. Power is a
step further which defines the rate of change of energy flow.The examples based on this are very easy and can be seen even in day
to day life.
Work:-

Work is said to be done if a force, acting on a body, displaces the body through a certain distance and the force has some
component along the displacements. Thus, work is done when the point of application of a force moves.It is defined as the
product of magnitude of displacement and the component of the force along the displacement.

Power:-

The rate at which work is done is called power.

Energy:-

Energy is the ability of the body to do some work. The unit of energy is same as that of work.
The chapter is important not only because it fetches 3-4 questions in most of the engineering examination but also because it is like
a linking pin to the previous and other forthcoming chapters of Mechanics.

Work:-

Work is said to be done if a force, acting on a body, displaces the body through a certain distance and the force has some
component along the displacements. Thus, work is done when the point of application of a force moves.
Work Done by a Constant Force:Consider a body experiencing a force F in a direction inclined at an angle with the positive direction of x-axis. Let the body be
displaced from A to B through a distance s.
First Definition:Work done is defined as the product of force and displacement.
Let

be a constant force acting on a body, and let the body undergo a displacement

work done by the force

is defined as,

, as shown in Figure given below, then the

W =

= (F cos)s = F(s cos) = Fs cos

Here is the angle between F and s.


Work done by the force is positive if the angle between force and displacement is acute (0<<90) as cos is positive. This
signifies, when the force and displacement are in same direction, work done is positive. This work is said to be done upon the body.
Second Definition:Work is defined as the product of magnitude of displacement and the component of the force along the displacement.
Third Definition:Work is also defined as the product of magnitude of force and the component of the displacement along the direction of
force.
Special cases of work done by a constant force:Case (a):When = 0, cos = 1
Force and displacement are in the same direction.
So, W = Fs. (See figure given below)
Case (b):When, = 180; cos = -1
Force and displacement are in opposite direction.
So, W = -Fs. (Figure given below)
Case (c)

[Zero Work]:-

When, = 90; cos = 0


Force is perpendicular to displacement
So, W = 0 (Figure given below)
Thus, when the force acts in a direction at right angle to the direction of displacement (cos90 = 0), no work is done (zero work).

Case (d) [Positive work]:When, 0 < < 90; cos is positive.


Force has a component in the direction of displacement.
So, W is positive.

Case (e) [Negative Work]:-

When, 90 < < 180, cos is negative


Force has a component in the direction opposite to displacement.
So, W is negative. (Figure given above)
Work done by the force is negative if the angle between force and displacement is obtuse (90<<180) as cos is negative. This
signifies, when the force and displacement are in opposite direction, work done is negative. This work is said to be done by the
body.
Work depends on the frame of reference:-

If a person is pushing a box inside a moving train, the work done in the frame of train will be
will be

, where

while that in the frame of earth

is the displacement of the train relative to the ground.

Work done by friction may be zero, positive or negative depending upon the situation. When force applied on a body is insufficient to
overcome the friction, work done by the friction force is zero. When this force is large enough to overcome the friction then, work
done by the friction force is negative. When force is applied on a body that is placed above another body, the work done by the
friction force on the lower body is positive.
Let us consider the situation in which a horizontal rough trolley, with a block and a man, is accelerating drawn from this.
(i)

In this case, work done by friction (between trolley and the block) is zero as observed by the man on trolley.

(ii)

Work done by friction (between trolley and the block) is positive as observed by an observer on the ground.

(iii)

Work done by friction is negative as observed by an observer who is moving along the direction of motion of trolley with higher

speed.
Conservative and Non-Conservative Forces:-

There are two types of forces:


(i) conservative forces, and
(ii) non-conservative forces.
If the work done by a force is independent of the path followed, it is called a conservative force, e.g. gravitational force, electrostatic
force,

etc.

If

the

work

done

by

force

depends

upon

the

path

followed,

it

is

called

non-conservative force, e.g. friction, viscous force, etc.


Alternatively, work done by a conservative force in a closed loop is zero & by non-conservative forces it is not zero for a round trip.
Work Done by a Variable Force:-

Consider a body being moved, along an arbitrary path, by a force which


continuously changes in magnitude as well as direction.
We first consider a force, acting on a body that varies in magnitude only. Assume that the magnitude of force is a function of position
x that is F(x), acting in the positive x direction. We should like to find out the work done, by F in moving a body from X1 to X2. Here W
is given by
W=
The physical significance of definite integral is area under the curve.
Hence, W is nothing but Area under the curve F(x) between the lines x=x1 and x=x2 as shown in figure given below.
Now, consider that the force F changes its magnitude as well as direction from point to point. The work done by the variable force F
on the body for two points on the curve a b, as the body moves from a to b will be,
Wab =
This integral is called a line integral. We can evaluate this integral and obtain the work done if we know how F and vary from point
to point.
Dimension formula for work:[M1 L2T-2]
Units of work:-

The unit of work done in S.I is joule (J) and in C.G.S system is erg.
1J = 1 N.m , 1 erg = 1 dyn.cm
joule:- Work done is said to be one joule if a force of 1 newton displaces a body through a distance of 1 m along the direction of
force.
1 J = (1 N) (1 m) = 1 kg m2s-2
erg:- Work done is said to be one erg if a force of 1 dyne displaces a body through a distance of 1 cm along the direction of force.
1 erg = (1 dyne) (1 cm) = 1 g cm2s-2
Relation between Joule and erg:- 1 J = 107 erg
Solved Problems:-

Problem 1:-

A 5.0-kg block moves in a straight line on a horizontal frictionless surface under the influence of a force that varies with position as
shown in below figure. How much work is done by the force as the block moves from the origin to x = 8.0 m?
Concept:The area under the force displacement curve is equal to the work is done by the force. So in the below figure, work is done by the
force as the block moves from the origin to x = 8.0 m will be equal to the sum of area under each curve and the line F= 0.
Solution:Area under the first curve (A1 = W1) which is a rectangle having length 10 N and width 2 S will be,
W1 = (10 N) (2 s) = 20 N. s
= (20 N. s) (1 J/1 N. s)
= 20 J

(1)

Area under the second curve (A2 = W2) which is a triangle having height 10 N and base 2 S will be,
W2 = (10 N) (2 s) = (20 N. s)
= (10 N. s) (1 J/1 N. s)
= 10 J

(2)

There no area (A3 = 0) under the third curve, therefore the work done for the third curve (A3 = W3) will be zero.
W3 = 0

(3)

Area under the fourth curve (A4 = W4) which is a triangle having height -5N and base 2 S will be,
W4 = (-5 N) (2 s) = (-10 N. s)
= (-5 N. s) (1 J/1 N. s)
= -5 J

(4)

Thus the total work is done by the force as the block moves from the origin to x = 8.0 m will be,
W = W1+ W2 + W3+ W4
= 20 J+10 J+0+ (-5 J)
= 25 J

(5)

From equation (5) we observed that, work is done by the force as the block moves from the origin to x = 8.0 m will be 25 J.

Problem 2:-

Figure below shows a spring with a pointer attached, hanging next to a scale graduated in millimeters. Three different weights are
hung from the spring, in turn, as shown. (a) If all weight is removed from the spring, which mark on the scale will the pointer
indicate? (b) Find the weight W.
Concept:The restoring force F acting on the spring is defined as,
F = -kx
Here k is the force constant and x is the elongation.
So, F = -kx
So the force constant k will be,
k = - F/x
and
the change in elongation x will be,
x = - F/k
Solution:(a) The force constant k will be,
k = - F/ x
= - [(-240 N) (-110 N)]/[60 mm-40 mm]
= 6500 N/m
Thus the change in elongation x with no force on the spring will be,
x = - F/k
= -(0 N) (-110 N)/( 6500 N/m)

= (-0.017 m) 103 mm/1 m)


= -17 mm
This is the amount less than the 40 mm mark, so the position of the spring with no force on it is 23 mm.
(b)
To obtain the weight W, first we have to find out the change in force F.
x = -10 mm compared to the 100 N picture.
To find out F, substitute -10 mm for x, 6500 N/m for k in the equation F = -kx,
F = -kx
= - (6500 N/m)(-10 mm10-3 mm/1 m)
= - (6500 N/m)(-0.010 m)
= 65 N
The weight W of the last object will be,
W = 110 N-65 N= 45 N
From the above observation we conclude that, the weight of the last object would be 45 N.
Problem 3:A 47.2-kg block of ice slides down an incline 1.62 m long and 0.902 m high. A worker pushes up on the ice parallel to the incline so
that its slides down at constant speed. The coefficient of kinetic friction between the ice and the incline is 0.110. Find (a) the force
exerted by the worker, (b) the work done by the worker on the block of ice, and (c) the work done by gravity on the ice.
Concept:Weight of the block W is defined as,
W= mg
Here m is the mass of the block and g is the free fall acceleration.
Work done W on a body is equal to the applied force P on the body times displacement x of the body.
W=Px
Solution:-

(a) The below figure shows a 47.2 kg block of ice slides down an incline 1.62 m long and 0.902 m high.
From the above figure,
sin = p/h
So the angle of incline will be,

= sin-1 (p/h)
= sin-1 (0.902 m/1.62 m)
= 33.8
The parallel components of the weight is,
WParallel = mgsin
And the perpendicular component is,
WPerpendicular = mgcos
The normal force on the ice is,
N=WPerpendicular
So the frictional force f will be,
f= k mgcos
Here k is the coefficient of kinetic friction, m is the mass and g is the free fall acceleration.
The push required to allow the ice to slide down at constant speed is then found by noting that P=WParallel -f.
Then the force exerted by the worker will be,
P=WParallel -f
= mgsin - k mgcos
= mg(sin - k cos)
To obtain the force exerted by the worker, substitute 47.2 kg for m, 9.81 m/s2 for g, 33.8 for , 0.110 for k in the
equation P= mg(sin - k cos),
P= mg(sin - k cos)
= (47.2 kg)( 9.81 m/s2)[ sin (33.8) - 0.110 cos(33.8)]
=(215 kg.m/s2) (1 N/1 kg.m/s2)
=215 N
From the above observation we conclude that, the force exerted by the worker will be 215 N.
(b) To obtain the worker done by the applied force W, substitute 215 N for P and -1.62 m for x in the equationW=Px,
W=Px
=(215 N)(-1.62 m)
= (-348 N.m) (1 J/1 N.m)
= -348 J
Thus the worker done by the applied force W would be -348 J.
(c) The work done W by the force gravity will be,
W=mgx cos(90-)
To obtain the work done W by the force gravity, substitute 47.2 kg for mass m, 9.81 m/s2 for g, 33.8 for and 1.62 m for x in the
equation W=mgx cos(90-),
W=mgx cos(90-)
=(47.2 kg)( 9.81 m/s2)(1.62 m) cos(90-33.8)
=(47.2 kg)( 9.81 m/s2)(1.62 m) cos(56.2)
=(417 kg.m2/s2) (1 J/1 kg.m2/s2)
=417 J

From the above observation we conclude that, the work done W by the force gravity will be 417 J.

Energy (E):Kinetic Energy:-

It is the ability of the body to do some work. The unit of energy is same as that of work.
Greater the amount of energy contained in the body, greater work it will able to do. In our daily life we see so many examples where
bodies are able to do work due to one reason or the other.
(a) A speedy bullet is able to penetrate a block of wood , thus, doing work against friction.
(b) A fast blowing wind is able to turn a wheel.
(c) A fan statrs rotating when electricity is fed to it.
All the agents which are able to do work are said to be energetic. we shall come across various types of energies during the study of
Physics, for examples-mechanicical energy, heat energy, light energy, electric energy etc. For the time being we shall concentrate
on the mechanical energy only.
Mechanical energy:The energy possessed by a body by virtue of its motion or by virtue of its position and/or configuration is called mechanical energy.
Mechanical energy is of two types:(a) Kinetic energy
(b) Potential energy

Definition of Kinetic Energy:-

It has been observed that a high speed bullet is able to pierce a block of wood, thus, doing work against friction.
Energy possessed by a body by virtue of its motion is called kinetic energy. Thus the kinetic energy is the form of energy contained
in an objects motion.
The side figure shows the change in kinetic energy of the ball at the different position of a hill. The energy just tranforms from one
part to the other part. So the total energy of the system remains conserved.
Mathematical Expression:Consider a body of mass m rolling over a rough horizontal surface. Its velocity while it crosses A is V and at B is zero as shown in
the below figure. In going from A to B, force of friction opposes the motion. The body has to do work against friction. It means that
work done by the body in going from A to B must be equal to the initial amount of energy (kinetic energy), contained in the body,
while at A.
If a is the acceleration of the body, then the force of friction F will be F = ma.
Let W be the work done by the body is going from A to B.

= FS cos 180
W = -FS (Since, cos 180 = -1)
So,

W = -mas
Initial velocity at A, u = V

Final velocity at B, v = 0

Acceleration = a

Distance travelled = S

Applying the relation,


v2 u2 = 2aS
Or, 0 V2 = 2aS
So, a = (V2/2S)
Substituting for a in equation (5),
W = -m (-V2/2S) S
Or, W = mV2
Since K.E. of the body is equal to the work done by the body, therefore,
K.E = mV2
Alternative Method:Let dW = work done by the body in moving through a distance dS

Integrating both sides, we can get total work done,

Since K.E. is equal to the work done,

Relation between kinetic energy (K) and momentum (p):-

Kinetic energy,
K = mv2
= (m2v2/m)
but, mv = p (momentum of body)
Or, K = p2/2m
(a) If p = constant,

K(1/m)

If two bodies of different masses have same momentum, body with a greater mass shall have lesser kinetic energy.
(b) If K = constant,

pm

If two bodies of different mass have same kinetic energy, body with a greater mass shall have greater momentum.
(c) If m = constant,

Kp2

For two bodies having same mass, the body having greater momentum shall have greater kinetic energy.
Work-Energy Theorem:The net work done (Wnet) by the forces acting on a body is equal to the change in the kinetic energy of the body.
So, Wnet = mvf2 mvi2
= Kf Ki
Conservation of Energy:-

Law of conservation of energy was first stated by Robert Mayor in 1842.


It states that, Energy can neither be created nor destroyed. It can be converted from one form to another. The sum of total
energy, in this universe, is always same.

The sum of the kinetic and potential energies of an object is called mechanical energy.
So, E = K+U
In accordance to law of conservation of energy, the total mechanical energy of the system always remains constant.
So, mgh + mv2 = constant
In the side figure, the kinetic and potentail energy of a simple pendulum is given. The K.E and P.E will be different at the different ,
different position of the bob. The loss of kinetic enegy will be eqaul to the gain in potential energy. So the energy just goes from one
form to other. This signifies, the total mecahnical energy (TME) of the system will be conserved.
Some Conceptual Question:Question 1:Does kinetic energy depend on the direction of the motion involved? Can it be negative? Does its value depend on the reference
frame of the observer?
Answer:No, kinetic energy does not depend on the direction of motion involved. Kinetic energy can never be negative. Yes, the value of
kinetic energy depends on the reference frame of the observer.
Kinetic energy (K) of a particle is defined as the,
K = mv2
Where m is the mass of the particle and v is the velocity of the particle.
Since kinetic energy is a scalar quantity, therefore there is no direction associated with kinetic energy and it has no components.
Thus kinetic energy does not depend on the direction of motion involved.
Since, mass of the body (m) is always a positive quantity and the term v2 in the kinetic energy is always positive, therefore, kinetic
energy of a body is always a positive quantity. Therefore kinetic energy can never be negative.
Newtons laws are valid only in inertial frames of reference. If we find Newtons second law to hold in one frame of reference, then it
holds in all inertial frames. If two observers in different inertial frames move at constant velocity v relative to one another and

observe the same experiment, they measure identical values for the forces, masses, and accelerations, and so they agree
completely in their analysis using Newtons second law.
In the Newtons second law, observers in different inertial frames will agree on the results of applying the work energy theorem.
However, unlike forces and accelerations, displacements and velocities measured by observers in different inertial frames will in
general be different, and so they will deduce different values for the work and kinetic energies in the experiment. Therefore the value
of kinetic energy depends on the reference frame of the observer.
Question 2:Does the work-energy theorem hold if friction acts on an object? Explain your answer.
Answer:No, the work-energy theorem does not hold if friction acts on an object.
Work-Energy theorem states that, the net work done by the forces acting on a body is equal to the change in the kinetic energy of
the body. In a frictional surface we do not count the part of work which will convert into the friction. The frictional force is in reality
quite complicated, involving the making and breaking of many microscopic welds, which deform the surfaces and result in changes
in internal energy of the surfaces (which may in part be revealed as an increase in the temperature of the surfaces).So the work
done by the force acting on the body is not totally converted into its kinetic energy and that contradicts the work energy theorem.
From the above observation we conclude that, the work-energy theorem does not hold if friction acts on an object.
Question 3:Why can a car so easily pass a loaded truck when going up hill? The truck is heavier, of course, but its engine is more powerful in
propertion (or is it)?. What considerations enter into choosing the design power of a truck engine and of a car engine?
Answer:In going up-hill by an object, energy is required. The amount of energy depends on the mass of the object. Thus, the object having
heavier mass required more energy to move uphill than the lighter one.
As the mass of the car is lesser than the mass of the loaded truck, lesser amount of energy is required by the car to move uphill. On
the other hand, large amount of energy is required by the loaded truck to move uphill. Therefore, a car can easily pass a loaded
truck when going uphill.
To design power of a truck engine and a car engine, the material should light to decrease the weight of the body. Therefore lesser
amount of energy will be required to move the body uphill.
Problem 1:A single force acts on a particle in rectilinear motion. A plot of velocity versus time for the particle is shown in below figure. Find the
sign (positive or negative) of the work done by the force on the particle in each of the intervals AB, BC, CD, and DE.

Concept:The net work done (Wnet) by the forces acting on a body is equal to the change in the kinetic energy of the body.

So, Wnet = mvf2 mvi2


Here m is the mass of the body, vi is the initial velocity of the body and vf is the final velocity of the body. If the kinetic energy of the
particle is decreasing in a certain region then the work done by the force on the particle will be negative and if the kinetic energy of
the particle is increasing in a certain region then the work done by the force on the particle will be positive.
Solution:A single force acts on a particle in rectilinear motion. A plot of velocity versus time for the particle is shown in the below figure.
In the interval AB, change in kinetic energy of the particle is positive. So kinetic energy of the particle is increasing, therefore the
sign of the work done will be positive.
In the interval BC, change in kinetic energy of the particle is zero since initial velocity (vi) of the particle is equal to the final velocity
(vf) of the particle in the interval BC. Therefore in this region work done will be zero.
In the interval CD, change in kinetic energy of the particle is negative. So kinetic energy of the particle is decreasing, therefore the
sign of the work done will be negative.
In the interval DE, change in kinetic energy of the particle is positive. So kinetic energy of the particle is increasing, therefore the
sign of the work done will be positive.
Problem 2:Calculate the kinetic energies of the following objects at the given speeds; (a) a 110-kg football linebacker running at 8.1 m/s; (b) a
4.2 bullet at 950 m/s; (c) the aircraft carrier (91,400 tons at 32.0 knots)
Concept:Kinetic energy K of a body having mass m moving with speed v is equal to the mv2.
So, K = mv2

(1)

Solution:(a) We have to find out the kinetic energy of the football having mass 110-kg and at a speed of 8.1 m/s.
To obtain Kinetic energy K of the football, substitute 110-kg for mass m and 8.1 m/s for speed the football v in the equation K =
mv2,
K = mv2
= (m) (v)2
= (110-kg) (8.1 m/s)2
= 3608.55 kg. m2/s2
= (3608.55 kg. m2/s2) (1 J/1 kg. m2/s2)
= 3608.55 J

(2)

Rounding off to two significant figures, the kinetic energy of the football will be 3600 J.
(b) We have to find out the kinetic energy of the bullet having mass 4.2-g and at a speed of 950 m/s.
To obtain Kinetic energy K of the bullet, substitute 4.2-g for mass m and 950 m/s for speed the bullet v in the equation K = mv2,
K = mv2
= (m) (v)2
= (4.2-g) (950 m/s)2
= ((4.2-g) (10-3 kg/1 g)) (950 m/s)2
= (4.210-3 kg) (950 m/s)2
=1895.25 kg. m2/s2

= (1895.25 kg. m2/s2) (1 J/1 kg. m2/s2)


(3)

= 1895.25 J

Rounding off to two significant figures, the kinetic energy of the bullet will be 1900 J.
(c) We have to find out the kinetic energy of the aircraft having mass 91,400 tons and at a speed of 32.0 knots.
To obtain Kinetic energy K of the aircraft, substitute 91,400 tons for mass m and 32.0 knots for speed the aircraft v in the
equation K = mv2,
K = mv2
= (m) (v)2
= (91,400 tons) (32.0 knots)2
= ((91,400 tons) (103 kg/1 ton)) ((32.0 knots)(0.5144 m/s / 1 knot))

= (914105 kg) (16.46 m/s)2


=1.241010 kg. m2/s2
= (1.241010 kg. m2/s2) (1 J/1 kg. m2/s2)
= 1.241010 J

(4)

From the equation (4) we observed that, the kinetic energy of the aircraft will be 1.241010 J.
Problem 3:A proton (nucleus of the hydrogen atom) is being accelerated in a linear accelerator. In each stage of such an accelerator the proton
is accelerated along a straight line at 3.601015 m/s2. If a proton enters such a stage moving initially with a speed
of 2.401015 m/s and the stage is 3.50 cm long, compute (a) its speed at the end of the stage and (b) the gain in kinetic energy
resulting from the acceleration. The mass of the proton is 1.6710-27 kg. Express the energy in electron volts.
Concept:Kinetic energy K of a body having mass m moving with speed v is equal to the mv2.
So, K = mv2

(1)

From equation (1), speed of the particle will be,


v = 2K/m

(2)

Work done (W) by a force F that moves a body through a displacement x will be,
(3)

W = Fx

Again in accordance to work energy theorem, the net work done by the forces acting on a body is equal to the change in the kinetic
energy K of the body.
So, K = Kf- Ki
=W

(4)

From equation (3) and (4) we get,


K = W
= Fx
= max

(Since, F = ma) (5)

Solution:(a) To obtain the final speed vf of the proton, first we have to find out the final kinetic Kf energy of the proton.
To obtain the change in the kinetic energy K, substitute 1.6710-27 kg for mass of the proton m, 3.601015m/s2 for a and 3.50 cm
for x in the equation K = max, we get,

K = max
= (1.6710-27 kg) (3.601015 m/s2)( 3.50 cm)
= (1.6710-27 kg) (3.601015 m/s2)( 3.50 cm10-2 m/1 cm)
=(2.1010-13 kg. m2/s2) (1 J/1 kg. m2/s2)
= (2.1010-13 J) (1 eV/1.610-19 J)
= 1.31106 eV
So, Kf- Ki = 2.1010-13 J (Since, K = Kf- Ki)
Or, Kf = (2.1010-13 J) + Ki
= (2.1010-13 J) + mvi2 (Since, Ki = mvi2 )
= (2.1010-13 J) + (1.6710-27 kg) (2.40107 m/s2)2
= [2.1010-13 J] +[(4.8110-13 kg. m2/s2) (1 J/1 kg. m2/s2)]
= [2.1010-13 J] +[4.8110-13 J]
=6.91 10-13 J
To obtain the final speed vf of the proton, substitute 6.91 10-13 J for Kf and 1.6710-27 kg for mass of the proton m in the
equation v = 2Kf/m, we get,
v = 2Kf/m
=2 (6.91 10-13 J) /(1.6710-27 kg) (1 kg. m2/s2/ 1 J)
=2.88107 m/s
From the above observation we conclude that, the final speed vf of the proton would be 2.88107 m/s.
(b) To find out the speed of proton at the end of the stage, we have to find out the change in
the kinetic energy K.
To obtain the change in the kinetic energy K, substitute 1.6710-27 kg for mass of the proton m, 3.601015m/s2 for a and 3.50 cm
for x in the equation K = max, we get,
K = max
= (1.6710-27 kg) (3.601015 m/s2)( 3.50 cm)
= (1.6710-27 kg) (3.601015 m/s2)( 3.50 cm10-2 m/1 cm)
=(2.1010-13 kg. m2/s2) (1 J/1 kg. m2/s2)
= (2.1010-13 J) (1 eV/1.610-19 J)
= 1.31106 eV
From the above observation we conclude that, the speed of proton at the end of the stage would be 1.31106 eV.

Potential Energy:-

Potential energy is the energy acquired by a body due to change in its position or its configuration. A force acting on a body
or a system can also change its potential energy.
Potential energy has its concept associated with conservative forces i.e., the forces for which the work done is independent of the
path followed. For example, gravitational force, electrostatic force etc.
Illustration:-

(a) Let a body raised upwards, against gravitational force, without acceleration. The force applied on the body moves it upwards
while gravitational force opposes it. In this case, there is no change in kinetic energy of the body while it acquires potential energy
due to a change in its position.
(b) Let a force be applied to a body which is used to (compress) a spring against a rigid support. Work in this case, is done against
elastic forces and is stored in the body in the form of potential energy.
All the forces, explained in this section, are conservative forces. On removal of these forces, the potential energy (stored) can be
released into another form of energy. Work done against non-conservative forces (friction, viscous drag etc.) cannot be released
back. That work is converted into heat and gets absorbed by surrounding systems.
The conservative force is given by the negative gradient of certain scalar quantity, called potential energy.
So, f = -dU/dx
Thus,

In the absence of an external force, the sum total of kinetic and potential energies always remains constant.
K + U = 0
Some other examples illustrating the concept of potential energy are given below:
(i) A wire supported by a rigid support and loaded at the other acquires potential energy.
(ii) A beam supported on two knife edges and loaded in the center acquires potential energy.
(iii) Water flowing in a river on a hill has potential energy stored in it. On falling down, potential energy is converted into electric
energy in hydraulic dams.

Every substance which is in state of possessing potential energy has a tendency to lose it. That is why a wound up spring tends to
open up. A stretched wire, a bent strip or a bent beam tends to acquire original configuration.
Thus, every system in equilibrium tends to exist in the state of minimum potential energy.
This basic fact has found a number of applications to explain the concept of a variety of phenomena in physics.
Here we shall discuss two cases:
(i) Potential energy of a body due to gravity, called gravitational potential energy.
(ii) Potential energy of a spring when it is elongated or compressed by an external force called elastic potential energy.
Gravitational Potential Energy:-

It is the energy acquired by a body by virtue of its position in the gravitational field.
A body having mass m always acted by a constant force F (= mg). It is directed towards the earth. Here g is acceleration due to
gravity.
Consider a body of mass m moving vertically downward from height y2 to y1, as shown in the Figure given below, the work done by
this constant force of gravity is given by

W = -mg(y2-y1) = - (mgy2 - mgy1)


Here W depends on the difference the height or position.
So, we can define gravitational Potential Energy associated with the body as
U = mgy
Hence W = -(U2 - U1) = -U
The negative sign implies that when gravity does positive work, the potential energy decreases. When gravity does negative work
(the body moves upward), the potential energy increases.
We can observe that when a body falls from a height, it accelerates and increases its speed and hence gains K.E. This is at the
expense of gravitational P.E. Hence we can relate potential energy and kinetic energy.
Relationship between Kinetic Energy and Potential Energy:-

We can relate P.E. and K.E. with the help of work energy theorem. Here the external force is gravitational force and the work done,
W, by this force is equal to change in K.E. of the body. According Work Energy Theorem this W is equal to change in P.E. i.e.
W = K = -U

=> K = + U = 0

Let the speed of body at height y1 and y2 be v1 and v2 respectively. (see Figure given below)
1/3mv22 - 1/2mv12 = -mg(y2 - y1)
1/2mv12 + mgy1 = 1/2mv22+ mgy1
Therefore the total energy, consists of K.E. and P.E., is conserved.
Now if the force is other than gravity and the work done by these forces is W', then according to Work-Energy theorem,
W = K + U
Elastic Potential Energy and Kinetic Energy:-

When a spring is elongated or compressed, work is done against the elastic restoring force of the spring (which we have discussed
in previous illustration, see Figure given below). The work done is stored in the spring as elastic potential energy. Consider a mass
'm' attached to a spring. An applied force P stretches the spring by pulling the mass to the right. As shown earlier for an applied
force P which is proportional to x, the work done is (1/2)kx2. This amount of work done is called the elastic potential energy of the
body.
The elastic restoring force acts in a direction opposite to the displacement. Hence, the worked one by the restoring force when the
body moves x1 to x2 is
Wel = -(1/2)kx22 -(-(1/2)kx12)
= -(U2 - U1) = -U
Let W' be the work done by the applied force P.
Total work done = change in kinetic energy.
W' + Wel = K,
W' = K + U,
W' = (1/2mv22 - 1/2mv12) + (1/2Kx22 - 1/2Kx12)
W' = (1/2mv22 + 1/2mv12) - (1/2mv22+ 1/2Kx12)
As defined earlier, the sum of the kinetic and potential energies of the body is the total mechanical energy. Thus the work of all
forces acting on the body with the exception of the elastic force equals the change in total mechanical energy of the body.
If there is no force other than the elastic restoring force acting on the body, then W' = 0, and the mechanical energy of the system is
conserved.
1/2mv12 + 1/2kx12 = 1/2mv22 + 1/2kx22
Conservation of Energy:Law of conservation of energy was first stated by Robert Mayor in 1842. It states that,

Energy can neither be created nor destroyed. It can be converted from one form to another. The sum total of energy, In
this universe, is always same.
Solved Problems:Problem 1:-

Two snow covered peaks are at elevations of 862 m and 741 m above the valley between them. A ski run extends from the top of
the higher peak to the top of the lower one as shown in below figure. (a) A skier starts from rest on the higher peak. At what speed
would he arrive at the lower peak if he coasts without using the poles? Assume icy conditions, so that there is no friction. (b) After
snow fall, a 54.4 kg skier making the same run also without using the poles only just makes it to the lower peak. By how much does
the internal energy of her skies and the snow over which she traveled increase?
Concept:Kinetic energy K of a body is defined as,
K = mv2
Here m is the mass of the body and v is the velocity of the body.
So, the initial kinetic energy Ki of the body having initial velocity vi will be,
Ki = mvi2
And the final kinetic energy Kf of the body having final velocity vf will be,
Kf = mvf2
Potential energy U is defined as,
U = mgh
Here m is the mass of the body, g is the free fall acceleration and h is the fall of height.
So, the initial potential energy Ui of the body having initial height yi will be,
Ui = mg yi
And the final potential energy Uf of the body having final height yf will be,
Uf = mg yf
Solution:(a) When the skier starts from rest on the higher peak, the initial velocity is zero results zero initial kinetic energy.
So, Ki = mvi2
= m(0)2
=0
In accordance to law of conservation of energy, the sum of initial potential and kinetic energy (Ui + Ki) of the skier at the top of the
peak is equal to the sum of final potential and kinetic energy (Uf + Kf) of the skier at the bottom.
So, Ui + Ki = Uf + Kf
As Ki = 0, the above equation Ui + Ki = Uf + Kf will be,

Ui + Ki = Uf + Kf
So, Kf = Ui - Uf
Substitute mvf2 for Kf, mg yi for Ui and mg yf for Uf in the equation Kf = Ui - Uf,
Kf = Ui - Uf
mvf2 = mg yi - mg yf
So, vf = 2 g (yi - yf)
To obtain the speed vf of the skier at the bottom, substitute 9.81 m/s2 for g, 862 m for yi and 741 m for yf in the equation vf = 2 g (yi yf),
vf = 2 g (yi - yf)
= 2 (9.81 m/s2) (862 m - 741 m)
= 48.7 m/s
= (48.7 m/s) (10-3 km/1 m)
= (48.7 m/s) (10-3 km/1 m) (3600 s/1 h)
= 175.320 km/h
Rounding off to three significant figures, the speed at which the skier arrives at the lower peak would be 175 km/h.
(b) The increase in internal energy E of her skis and the snow over which she travelled will be equal to difference of potential
energy at the bottom and top (Ui - Uf).
So, E = Ui - Uf
= mg yi - mg yf
= mg (yi yf)
To obtain the increase in internal energy E of her skis and the snow over which she travelled, substitute 54.4 kg for mass m, 9.81
m/s2 for g, 862 m for yi and 741 m for yf in the equation E = mg (yi yf),
E = mg (yi yf)
= (54.4 kg) (9.81 m/s2) (862 m - 741 m)
= - 6.46104 J
The negative sign signifies that, the internal energy of the snow and skis increased by 6.46104 J.
From the above observation we conclude that, the increase in internal energy E of her skis and the snow over which she travelled
would be 6.46104 J.
Problem 2:One end of a vertical spring is fastened to the ceiling. A weight is attached to the other end and slowly lowered to its equilibrium
position. Show that the loss of gravitational potential energy of the weight equals one-half the gain in spring potential energy.
Concept:The potential energy (Us) of a spring is defined as the,
Us = ky2

(1)

Where k is the spring constant and y is the extension of the spring from its original position.
Gravitational potential energy (Ug) of the weight (mg) would be,
Ug = -mgy

(2)

Negative sign indicates that force is acting against the gravity.

Solution:Force acting (F) on the spring due to an extension y will be,


(3)

F = ky

Where, k is the spring constant.


Weight is equal (mg) to the force due to extension of the spring (F).
So, F = mg

(4)

From equation (3) and (4), we get,


F = ky = mg

(5)

From equation (1), we get,


Us = ky2
= (ky) (y)

(6)

To obtain Us, substitute mg for ky in the equation, Us= (ky) (y),


Us= (ky) (y)
= mgy
So, 2 Us = mgy
= - (-mgy)
= - Ug

(Since, Ug = -mgy)

(7)

Negative sign indicates that force is acting against the gravity.


From equation (7) we observed that, the loss of gravitational potential energy of the weight equals one-half the gain in spring
potential energy.
Problem 3:A very small ice cube is released from the edge of a hemispherical frictionless bowl whose radius is 23.6 cm as shown in below
figure. How fast is the cube moving at the bottom of the bowl?

Concept:In an isolated system in which only conservative forces act, the total mechanical energy remains constant. That is, the initial value of
the total mechanical energy (Ki +Ui) is equal to the final value (Kf +Uf). Here Ki is the initial kinetic energy, Ui is the initial potential
energy, Kf is the final kinetic energy and Uf is the final potential energy.
So, Kf +Uf = Ki +Ui
mvf2 + mgyf = mvi2 + mgyi
Solution:Since the ice cube is released from the edge of a hemispherical frictionless bowl, so we have to substitute rfor yf (negative sign due
to down ward direction) , 0 for vi (at the point of release) and 0 for yi (at the point of release) in the equation mvf2 + mgyf =
mvi2 + mgyi.
mvf2 + mgyf = mvi2 + mgyi

vf2 + gyf = vi2 + gyi


vf2 + g(-r) = (0)2 + g(0)
vf2 = gr
So, the final velocity vf of the cube at the bottom of the bowl would be,
vf = 2gr
To obtain the final velocity vf, substitute 9.81 m/s2 and 23.6 cm for r in the equation vf = 2gr,
vf = 2gr
= 2(9.81 m/s2) (23.6 cm)
= 2(9.81 m/s2) (23.6 cm10-2 m/1 cm)
= 2.15 m/s
From the above observation we conclude that, the speed of the cube at the bottom of the bowl would be 2.15 m/s.
Work-Energy Theorem:It states that work done on/by a body gets converted into energy.
The type of energy depends upon the nature of force/forces acting on it.

(a) Body under the action of an unbalanced force.

Work-Energy principle for a body, under the action of an unbalanced force, can be stated as follows:
Constant force:It states that, The net work done (Wnet) by the forces acting on a body is equal to the change in the kinetic energy of the
body.
So, Wnet = mv2 mu2
= Kf Ki

If work is done on the body, its K.E increases and if work is done by the body, its K.E. decreases.

Consider a force F acting on a moving body. As a result of this, the body moves from A to B and gets its velocity increased from u to
v as shown in the below figure.
If dW is the work done in moving through a small distance ds,

So, dW = F ds cos 0
= F ds
= ma ds
Here a is the acceleration of the body.
Since, a = m(dv/dt)
So, dW = [m(dv/dt)].ds
= m dv (ds/dt)
dW = mv dv
Integrating both sides,

[W-0] = m[v2/2]
or, W = m[v2/2 u2/2]
or, W = mv2 mu2 = Final K.E Initial K.E
Thus, Work done = Change in kinetic energy
Therefore, the change in kinetic energy of a body equals the total work done by all the forces (conservative and non-conservative).
This is in accordance with the law of conservation of energy.
(b) Body under the action of a balanced force:-

Work done on/by a body shall result in a change of kinetic energy of the body if the force acting on it is unbalanced. The body may
move under the action of the balanced force.
In that case:
(i) there shall be no acceleration in the body
(ii) there shall be no change in kinetic energy
(iii) work done on the body by the applied force shall be converted into the potential energy which, in turn, shall result in change of
position or configuration of the body.
Conservation of Mechanical Energy:-

Mechanical energy of a system is the sum total of its kinetic and potential energy.
It states that, sum total of energy (of all kinds) of a system always remains constant while conservation of energy from one form to
another may take place continuously.
An interesting case of the work-energy theorem occurs when all the forces acting on a body are conservative. In this case, one can
define a potential energy for each of these forces:

where A and B are initial and final positions, respectively.


The work-energy theorem can now be re-written by using the relations,

This result states that the total mechanical energy,


Etotal = Ekinetic + Epotential = 1/2mv2 + U1 + U2 + ... is conserved.
Hence, Ekinetic + Epotential = constant.

Thus, if non-conservative forces are not present or if these forces are present but do not do any work, the mechanical energy of the
system remains constant.
On a roller coaster, energy is transformed from potential energy to kinetic energy and vice versa. Provided that external forces (such
as friction forces and applied forces) do not do work, the total amount of mechanical energy will be held constant.

The motion of the slider in the animation below is similar to the motion of a roller coaster car on a roller coaster track. The similarity
of the two motions is associated with the work-energy relationship. The energy bar charts accompanying the animation depict this
relationship. Energy bar charts are a conceptual tool amount the amount of each form of energy possessed by an object as it an
undergoes a particular motion. Observe from the animation that the total mechanical energy (TME) of the slider remains constant
throughout the motion. The total mechanical energy is the sum of the two forms of mechanical energy - kinetic energy (KE) and
potential energy (PE). While the individual amounts of kinetic and potential energy are undergoing change, their sum will always be
the same amount.

Conservative force (F):Conservative force is equal to the negative gradient of potential V of the field of that force. This force is also called central force.
So, F = - (dV/dr)
The line integral of a conservative force around a closed path is always zero.
So,

Non-conservative force:-

Force is non-conservative if the work done by that force on a particle that moves between two points depends on the path taken
between these points.
Refer this video for better understanding about conservation of energy:Solved Problems:Problem 1:A proton (nucleus of the hydrogen atom) is being accelerated in a linear accelerator. In each stage of such an accelerator the proton
is accelerated along a straight line 3.601015 m/s2. If a proton enters such a stage moving initially with a speed of 2.40107 m/s and
the stage is 3.50 cm long, compute (a) its speed at the end of the stage and (b) the gain in kinetic energy resulting from the
acceleration. The mass of the proton is 1.6710-27 kg. Express the energy in eV.
Concept:Kinetic energy K of a body having mass m moving with speed v is equal to the mv2.
So, K = mv2

(1)

From equation (1), speed of the particle will be,


v = 2K/m

(2)

Work done (W) by a force F that moves a body through a displacement x will be,
(3)

W = Fx

Again in accordance to work energy theorem, the net work done by the forces acting on a body is equal to the change in the kinetic
energy K of the body.
So, K = Kf- Ki
=W

(4)

From equation (3) and (4) we get,


K = W
= Fx
= max

(Since, F = ma) (5)

Solution:(a) To obtain the final speed vf of the proton, first we have to find out the final kinetic Kf energy of the proton.
To obtain the change in the kinetic energy K, substitute 1.6710-27 kg for mass of the proton m, 3.601015m/s2 for a and 3.50 cm
for x in the equation K = max, we get,
K = max
= (1.6710-27 kg) (3.601015 m/s2)( 3.50 cm) = (1.6710-27 kg) (3.601015 m/s2)( 3.50 cm10-2 m/1 cm)
=(2.1010-13 kg. m2/s2) (1 J/1 kg. m2/s2) = (2.1010-13 J) (1 eV/1.610-19 J) = 1.31106 eV
So, Kf- Ki = 2.1010-13 J (Since, K = Kf- Ki)
Or, Kf = (2.1010-13 J) + Ki
= (2.1010-13 J) + mvi2= (2.1010-13 J) + (1.6710-27 kg) (2.40107 m/s2)2
= [2.1010-13 J] +[(4.8110-13 kg. m2/s2) (1 J/1 kg. m2/s2)]= [2.1010-13 J] +[4.8110-13 J]
=6.91 10-13 J
To obtain the final speed vf of the proton, substitute 6.91 10-13 J for Kf and 1.6710-27 kg for mass of the proton m in the
equation v = 2Kf/m, we get,
v = 2Kf/m =2 (6.91 10-13 J) /(1.6710-27 kg) (1 kg. m2/s2/ 1 J)
=2.88107 m/s
From the above observation we conclude that, the final speed vf of the proton would be 2.88107 m/s.
(b) To find out the speed of proton at the end of the stage, we have to find out the change in the kinetic energy K.
To obtain the change in the kinetic energy K, substitute 1.6710-27 kg for mass of the proton m, 3.601015m/s2 for a and 3.50 cm
for x in the equation K = max, we get,
K = max
= (1.6710-27 kg) (3.601015 m/s2)( 3.50 cm) = (1.6710-27 kg) (3.601015 m/s2)( 3.50 cm10-2 m/1 cm)
=(2.1010-13 kg. m2/s2) (1 J/1 kg. m2/s2) = (2.1010-13 J) (1 eV/1.610-19 J)
= 1.31106 eV
From the above observation we conclude that, the speed of proton at the end of the stage would be 1.31106 eV.
Problem 2:A 3700-lb automobile (m = 1600 kg) starts from rest on a level road and gains a speed of 45 mi/h (= 72 km/h) in 33 s. (a) What is the
kinetic energy of the auto at the end of the 33s? (b) What is the average net power delivered to the car during the 33-s interval? (c)
What is the instantaneous power at the end of the 33-s interval assuming that the acceleration was constant?
Concept:Kinetic energy K of a body having mass m moving with speed v is equal to the mv2.
So, K = mv2

(1)

The power (P) due to the force acting on a body is equal to the work done on the body (W) divided by time (t).
Power delivered (P) to the body is equal to the force acting on the body (F) times speed of the body (v).
So, P = W/t
So, P = Fv

(2)
(3)

But force acting on the body (F) is equal to the mass of the body (m) times acceleration of the body (a).
F = ma

(4)

To obtain the power (P) in terms of mass (m) and acceleration (a), substitute ma for F in the equation P = Fv,

P = Fv
(5)

= mav

Acceleration (a) is the rate of change of velocity (v).


(6)

So, a = v/t

Substitute the value of a in the equation (5), the power (P) will be,
P = mav
= m (v/t) v = mv2/t

(7)

Solution:(a) We have to find out the kinetic energy of the auto having mass 1600 kg and at a speed of 72 km/h.
To obtain Kinetic energy K of the auto, substitute 1600 kg for mass m and 72 km/h for speed the football v in the equation K =
mv2,
K = mv2
= (1600 kg) (72 km/h)2
= (1600 kg) ((72 km/h)(103 m/1 km) (1 h/3600 s)) 2= (3.2105 kg. m2/s2) (1 J/1 kg. m2/s2)
= 3.2105 J

(8)

From equation (8) we observed that, the kinetic energy of the auto will be 3.2105 J.
(b) We have to find out the average net power delivered to the car during the 33 s interval.
To obtain the net power (P) of the auto, substitute 3.2105 J for kinetic energy K and 33 s for the time t in the equation P = W/t,
P = W/t
= (3.2105 J)/33 s = (9.7103 J/s) (1 W / 1 J/s)
= 9.7103 W

(9)

From equation (9) we observed that, the average net power delivered to the car during the 33 s interval will be 9.7103 W.
(c) We have to find out the instantaneous power at the end of the 33 s interval assuming that the acceleration was constant.
To obtain power (P), substitute 1600 kg for mass m, and 72 km/h for speed v and 33 s for time t in the equation P = mv2/t,
P = mv2/t
= (1600 kg) (72 km/h)2/33 s
= (1600 kg) ((72 km/h)(103 m/1 km) (1 h/3600 s)) 2 /33 s = (1.9104 kg. m/s3) (1 W/1 kg. m/s3)
= 1.9104 W

(10)

From equation (10) we observed that, the instantaneous power at the end of the 33 s interval will be 1.9104W.
Problem 3:The string in below figure has a length L = 120 cm, and the distance d to the fixed peg is 75.0 cm. When the ball is released from
rest in the position shown, it will follow the arc shown in the figure. How fast will it be going (a) when it reaches the lowest point in its
swing and (b) when it reaches its highest point, after the string catches on the peg?

Concept:Kinetic energy K of an object is defined as,


K = mv2
Here m is the mass of the object and v is the speed of the object.
Potential energy U is defined as,
U = mgy
Here, m is the mass of the object, g is the free fall acceleration and y is the height.
In accordance to conservation of energy, the total change in kinetic energy Ktoatal of all the particles that make up the system is
equal in magnitude, but opposite in sign, to the total change in the potential energy -Utotal of the system.
Ktoatal = -Utotal
So,
K-K0= -(U-U0)
Solution:Assume that, U0= K0 = 0
Substitute 0 for U0 and 0 for K0 in the equation K-K0= -(U-U0), we get,
K-0 = -(U-0)
So, K= -U
Substitute mv2 for kinetic energy K and mgy for potential energy U in the equation
K= -U, we get,
K= -U
mv2= -mgy
v2/2 = -gy
Or, v = 2g(-y)
(a) To obtain the speed v of the ball, when it reaches the lowest point in its swing, substitute 9.81 m/s2 for free fall
acceleration g and -120 cm for y (negative sign for negative axis) in the equation v = 2g(-y), we get,
v = 2g(-y)
=2(9.81 m/s2)(-(-120 cm))
=2(9.81 m/s2)(120 cm)(10-2 m/1cm)
=4.85 m/s
From the above observation we conclude that, the speed v of the ball, when it reaches the lowest point in its swing would be 4.85
m/s.
(b) The value of y, when the ball reaches its highest point would be,
y = -[120 cm-(120 cm-75.0 cm)-(120 cm-75.0 cm)]
= (- 30 cm) (10-2 m/1 cm)

= -0.30 m
To obtain the speed v of the ball, when the ball reaches its highest point, substitute 9.81 m/s2 for free fall acceleration g and -0.30 m
for y (negative sign for negative axis) in the equation v = 2g(-y), we get,
v = 2g(-y)
=2(9.81 m/s2)(-(-0.30 m))
=2.43 m/s
From the above observation we conclude that, the speed v of the ball, when the ball reaches its highest point, after the string
catches on the peg would be 2.43 m/s.
Motion in a Vertical Circle
Whenever a body is released from height, it travels vertically
downward towards the surface of earth. This is due to the force
of gravitational attraction exterted on body by the earth. The
acceleration produced by this force is called acceleration due
to gravity and is denoted by g. Value of g on the surface
of earth is taken to the 9.8 m/s2 and it is same for all the
bodies. It means all bodies (whether an iron ball or a piece of
paper), when dropped (u=0) from same height should fall with
same rapidity and should take same time to reach the earth. Our
daily observation is contrary to this concept. We find that
iron ball falls more rapidlly than piece of paper. This is due
to the presence of air which offers different resistance to them. In the absence of air both
would have taken same time to reach the surface of earth.
A particle of mass m is attached to a light and inextensible string. The other end of the string
is fixed at O and the particle moves in a vertical circle of radius r equal to the length of the
string as shown in the figure.

Consider the particle when it is at the point P and the string makes an angle with vertical.
Forces acting on the particle are:
T = tension in the string along its length, and
mg = weight of the particle vertically downward.
Hence, net radial force on the particle is FR = T - mg cos
=> T - mg cos = mv2/R

=> T = mv2/R + mg cos


Since speed of the particle decreases with height, hence tension is maximum at the bottom, where
cos = 1 (as = 0).
=> Tmax = mv2/R + mg; Tmin = mv'2/R - mg (at the top)
Here,

v' = speed of the particle at the top.

Critical Velocity:-

It is the minimum velocity given to the particle at the lowest point


to complete the circle.
The tendency of the string to become slack is maximum when the particle is at the topmost point
of the circle.
At the top, tension is given by T = mvT2/R - mg, where vT = speed of the particle at the top.
=>

mvT2/R

= T + mg

For vT to be minimum, T 0 => vT = gR


If vB be the critical velocity of the particle at the bottom, then from conservation of energy
Mg(2R) +

1/2 mvT2

= 0 + 1/2 mvB2

As vT = gR => 2mgR + 1/2 mgR = 1/2 mvB2


Thus,

Note: In case the particle is attached with a light rod of length l, at the highest point its
minimum velocity may be zero. Then, the critical velocity is 2gl.
Problem 1:-

A heavy particle hanging from a fixed point by a light inextensible string of length l is
projected horizontally with speed (gl). Find the speed of the particle and the inclination of
the string to the vertical at the instant of the motion when the tension in the string is equal
to the weight of the particle.
Solution :Let tension in the string becomes equal to the weight of the particle when particle reaches the
point B and deflection of the string from vertical is . Resolving mg along the string and
perpendicular to the string, we get net radial force on the particle at B, i.e.
FR = T - mg cos

... (i)

If vB be the speed of the particle at B, then


FR = mvB2/l

... (ii)

From (i) and (ii), we get,

T - mg cos = mvB2/l
Since at B, T = mg => mg(1 - cos ) =

... (iii)
mvB2/l

=> vB2 = gl (1 - cos )

... (iv)

Conserving the energy of the particle at points A and B, we have


1/2mvA2 = mgl (1 - cos ) + 1/2mvB2
where vA=gl and vB=gl(1-cos ) => gl =2gl(1-cos ) + gl(1-cos )
=> cos = 2/3

... (v)

Putting the value of cos q in equation (iv), we get v = gl/3


Problem 2 (JEE Advanced):A particle move along a circle of radius (20/) m with constant tangential acceleration. If the
speed of the particle is 80 m/s at the end of the second revolution after motion has begun, then
find out the tangential acceleration.
Solution :Let tangential acceleration, at = a
Then,
v =

2ad

(d = distance travelled)

So, a = v2/ad = v2/2(2R) = v2/(4R)


Thus, a = (80)2/[(4) (20/)] = 80 m/s2
From the above observation we conclude that, the tangential acceleration would be 80 m/s2.

As the object goes round the circle the tension in the string varies being greatest at the bottom of the circle and least at the top.

As the object goes round the circle the tension in the string varies being least at the top of the circle.

If the string is to break it will be at the bottom of the path where it has to not only support the object but also pull it up out of it straight-line path.

The condition for the object to execute a complete vertical circle without the string is independent of the mass of the object.

It is slightly easier to cause an object attached to the end of a rigid rod to execute a vertical circle than it is to cause an object attached to the end of a string to
execute the same circle.

Power:-

While defining work, time element is not involved in it. Sometimes it is necessary to consider the rate at which work is done as well
as the total work done.
The rate at which work is done is called power.
If work is done at a uniform rate, average power P is defined as,
P = W/t
If the work is done at a variable rate, instantaneous power P is defined as,
P = dW/dt
(a) Instantaneous power in terms of velocity:-

Let a force

, acting on a body, displace it through a displacement

Thus, Instantaneous power,

For a constant force,

. Work done, W, is given by:

Here, v is the velocity of the body.


Therefore, Instantaneous power is defined as the product of force and displacement.
(b) Instantaneous power in terms of mechanical energy:Mechanical energy E of body is the sum total of its kinetic energy (K) and potential energy (U).
Or,

Therefore, instantaneous power, P = dE/dt


Thus, instantaneous power is defined as the rate at which mechanical energy is transformed.
Dimensions of Power:-

Power, P = Work / Time


= [M1L2T-2] / [T1]
= [M1L2T-3]
So, Dimensions of power are 1,2,-3 in mass length and time, respectively.
Units of Power:(a) Absolute Units:(i) C.G.S. System:Absolute units of power, in C.G.S. system, is erg s-1.
Erg/second:- Power of an agent is said to be 1 erg s-1 if it does one erg of work in one second.
(ii) S.I:Absolute unit of power, in S.I., is joule per second or watt (W).
Watt (Js-1):- Power of an agent is said to be one watt if it does 1 joule of work in one second.
1 W = 1 Js-1.

A bigger unit is also, often used.


1 kilowatt (kW) =1000 W
= 1000 Js-1
1 Megawatt = 106 Watt
Another unit of power, horse power (HP), is often used in our daily life.
1 HP = 746 W
1 kW = 1.34 HP
1 MW = 1340 HP
(b) Gravitational Units:(i) C.G.S. System:Gravitational units of power, in C.G.S. system, is gram centimeter per second (g cm s-1).
Gram centimeter/second (g cm s-1):- Power of an agent is said to be one garm centimeter per second if it does work at the rate of
980 erg per second.
(ii) S.I.:Gravitational unit of power, in S.I. is kilogram meter per second.
Kilogram meter per second (kg m s-1):-Power of an agent is said to be one kilogram meter per second if it does work at the rate of
9.8 joule per second.
Solved Problems:Problem 1:A 57-kg woman runs up a flight of stairs having a rise of 4.5 m in 3.5 s. What average power must she supply?
Concept:Work done W is defined as,
W = Fs
Here F is the force and s is the displacement.
Power P is defined as,
P = W/t
Here W is the work done and t is the time.
Solution:As the woman runs up a flight of stairs, there are two forces will act on the woman. One is the gravitational force which is in the
downward direction and other one is the normal force N of the floor which balances the weight of the body (gravitational force) and
the direction is just opposite to the direction of gravitational force.
So, F = N
= weight of the body
= mg

(Since, weight of the body = mg)

Here m is the mass of the body and g is the acceleration due to gravity.
To obtain the force F exerted by the woman to propel herself up the stairs, substitute 57 kg for m and 9.8 m/s2 for g in the
equation F = mg,
F = mg
= (57 kg) (9.8 m/s2)

= (560 kg.m/s2) (1 N/1 kg.m/s2)


= 560 N
In accordance to the reference of woman the stairs are moving down. Thus the woman will exert a force in the down ward direction.
To obtain the work done W by the woman, substitute 560 N for F and 4.5 m for s in the equation W = Fs,
W = Fs
= (560 N) (4.5 m)
= (2500 N.m) (1 J/1 N.m)
= 2500 J
As the direction of force is in the direction of displacement, thus the work done is positive here.
To obtain the average power P supplied by the woman, substitute 2500 J for W and 3.5 s for t in the equationP = W/t,
P = W/t
= 2500 J/3.5 s
= (710 J/s) (1 W/ 1 J/s)
= 710 W
From the above observation we conclude that, the average power P supplied by the woman would be 710 W.
Problem 2:The motor on a water pump is rated at 6.6 hp. From how far down a well can water be pumped up at the rate of 220 gal/min?
Concept:Mass flow rate (R) of water is equal to the volume flow rate (Q) times density of the water (d).
So, R = (Q) (d)
The power (P) of the motor on the water pump is defined as,
P = (h) (g) (R)
Where h is the height, g is the acceleration due to gravity and R is the mass flow rate.
Solution:First we convert the units from non- SI system to SI system.
Power of motor, P = 6.6 hp
= (6.6 hp) (745.6 W/1 hp)
= 4920.96 W
Rounding off to three significant figure, the power will be 4920 W.
Volume Flow rate, Q = 220 gal/min
= (220 gal/min) (3.785 L/1 gal) (1 min/60 s)
= 13.9 L/s
Mass flow rate (R) of water is equal to the volume flow rate (Q) times density of the water (d).
So, R = (Q) (d)

(1)

To find mass flow rate (R), substitute 13.9 L/s for the volume flow rate Q and 1.00 kg/L for the density of the water (d) in the
equation R = (Q) (d),
R = (Q) (d)
= (13.9 L/s) (1.00 kg/L)
= 13.9 kg/s

(2)

The power (P) of the motor on the water pump is defined as,
(3)

P = (h) (g) (R)

Where h is the height, g is the acceleration due to gravity and R is the mass flow rate.
From equation (3), the height would be,
h = P/gR
To find the height (h), substitute 4920 W for power (P), 9.81 m/s2 for g and 13.9 kg/s for R in the equation h =P/gR,
h = P/gR
= (4920 W)/( 9.81 m/s2) (13.9 kg/s)
= 36.1 W/ (m/s2. kg/s)
= (36.1 W/ (m/s2. kg/s)) ((kg.m2/s3) / 1W)
= (36.1 m) (1 ft/0.3048 m)
(4)

= 118.11 ft

Rounding off to two significant figures, the height would be 120 ft.
From the above observation we conclude that, the water can be pumped from a well at a height 118.11 ft.
Problem 3:What power is developed by a grinding machine whose wheel has a radius of 20.7 cm and runs at 2.53 rev/s when the tool to be
sharpened is held against the wheel with a force of 180 N? The coefficient of friction between the tool and the wheel is 0.32.
Concept:Power delivered (P) to the body is equal to the force acting on the body (F) times speed of the body (v).
(1)

So, P = Fv

Frictional force (F) acting on the body is equal to the coefficient of friction (k) times the normal force (N).
(2)

F = kN

Solution:A wheel has a radius of 20.7 cm and run at 2.53 rev/s when the tool to be sharpened is held against the wheel with a force of 180 N.
The linear velocity (v) of the wheel is equal to the angular velocity of the wheel (w) times radius of the wheel (r).
v= wr

(3)

But angular velocity of the wheel is equal to the 2 times frequency (f) of the wheel.
w = 2f

(4)

To obtain linear velocity (v) in terms of frequency f, substitute 2f for w in the equation v= wr,
v= wr
=2fr (5)
To obtain linear velocity (v) of wheel, put 20.7 cm for radius of the wheel r, 2.53 rev/s for w, and 3.14 for in the equation v =2fr,
v =2fr
=2(3.14)( 2.53 rev/s)(20.7 cm)
=2(3.14)( 2.53 rev/s)(20.7 cm) (10-2 m/1 cm)
=3.29 m/s

(6)

To obtain the frictional force (F), substitute 0.32 for the coefficient of friction k and 180 N for normal force (N) in the
equation F = kN,
F = kN

= 0.32 (180 N)
= 57.6 N

(7)

To obtain the power (P) developed by the grinding machine, substitute 57.6 N for F and 3.29 m/s for linear velocity v in the
equation P = Fv,
P = Fv
= (57.6 N) (3.29 m/s)
= 189.504 N. m/s
= (189.504 N. m/s) (1 W/ N. m/s)
= 189.504 W

(8)

Rounding off to two significant figure, power (P) developed by the grinding machine would be 190 W.
Problem 4:A jet airplane is traveling 184 m/s. In each second the engine takes in 68.2 m3 of air having a mass of 70.2 kg. The air is used to
burn 2.92 kg of fuel each second. The energy is used to compress the products of combustion and to eject them at the rear of the
engine at 497 m/s relative to the plane. Find (a) the thrust of the jet engine and (b) the delivered power (horse power).
Concept:Force or thrust F is equal to the rate of change of momentum (p/ t).
F = p/ t
Power P is defined as,
P = Fv
Here F is the force and v is the velocity.
Solution:(a)
First we have to find out the change in momentum p of the jet engine.
The change in momentum p will be,
p = (70.2 kg) (497 m/s-184 m/s) +(2.92 kg) (497 m/s
= 2.34104 kg.m/s
To obtain the thrust F of the jet engine, substitute 2.34104 kg.m/s for p and 1 s for t in the equation F = p/ t,
F = p/ t
= (2.34104 kg.m/s)/(1 s)
= (2.34104 kg.m/s2) (1 N/1 kg.m/s2)
=2.34104 N
From the above observation we conclude that, the thrust F of the jet engine would be 2.34104 N.
(b)
To obtain the delivered power P of the jet engine, substitute 2.34104 N for thrust F and 184 m/s for the velocity v of the jet airplane
in the equation P = Fv, we get,
P = Fv
= (2.34104 N) (184 m/s)
= (4.31106 N.m/s) (1 W/1 N.m/s)
= 4.31106 W

= (4.31106 W) (0.001341 hp/1 W)


= 5780 hp
From the above observation we conclude that, the delivered power P of the jet engine would be 5780 hp.
Solved Examples on Work, Power and Energy:Problem 1:-

A small object of mass m = 234 g slides along a track with elevated ends and a central flat part, as shown in below figure. The flat
part has a length L = 2.16 m. The curved portions the tracks are frictionless; but in traversing the flat part, the object loses 688 mJ of
mechanical energy, due to friction. The object is released at point A, which is a height h = 1.05 m above the flat part of the track.
Where does the object finally come to rest?

Concept:-

The potential energy U is defined as,


U = mgh
Here, m is the mass of the body, g is the free fall acceleration and his the fall of height.
Solution:To find the potential energy of the object at the point A, substitute 234 g for mass of the object m, 9.81 m/s2for free fall
acceleration g and 1.05 m for height h in the equation U = mgh,
U = mgh
= (234 g) (9.81 m/s2) (1.05 m)
= (234 g10-3 kg/1 g) (9.81 m/s2) (1.05 m)
= 2.41 kg. m2/s2
= (2.41 kg. m2/s2) (1 J/1 kg. m2/s2)
= 2.41 J
The curved portions of the track are frictionless; but in traversing the flat part, the object losses 688 mJ of mechanical energy, due to
friction.
The number of times (n) that the particle will move back and forth across the flat portion is the ratio of the potential energy (2.41 J)
of the object at the point A to the mechanical energy (688 mJ)that the object losses due to friction.
So, n = 2.41 J/688 mJ
= (2.41 J/688 mJ) (1 mJ/10-3 J)
= 3.50

The number of times n signifies that, the particle will come to a rest at the center of the flat part while attempting one last right to left
journey.
Problem 2:-

The magnitude of the force of attraction between the positively charged proton and the negatively charged electron in the hydrogen
atom is given by,
F = K e2/r2,
where e is the electric charge of the electron, k is a constant, and r is the separation between electron and proton. Assume that the
proton is fixed. Imagine that the electron is initially moving in a circle of radius r1 about the proton and jumps suddenly into a circular
orbit of smaller radius r2 as shown in the below figure. (a) Calculate the change in kinetic energy of the electron, using Newtons
second law. (b) Using the relation between force and potential energy, calculate the change in potential energy of the atom. (c) By
how much has the total energy of the atom changed in this process? (This energy is often given off in the form of radiation.)

Concept:-

Kinetic energy K of a body is defined as,


K = mv2
Here m is the mass of the body and v is the velocity of the body.
In a circular motion, the centripetal force Fc acting on the particle is defined as,
Fc = mv2/r
Here m is the mass particle, v is the velocity of the particle and r is the radius of the circle.
Apply Newtons second law to the motion of electron in a circular orbit of the hydrogen atom where the proton is at the center of
circle.
So the centripetal force (Fc = mv2/r) must be equal to the force of attraction (F= ke2/r2) between the positively charged proton and the
negatively charged electron in the hydrogen atom.
So, Fc = F
mv2/r = ke2/r2
Or, mv2 = ke2/r
The potential energy difference U is defined as,

U = - Fdr
Here F is the force.
The total energy E will be,
E = K + U
Solution:-

(a) The kinetic energy K of the electron will be,


K = mv2
= (ke2/r)

(Since mv2 = ke2/r)

Since the electron is initially moving in a circle of radius r1 about the proton and jumps suddenly into a circular orbit of smaller
radius r2, therefore the change in kinetic energy K will be,
K = ke2 (1/r2 1/r1)
From the above observation we conclude that, the change in kinetic energy of the electron K will be ke2(1/r2 1/r1).
(b) To obtain the change in potential energy U, substitute ke2/r2 for F in the equation U = -

Fdr and integrating from the limit r1 to r2,

Thus the change in potential energy U of the atom will be -ke2 (1/r2 1/r1).
c) To obtain the total energy E of the atom changed in this process, substitute ke2 (1/r2 1/r1) for K and -ke2 (1/r2 1/r1) for
U in the equation E = K + U,
E = K + U
= ( ke2 (1/r2 1/r1)) + (-ke2 (1/r2 1/r1))
= -( ke2 (1/r2 1/r1))
From the above observation we conclude that, the total energy E of the atom changed in this process would be -( ke2 (1/r2
1/r1)).
Problem 3:-

A particle of mass 2.0 kg moves along the x axis through a region in which its potential energy U(x) varies as shown in below figure.
When the particle is at x = 2.0 m, its velocity is -2.0 m/s. (a) Calculate the force acting on the particle at this position. (b) Between
what limits does the motion takes place? (c) How fast is it moving when it is at x = 7.0 m?

Concept:-

Force acting on the particle F having potential energy U(x) will be,
F = - dU(x)/dx
Kinetic energy K of a particle is defined as,
K = mv2
Here, mass of the particle is m and speed of the particle is v.
So the velocity of the particle will be,
v = 2K/m
Total energy E is equal to the sum of kinetic energy K and potential energy U.
E = K+U
So the kinetic energy K will be,
K = E-U
Solution:-

(a) The above figure shows a particle of mass 2.0 kg moves along the x axis through a region in which its potential energy U(x)
varies.
Using figure,
U = -17 J (-3 J)
= -14 J
x = 4 m-1 m
= 3 m
To obtain the force acting on the particle F at x = 2.0 m, substitute -14 J for U and 3 m for x in the equationF = - dU(x)/dx,
F = - dU(x)/dx
= - (-14 J/3 m)
= (4.7 J/m) (1 N/(1 J/m))
= 4.7 N

From the above observation we conclude that, force acting on the particle F at x = 2.0 m would be 4.7 N.
(b) To find the kinetic energy of the particle at at x = 2.0 m, substitute 2.0 kg for mass m and
-2.0 m/s for v in the equation K = mv2,
K = mv2
=1/2 (2.0 kg) (-2.0 m/s)2
= 4 kg.m2/s2
= (4 kg.m2/s2) (1 J/1 kg.m2/s2)
=4J
From the figure, the potential energy U is,
U = -7 J
So the total energy E is equal to the sum of kinetic energy K and potential energy U.
E = K+U
= 4 J+ (-7 J)
= -3 J
This signifies that, the particle is constrained to move between x = 1 m and x = 14 m.
(c) For x= 7 m, the total energy is E = -3 J and potential energy U = -17 J.
To obtain the kinetic energy K, substitute -3 J for E and -17 J for U in the equation K = E-U, we get,
K = E-U
= (-3 J) (-17 J)
= 14 J
To obtain the speed of the particle when it is at x = 7.0 m, substitute 14 J for K and 2.0 kg for m in the equation v = 2K/m,
v = 2K/m
= 2(14 J) /(2.0 kg) ((1 kg.m2/s2)/1 J)
= 3.7 m/s
From the above observation we conclude that, the speed of the particle when it is at x = 7.0 m would be 3.7 m/s.
Problem 4:-

Delivery trucks that operate by making use of energy stored in a rotating flywheel have been used in Europe. The trucks
are changed by using an electric motor to get the flywheel up to its top speed of 624 rad/s. One such fly wheel is a solid,
homogeneous cylinder with a mass of 512 kg and a radius of 97.6 cm. (a) What is the kinetic energy of the flywheel after charging?
(b) If the truck operates with an average power requirement of 8.13 kW, for how many minutes can it operate between chargings?
Concept:-

Rotational kinetic energy (Kr) of a flywheel is defined as,

Kr = Iw2

(1)

Here I is the rotational inertia of the flywheel and w is the angular velocity of the flywheel.
Momentum of inertia (I) of the flywheel is defined as,
I = mr2

(2)

To obtain the rotational kinetic energy (Kr) in terms of mass (m) and radius (r), substitute mr2 for I in the equation Kr = Iw2,
Kr = Iw2
= (1/2 mr2)(w2)
= (mr2)(w2) (3)
The power (P) due to the force acting on a body is equal to the work done on the body (W) divided by time (t).
(4)

P = W/t

From equation (4), the time (t) will be,


t = W/P

(5)

Solution:-

(a) To obtain the rotational kinetic energy (Kr) of the flywheel after charging, substitute 512 kg for mass m, 97.6 cm for radius r and
624 rad/s for angular velocity w in the equation Kr = (mr2)(w2),
Kr = (mr2)(w2)
= (512 kg) ((97.6 cm)(10-2 m/1 cm)) 2(624 rad/s) 2
= 4.75107 kg. m2/s2
= (4.75107 kg. m2/s2) (1 J/ kg. m2/s2)
= 4.75107 J

(6)

From equation (6) we observed that, the rotational kinetic energy (Kr) of the flywheel after charging will be 4.75107 J.
(b) We have to find out the time in minutes which can operate between chargings, when the truck operates with an average power
requirement of 8.13 kW.
To obtain time (t), substitute 4.75107 J for work W and 8.13 kW for power P in the equation t = W/P,
t = W/P
= 4.75107 J / 8.13 kW
= (4.75107 J / 8.13 kW) (1 kW/ 103 W)
= 5842.6 J/W
= (5842.6 J/W) (1 s/(1 J/W))
= 5842.6 s
= (5842.6 s) ((1/60 minutes)/ 1 s)

= 97.4 minutes

(7)

From equation (7) we observed that, the truck can operate 97.4 minutes between chargings.
Problem 5:-

A 263-g block is dropped onto a vertical spring with force constant k = 2.52N/cm as shown in the below figure. The block sticks to
the spring, and the spring compress 11.8 cm before coming momentarily to rest. while the spring is being compressed, how much
work is done (a) by the force of gravity and (b) by the spring? (c) What was the speed of the block just before it hit the spring? (d) If
this initial speed of the block is doubled, what is the maximum compression of the spring? Ignore friction.

Concept:

Work done by the force of gravity Wg is defined as,


Wg = mgh
Here m is the mass, g is the free fall acceleration and h is the height.
Work done by the force of spring Ws is defined as,
Ws = kx2
Here k is the force constant and x is the extension.
Kinetic energy K of a body is defined as,
K = mv2
Here m is the mass and v is the speed of the object.
So, v=2K/m
Solution:-

(a) Here the work done by the force of gravity Wg will be,
Wg = m(-g)h
= -mgh
To obtain the work done by the force of gravity Wg, substitute 263 g for m, 9.81 m/s2 for g and -11.8 cm for hin the equation Wg= mgh,

Wg= -mgh
= -(263 g)(9.81 m/s2)(-11.8 cm)
= -(263 g(10-3 kg/1 g))(9.81 m/s2)(-11.8 cm(10-2 m/1 cm))
= (0.304 kg.m2/s2) (1 J/1 kg.m2/s2)
= 0.304 J
Thus the work done by the force of gravity Wg would be 0.304 J.
(b) As the spring is being compressed, the work done by the force of spring Ws will be,
Ws = - k (-x)2
To obtain the work done by the force of spring Ws, substitute 2.52 N/cm for k, and -11.8 cm for x in the equation Ws = - k (-x)2,
Ws = - k (-x)2
= - (2.52 N/cm)(-11.8 cm)2
= - (2.52 N/cm) (100 cm/1 m)(-11.8 cm (10-2 m/1 cm))2
= (-1.75 N.m) (1 J/1 N.m)
= -1.75 J
Thus the work done by the force of spring Ws would be -1.75 J.
(c) The kinetic energy K just before hitting the block would be,
K = 1.75 J-0.304 J
= 1.45 J
To obtain the speed v of the block just before it hit the spring, substitute 1.45 J for K and 263-g for m in the equation v=2K/m,
v=2K/m
= 2(1.45 J)/(263 g)
=2(1.45 J)/(263 g (10-3 kg/1 g))
=2(1.45 J)/(0.263 kg)
=2(1.45 J) (1 kg.m2/s2/1 J)/(0.263 kg)
=3.32 m/s
Thus the speed v of the block would be 3.32 m/s.
(d) As the initial speed of the block is doubled, the initial kinetic energy term will be quadruples to 5.78 J.
The compression will then be given by,
-5.78 J= - (252 N/m)y2-(0.263 kg)(9.81 m/s2)y
(252 N/m)y2+(0.263 kg)(9.81 m/s2)y -5.78 J=0

(252 N/m)y2+(5.16 kg.m/s2)y-11.56 J =0


The above equation is a quadratic equation. The solution of the above equation will be,
y = [[-5.16(5.16)2-4(252)(-11.56)]/2(252)] m
= 0.225 m
From the above observation we conclude that, the maximum compression of the spring would be 0.225 m.
Problem 6:-

Suppose that your car averages 30 mi/gal of gasoline. (a) How far could you travel on 1 kW.h of energy consumed? (b) If you are
driving at 55 mi/h, at what rate are you expending energy? The heat of combustion of gasoline is 140 MJ/gal.
Concept:-

Power (P) is equal to the energy (W) divided by time (t).


P = W/t
So the energy W will be,
W = (P) (t)
So the time (rate) t will be equal to,
t = W/P
Solution:-

(a) First we have to convert kilowatt-hours to joules.


1 kW.h = (1 kW) (103 W/1 kW) (1 h) (60 min/1 h) (60 s/1 min)
= (3.6106 W.s) (1 J/1 W.s)
= 3.6106 J
The car gets 30 mi/gal, and one gallon of gas produces 140 MJ of energy.
The gas required to produce 3.6106 J will be,
3.6106 J (1 gal/140106 J) = 0.026 gal.
Thus the distance traveled on this much gasoline will be,
(0.026 gal) (30 mi/1 gal) = 0.78 mi
From the above observation we conclude that, you could travel 0.78 mi on 1 kW.h of energy consumed.
(b) As you are driving at 55 mi/h, therefore the time t to travel 0.78 mi will be,
t = (0.78 mi) (1 h/55 mi)
= 0.014 h
= 51 s
To obtain the rate of energy expenditure (P), substitute 3.6106 J for energy W and 51 s for time t in the equation P = W/t,

P = W/t
= (3.6106 J)/(51 s)
= (71, 000 J/s) (1 W/(1J/s))
= 71,000 W
From the above observation we conclude that, the rate of energy expenditure (P) would be 71,000 W.
Problem 7:-

A particle is projected horizontally along the interior of a frictionless hemispherical bowl of radius r, which is kept at rest in below
figure. We wish to find the initial speed v0 required for the particle to just reach the top of the bowl. Find v 0 as a function of 0, the
initial angular position of the particle.
Concept:-

In an isolated system in which only conservative forces act, the total mechanical energy remains constant. That is, the initial value of
the total mechanical energy (Ki +Ui) is equal to the final value (Kf +Uf). Here Ki is the initial kinetic energy, Ui is the initial potential
energy, Kf is the final

kinetic energy and Uf is the final potential energy.


So, Kf +Uf = Ki +Ui
mvf2 + mgyf = mvi2 + mgyi
Here m is the mas of the body, vi is the initial velocity, vf is the final velocity, yi is the initial height and yf is the final height and g is
the acceleration due to gravity.
Solution:Initially the particle has the kinetic energy Ki (=1/2 mv02) and potential energy is Ui (= mgy). Finally the particle has the kinetic
energy at the top of the bowl is zero (Kf=0) and the potential energy is also zero (Uf = 0) at rest position, since we are taking
horizontal axis as our reference point.
So, Ui = 0
Thus, Ki +Ui = Kf +Uf
= 0+0
=0
1/2 mv02+ mgy =0

So, v0 = -2gy
Here y is the distance beneath the rim and applying geometry it will be equal to r cos 0. Negative sign signifies the downward
direction.
To find out initial speed v0 of the particle, substitute r cos 0 for y in the equation v0 = -2gy,
v0 = -2gy
= -2g(r cos 0)
= 2g(r cos 0)
From the above observation we conclude that, out initial speed v0 of the particle would be 2g(r cos 0).
Problem 8:A 1 kg block collides with a horizontal light spring of force constant 2 N/m. The block compresses the spring m from rest position. Assuming that
the coefficient of kinetic friction between the block and the horizontal surface is 0.25. What was the speed of the block at the instant of
collision.
Solution:When the block compresses the spring let x be the amount of compression.
i.e. x = 4m
Let v be the velocity of the lock when it collides with the spring.
Loss in K.E. of the block = gain in elastic potential energy + work done against friction
(1/2)mv2 - 0 = (1/2)kx2 + mgx,
(1/2)mv2 = (1/2) 2 42 + 0.25 1 9.8 4,
v2 = 2 42 + 0.25 1 9.8 4 2 = 51.6,
v = 51.6 = 7.2m/sec
From the above observation, we conclude that, the speed of the block at the instant of collision would be 7.2 m/sec.

Gravitation
One of the most profound mysteries of nature is - why objects fall down?
This was first solved by Sir Isaac Newton in the seventeenth century - it is known as "the law of gravitation". Gravitation is one of the
four fundamental forces of nature. If acts between all material bodies in the universe - it is universal.
This chapter deals with the universal law of gravitation, Gravitational field, Gravitational potential energy and gravitational potential,
Acceleration due to gravity, Kepler's law, Motion of planets and satellites in circular orbital and elliptical orbit in great detail with
solved examples.
"Gravitation" is one of the easiest and important chapters of Mechanics in the Physics syllabus of IIT JEE, AIEEE and other
engineering examinations. These laws are not new to any aspirant as he is reading a number of stories about apple falling on the
head of Sir Issac Newton. The examples based on this are very easy and can be seen even in day to day life.
The chapter is important not only because it fetches 3-4 questions in most of the engineering examination but also because it is
prerequisite to the other topics of Physics like Heat, Electro-magnetism and Modern Physics.
Newtons Law of Gravitation:Statement:-

Every particle of matter in this universe attracts every other particle with a forcer which varies directly as the product of
masses of two particles and inversely as the square of the distance between them.
The force of attraction between any two bodies in the universe is known as the force of gravitation. This force is mutual and acts
along the line joining the centers of two bodies.
Consider two bodies of masses m1 and m2 as shown in below figure. Let r be the distance between their centers. Let F be the
magnitude of the force of attraction

between them. According to the law of gravitation,

(a) F m1m2
(b) F1/r2
Combining the above two factors, we get,
Fm1m2/r2

or
F = G [m1m2/r2]
Here G is a constant of proportionality. This constant has the same value everywhere and is known as universal constant of
gravitation or gravitational constant or constant of universal gravitation.

Definition of G:-

Let m1 = m2 = 1 unit r = 1 unit


Then, from equation (3), we get,
F = G (1 1/12) = G
or G = F
Thus, the gravitational constant may be defined as the magnitude of force of attraction between two bodies each of unit mass and
separated by a unit distance from each other.
Units and value of G:-

(a) C.G.S System:Unit:- dyne cm2 g-2


Value:- 6.67 10 -8 dyne-cm2 gram-2
(b) SI System:Unit:- N m2 kg-2
Value:- 6.67 10 -11 N m2 kg-2
Dimensional formula of G:G = Fr2/Mm =[MLT-2][L2]/[M2] = [M-1L3T-2]
Derivation of Newtons law of gravitation using Keplers third law and vice-versa:-

Any of these two laws can be derived by using the second one.
(a) Derivation of Newtons law of gravitation using Keplers law:-

By using some approximation that the orbit of a planet be assumed to be circular instead of elliptical, we can deduce Newtons law
of gravitation from Keplers law.
Consider two planets A and B having masses M1 and M2 revolving around the sun, in circular orbits, of radii R1 and R2 with angular
velocities 1 and 2 respectively as shown in the below figure. Centripetal forces F1 and F2 acting on them are,
F1 = M1R112
F2 = M2R222
So, F1/F2 = M1R112 / M2R222
= M1R1/M2R2 (2/T1)2 (T2/2)2
Here T1 and T2 are the time periods of revolution of two planets.
F1/F2 = M1R1/M2R2 (T22/T12)
According to Keplers third law, T22/T12 = R23/R13
Substituting for T22 /T12, F1/F2 = M1/M2

R1/R2 R23/R13 = (M1/M2) (R22/R12)

Or, FM/R2
That is force of attraction varies directly as mass of body and inversely as the square of the distance.
or F = K (M/R2) If K = Gm
Here m is the mass of the sun.
F = GMm/R2
Which is Newtons law of gravitation.
(b) Derivation of Keplers third law using Newtons law of gravitation:For simplicity let us assume that the earth revolves round the sun in circular orbit. Let R be the radius of the orbit, T and
time period and velocity of the earth around the sun respectively.
T = 2R/
or, T2 = 42R2/

= 42R3/

... (1)

be the

If M and m be masses of the sun and earth respectively, we have,


mv2/R = G Mm/R2
Thus, v2R = GM

... (2)

Substituting equation (2) in equation (1), we get,


T2 = 42R3/GM = KR3
Here K = 42/GM = a constant
Therefore, T2 R3
This is Keplers third law.

The gravitational force is an attractive force.

The gravitational force between two particles does not depend on the medium.

The gravitational force between two particles is along the straight line joining the particles (called line of centers).

Gravitational Field and Intensity


The space around a body where the gravitational force exerted by it can be experienced by any other particle is
known as the gravitational field of the body. The strength of this gravitational field is referred to as intensity, and it
varies from point to point.
Consider the gravitational field of a particle of mass m located at the origin (O).
Suppose that a test particle of mass m0 is placed at the point P(x, y, z). The force of gravitational attraction
exerted on the test particle is given by,
g

= (GMmo/r2)

where the position vector


r = OP = |
and the unit vector,

|=|
=

= r,
|=r
/r

The intensity of this gravitational field at a point (P) is given by the force per unit mass on a test particle kept at
P, i.e.
=

g/mo

Where

is the gravitational intensity and

is the gravitational force acting on the mass m0. The

gravitational field is, therefore, a vector field.


The gravitational field at P due to a particle of mass m kept at the point O (origin) is given by
=
where
and

g/mo

= {-(Gmmo/r2) } * 1/mo = Gm/r2

= xi + yj + zk represents the position vector of the point P with respect to the source at the origin
=

/ r represents the unit vector along the radial direction.

The superposition principle extends to gravitational field (intensities) as well:


=
where

1,

2,....

+....+

are the gravitational field intensities at a point due to particle 1, 2, ......, n respectively.

For a continuously distributed mass, the formula changes to

=d , where d

gravitational field intensity due to

an elementary mass dm.


The gravitational field of a ring on its axis
Let us consider a ring of mass M in the plane perpendicular to the plane of the paper. We want to find the
gravitational field on its axis at a distance x.
Consider a differential length of the ring of mass dm.
dE = Fdm/z2

The Y-components of the fields due to diametrically opposite elements cancel each other. Thus, the X-components
add up.
E = Gdm/z2 cos = Gcos/z2 dm = GMcos/z2 = GMx/(a2+x2)3/2
Field due to a uniform thin spherical shell
Consider a thin spherical shell of radius 'a', mass M and of negligible thickness. Out of the spherical shell we
consider a small ring of thickness (R d). The shaded ring has mass dm = (M/2) sin d. The field at P due to
thing ring is
dE = Gdm/z2 cos = GM/2 (sin d cos/z2)

From OAP,
z2 = a2 + r2 - 2ar cos

or

2z dz = 2ar sin d

or

sin d = z dz/ar

Also, from OAP,


a2 = z2 + r2 - 2zr . cos ; cos = (z2+r2-a2)/zr
Thus, dE = GM/(4ar2)[1-(a2-r2)/z2]dz
or

dE = GM/(4ar2)[z+(a2-r2)/z2]

Case I:
P is outside the shell, r > a
E = GM/(4ar2 ) [z+(a2-r2)/z2 ](r-a)(r+a) = GM/r2
We see that the shell may be treated as a point particle of the same mass placed at its centre to calculate the
gravitational field at an external point.
Case II:
P is outside the shell, r > a
E = GM/(4ar2)[z+(a2-r2)/z2 ](a-r)(a+r) = 0
We see that the field inside a uniform spherical shell is zero.
Gravitational field outside a solid sphere
The sphere can be thought of as composed of many shells from radius = 0 to radius = a.
The point P is at a distance r from the centre of all these concentric shells.
=>

E = G/r2[M1 + M2 +...]
E = GM/r2

Gravitational field inside a uniform solid sphere of radius 'R'


To find the field at a point P inside the sphere at a distance r < R form the centre, let us consider a sphere of
radius r.

Consider a point P on the surface of the shaded sphere. Since this point is inside the shells having radii larger than
r, they do not contribute to the field at P. Shells that are less than radius 'r', contribute to the gravitational field at
P.
The mass of the sphere of radius r is

..

M' = (M.4/3 r3)/(4/3 R3 ) = (Mr3)/R3


EP = (GM')/r2 = (GM r)/R3

The adjacent graph shows the variation of E due to a solid sphere of radius R with the distance r from its centre.

E=

GM/r2
3

E = (GM/R ) r

(r > R)
(r < R)

This result holds good for the earth if it is assumed to be a uniform solid sphere.

As by definition, g = Fg/m and also E = Fg/m , so g = E, i.e. acceleration due to gravity and gravitational intensity E at a point are
synonymous.
Illustration:
Two concentric shells of masses M1 and M2 are present. Calculate the gravitational force on 'm' due to M1 and M2 at
points P, Q and R.

Solution:
Field at P, EP = 0
=>
F=0
Field at Q, due to M2 will be zero but there will be field due to m1,
EQ = (GM1)/b2
=>

F = (GM1m)/b2

Field at R, is the sum of fields due to M1 and M2, ER = G(M1+M2 )/c2


Acceleration due to Gravity (g)
The Earth attracts a mass m on its surface by a force F given as:
F = GMe m/Re2 ,
where Me is the mass of the Earth and Re its radius.
This force imparts an acceleration to the mass m, which is known as acceleration due to gravity (g).
By Newton's second law, acceleration = Force/ Mass ,
=> g = F/m = GMe/Re2
Illustration: Find the value of g at the surface of Earth?
Radius of Earth = 6.37 106 meter.
Mass of Earth = 6 1024 kg.
Solution:

g = GM/R2
G = 6.67 10-11 Nm2 kg-2
M = 6 1024 kg
R = 6.37 104 m

As

...... (1)

Put all values in (1), we get g = 9.8 m/s2

Ans.

Variation of Acceleration due to Gravity (g)


i)

Due to altitude

Consider a mass m at a height h from the surface of the earth. Now, the force acting on the mass due to gravity is
F=G

, where M is the mass of the earth and R is the radius of the earth.

If the acceleration due to gravity at the given height is g', then mg' = G

=>

g' = G
(Expanding binomially and neglecting the higher order terms).

(ii)

Due to depth:

If a particle of mass m is kept at a depth 'd' form the surface of earth, then gravitational force exerted on the
particle of mass 'm'.
(GM' m)/(R-d)2 = GM(R-d)/R3

where M' = mass of earth within radius of (R - d)


..

M' = M/R3(R-d)3 = (GM/R3) R(1-d/R) = g(1-d/R)

(iii) Due to rotation of the earth:


Consider a body at a point with altitude , on the surface of the earth.
Let R = radius of the earth and = angular velocity of the earth about its own axis.
F.B.D. of the body with respect to an inertial frame is shown in the adjacent figure.

Acceleration of the body with respect to the earth's centre O is (Rcos )2 directed towards the axis of rotation (i.e.
the centripetal acceleration).
From Newton's second law in the radial direction

or
or

mg - Fn = m(R cos ) 2 cos


Fn = m[g - R2 cos2 ]
Fn/m = g' = g - R2 cos2

where g' is the apparent value of the acceleration


At poles, = 90o => g' = g
At the equator, = 0o => g' = g - R2
Figure given below has illustrated the variation of g with the distance of separation from Earth's centre.

Illustration:
Find the value of g at a height equal to the radius of Earth.
Solution:
g = GM/(R+h)2 and g0 = GM/R2 (At the surface of Earth).
g/g0 = R2/(R+h)2 = R2/4R2 .h = R
=> g = g0/4 = 9.8/4 = 2.45 m/s2.
Caution:

Ans.

Here h = R, so we cannot apply g' = g(1-2h/Re)

Illustration:
At what angular velocity Earth should rotate, for the weight of an object at the equator to be zero? What would be
the duration of a day in this case?
Radius of Earth = 6.4 106 m
g0 = 9.8 m/s2
Solution:
For the weight to be zero, the value of g should be zero. That is
Here,
g' = g0 - Re2 = 0
or = (g0/Re ) = (9.8/(6.4106 )) = 1.2 10-3 rad/s
The duration of one day will be equal to the time period of rotation
T = 2/ = 2/1.2*10-3 sec.
Orbital Velocity
When a satellite revolves in an orbit around a planet, it requires centripetal force to do so. This centripetal force is
provided by the gravitational force between the planet and the satellite. If the mass of the satellite is m and that of
the planet is M, then the gravitational force between them at a height h above the surface of the planet is

F = GMm/(R+h)2', where R is the radius of the earth.


If the speed of the satellite in its orbit is (R + h), then the required centripetal force is mv 2/(R+h).
..

mv2/(R+h) = (G Mm)/(R+h)2

or

v = (GM/(R+h))=(GM/R(1+h/R)) = (gR/(1+h/R))

If the height is very small compared to the radius of the earth, then
v = gR
If the time period of the satellite is 24 hrs rotating in the same sense as the rotation of the earth and if the plane of
the orbit is at right angle to the polar axis of the planet (earth), then the satellite will always be above a certain
place of the earth.
This kind of a satellite is called geostationary satellite.
Illustration:
An artificial satellite of mass 100 kg is in circular orbit at 500 km above the earth's surface. Take the radius of the
earth as 6.5 106 m.
(a)

Find the acceleration due to gravity at any point along the satellite path.

(b)

What is the centripetal acceleration of the satellite?

Solution:
Here, h = 500 km = 0.5 106 m
R = 6.5 106 m
r = R + h = 6.5 106 + 0.5 106 = 7.0 106 m
(a)

g' = g(R/R+h)2 = 9.8((6.5 * 106)/(7.0 * 106)) = 8.45 m/s2

(b)

Centripetal force on the satellite, F = mv2/R

..

Centripetal acceleration, a = F/m = v2/r = ((gR2/r))2/r


= (gR2)/r2 =g R2/(R+h)2 = 8.45 m/s2

Gravitational Potential Energy

When two or more bodies interact with each other, some work has to be done in assembling them together in their respective
places.
Total work done in assembling bodies together, in their respective places, is called gravitational potential energy of the system.
Potential energy of a system of two masses is defined as the amount of work done in bringing these two masses from
infinity to their respective places.

In the right side figure, as the bag moves upwards, kinetic energy decreases, and gravitational energy increases. At the highest
point (B) Kinetic energy is zero, and Gravitational potential energy is highest.
As the bag did not get to, it start falling from point B downwards due to gravity. It starts falling slowly (kinetic energy is low) and then
speeds

At

point

up

A,

bag

is

at

full

speed

and

kinetic

energy is

downwards.

highest,

whiles

gravitational

energy

is

nearly

lost.

When the bag of gold coins hits the ground, kinetic energy is converted into heat and sound by the impact.
It is worth noting that the higher the gravitational energy of an object moving downwards, the lower the kinetic energy, and the lower
the kinetic energy of an object moving upwards, the higher its gravitational energy.
Gravitational potential energy of a system:Change in gravitational potential energy of a system is defined as the -ve of the work done by the gravitational force as the
configuration of the system is changed.

Uf - Ui = Wgr
Change in gravitational potential energy of two point masses m1 and m2 as their separation is changed from r1 to r2 is given by
U(r2) - U(r1) = Gm1m2 [1/r1 - 1/r2]
If, at infinite separation, gravitational potential energy is assumed to be zero, then the gravitational potential energy of the above two
point mass system at separation r,

The below figure shows that the energy of the dart/gun system is initially present in the form of the elastic potential energy (PEs)
and gravitational potential energy (PEg). The springs of the dart gun are compressed which accounts for the elastic potential
energy. Furthermore, the dart is initially elevated at a height of 1-meter above the ground which accounts for the gravitational
potential energy. The presence of these two initial forms of energy are shown by the PEg and PEs bars of the bar chart. Once
projected, the dart no longer has elastic potential energy since the springs of the dart are no longer compressed. However, the dart
does have a large amount of kinetic energy (energy of motion) since it is now moving at a high speed as it leaves the dart. The dart
also has gravitational potential energy since it is still elevated to some height above the ground. As the dart ascends towards its
peak, it is continuously slowing down under the influence of the downward force of gravity. During this ascent, there is a
transformation of the mechanical energy from the form of kinetic energy (energy of motion) to gravitational potential energy (the
stored energy of vertical position). At the peak, there is only a small amount of kinetic energy (the dart still has a horizontal motion)
and a large amount of gravitational potential energy (the dart is at its highest vertical position). Finally, as the dart descends to the
ground, the force of gravity speeds it up. As it falls, there is an increase in kinetic energy (due to the gain in speed) and a decrease
in gravitational potential energy (due to a loss in vertical position).

Gravitational Potential:Gravitational field around a material body can be described not only by gravitational intensity vector
but also
by a scalar function, the gravitational potential V. The gravitational at any point may be defined as the potential
energy per unit mass of a test mass placed at that point.
V = U/m (where U is the gravitational potential energy of the test mass m).
Thus, if the reference point is taken at infinite distance, the potential at a point in the gravitational field is equal to amount of work
done by the external agent per unit mass in bringing a test mass from infinite distance to that point. The expression for the potential
is given by

With the above definition, the gravitational potential due to a point mass M at a distance r from it is

Potential is a scalar quantity. Therefore, at a point in the gravitational field of a number of material particles, the resultant potential is
the arithmetic sum of the potentials due to all the particles at that point. If masses m1, m2, ......, mn are at distances r1, r2, r3, ......,
rn then potential at the given point is
V = -G(m1/r1 + m2/r2 + m3/r3 +..........)
The field and the potential are related as, E = -dV/dr
Gravitational potential due to a shell
(i) at a point outside the shell is: -GM/r (r>R)
(ii) at a point on the surface of the shell is: -GM/R
(iii) at a point inside the shell is: -GM/R
Gravitational potential (V) due to a uniform solid sphere
(i) Outside of the sphere at a distance r form the center, V = -GM/r.
(ii) Inside the sphere at a distance r from the center, V = -GM/R3 (R2/2 - r2/6)
Relation between Gravitational Potential and The Gravitational Field Strength
Gravitational field strength at a point is given by
E = GM/r2

We can also describe the gravitational field of a body by a scalar function called the potential. Let us begin with a test particle of
mass m0 at an infinite separation from the body and move the test particle toward the body until their separation is r, where the
potential energy is U(r). Now the gravitational potential V at that point is defined as V(r) = U(r)/m0.
As we can calculate the radial component of gravitational force
from U(r) according to the relation
F = -dU/dr => m0E = -m0 dv(r)/dr
Hence, we can find the radial component of the gravitational field intensity from V(r)
According to the relation, E = -dV/dr.

Escape Velocity
Escape velocity on the surface of earth is the minimum velocity given to a body to make it free from the gravitational field,
i.e. it can reach an infinite distance from the earth.
Let ve be the escape velocity of the body on the surface of earth and the mass of the body to be projected be m. Now, conserving
energy at the surface of the earth and infinity,
1/2 mve2 - GMm/R = 0 => ve = 2GM/R.

Binding Energy
Binding energy of a system of two bodies is the amount of minimum energy needed to separate the bodies to a large distance.
If two particles of masses m1 and m2 are separated by a distance r, then the gravitational potential energy of the system is given by
U = Gm1m2/r
Let T amount of energy is supplied to the system to separate the bodies by a large distance. When the bodies are separated by a
large distance, gravitational potential energy of the system is zero. For minimum T, conserving energy for initial and final positions,
T+U=0
=> T - Gm1m2/r = 0 or T = Gm1m2/r
Hence, binding energy of a system of two particles separated by a distance r is equal to
T = Gm1m2/r, where m1 and m2 are the masses of the particles.
Gravitational Self Energy of a Uniform Sphere

Consider a sphere of radius R and mass M uniformly distributed. Consider a stage of formation at which the radius of the spherical
core is r. Its mass will be 4/3 r3 at that time, where r is the density of the sphere.
Let an additional mass be added so that the radius of the core be increased by dr in the form of a spherical layer over the core. The
mass of this layer will be 4r2dr.
The mutual gravitational potential energy of the above mass and the spherical core of radius r,
dU = (-G(4/3r3)(4r2dr.))/r = 16/3 22Gr4dr
Hence, total energy evolved in the formation of the spherical body of radius R i.e., self energy.

Gravitational potential at any point is a scalar quantity.

Its unit, in C.G.S. system, is erg g-1 and in S.I. it is J kg-1.

With an increase in r, it becomes less negative i.e., it increases as we move away from the source of gravitational fleld.

Maximum value of gravitational potential is zero and this happens only at infinite distance from the source.

Problem 1:What is the gravitational Potential energy of the Moon-Earth system, relative to the potential energy at infinite separation?
Solution:M = Mass of Earth = 5.98 1024 kg
m = mass of Moon = 7.36 = 1022 k
d = mean separation between Earth and moon = 3.82 108 m.
So, gravitational potential energy of the Moon-Earth system (U)
= -GMm/d
Put all values U = -7.68 1028 J
______________________________________________________________________________________
Problem 2:A projectile is fired vertically from the Earth's surface with an initial velocity of 10 km/s. Neglecting atmospheric retardation, how far
above the surface of the Earth would it go? Take the earth's radius as 6400 km.
Solution:Let the projectile go up to a height h. Then the law of conservation of mechanical energy gives
1/2 mv2 - GMm/R = -GMm/R+h
h = v2R2/2GM-v2R
= v2R2/2gR2-v2R
where g is the acceleration due to gravity on the surface of the Earth.
= ((104)2 (6.4106)2)/(2(9.8)(6.4106)2-(104)2 (6.4106))
_____________________________________________________________________________________
Problem 3:-

Two spherical bodies of masses 2M and M and of radii 3R and R, respectively, are held at a distance 16R from each other in free
space. When they are released, they start approaching each other due to the gravitational force of attraction. Then, find:
(a)

the ratio of their acceleration during their motion.

(b)

their velocities at the time of impact.

Solution:(a) Due to the mutual attraction, the masses attract each other.
If the acceleration are a1 and a2, the net external force on the system = 0
=>

cm

= 0 => m1a1 - m2a2 = 0

or a1/a2 = m2/m1 =

(b) Taking both the bodies as a system, from conserving momentum of the system,
m1v1 - m2v2 = 0 => m1/m2 = v2/v1 = 2
Now, conserving the total mechanical energy, we have
1/2 (2M) V12 + 1/2MV22 - G(2m)M/4R = - G(2m)M/16R and solving it we get.
v1 = GM/8R and v2 = 2GM/8R
Note:

The velocities and acceleration are w.r.t. the inertial reference frame (i.e. the center of mass of the system).

Problem 4:At a point above the surface of earth, the gravitational potential is -5.12 107J/kg and the acceleration due to gravity is 6.4 m/s2.
Assuming the mean radius of the earth to be 6400 km, calculate the height of this point above the earth's surface.
Solution:Let r be the distance of the given point from the centre of the earth. Then,
Gravitational potential = -GM/r = -5.12 107 J/kg

...... (1)

and acceleration due to gravity,


g = GM/r2 = 6.4 m/s2
Dividing (1) by (2), we get
r = 5.12*107/6.4 = 8 106 m = 8000km

...... (2)

Thus, height of the point from earth's surface = r - R = 8000 - 6400 = 1600 km
______________________________________________________________________________________
Problem 5:The mass of Jupiter is 318 times that of earth, and its radius is 11.2 times the earth's radius. Estimate the escape velocity of a body
from Jupiter's surface. [Given: The escape velocity from the earth's surface is 11.2 km/s.]

Solution:Hence, MJ = 318 Me; RJ = 11.2 Re; ve = 11.2 km/s


We know, vJ = 2GMj/Rj and ve = 2GMe/Re
so, vJ/ve = (Mj/Me Re/Rj)
=> vJ = ve (Mj/Me Re/Rj)
vJ = 11.2{318Me/Me Re/11.2Re}1/2 = 11.2(318/11.2)1/2 = 59.7 km/s
___________________________________________________________________________________
Problem 6:Find the escape speed from a point at a height of R/2 above the surface of earth. Assuming mass of earth as M and its radius as R.
Solution:Conserving mechanical energy of a point mass m which is to escape, and earth system we have,
-GMm/(R+R/2) + 1/2mv2 = 0
=> v = (4GM/3R)
________________________________________________________________________________________
Problem 7:The escape velocity of a body on the surface of the Earth is 11.2 km/s. A body is projected away with twice this speed. What is the
speed of the body at infinity? Ignore the presence of other heavenly bodies.
Solution:If v is the velocity of projected and v' is the velocity at infinity, then we have by energy conservation principle.
mv2 - GMm/R = 1/2mv'2 + 0
Here v = 2ve
Thus, (1/2) . 4ve2 - GM/R = 1/2v'2
=> 2ve2 - GM/R = 1/2v'2
Now ve = 2GM/R
=> 2ve2 - v2e/2 = 1/2v'2
or, v'2 = 3 ve2
or, v' = 3 ve = 3 11.2 km/s = 19.4 km/s.

Satellites and Planetary Motion

It was Copernicus who, first of all, introduced the idea that the central body of our planetary system was Sun rather than Earth.
Kepler later on confirmed by putting it on a solid mathematical back ground. Kepler announced two of his laws in 1609 and the third
one 10 years later. The laws can be stated as below:
(a) Keplers first law (law of elliptical orbit):- A planet moves round the sun in an elliptical orbit with sun situated at one of
its foci.
(b) Keplers second law (law of areal velocities):- A planet moves round the sun in such a way that its areal velocity is
constant.
(c) Keplers third law (law of time period):- A planet moves round the sun in such a way that the square of its period is
proportional to the cube of semi major axis of its elliptical orbit.
T2 r 3
Here R is the radius of orbit.
T2 = (42/GM) r 3
Earth and its Satellite:Consider a satellite of mass m revolving in a circular orbit around the Earth, which is located at the centre of its orbit. If the satellite
is at a height h above the Earth's surface, the radius of its orbit r = Re + h, where Re is the radius of the Earth. The gravitational
force between Me & m provides the centripetal force necessary for circular motion, i.e.,
GMem/(Re+h)2 = mv2/(Re+h)
Or v2 = GMe/(Re+h)
or
v = GMe/(Re+h)
Hence orbital velocity depends on the height of the satellite above Earth's surface. Time period T of the satellite is the time taken to
complete one revolution.
Therefore, T = 2r/v = 2(Re + h)(Re+h)/GMe
or T2 = 42(Re+h)3/GMe where r = Re + h
If time period of a satellite is 24 hrs. Then,
r = [GMeT2/42]1/3 = 42400 km and h = 36000 km.
This gives the height of a satellite above the Earth's surface whose time period is same as that of Earth's. Such a satellite appears
to be stationary when observed from the Earth's surface and is hence known as Geostationary satellite.
For a satellite very close to the surface of Earth i.e. h << Re then
r Re
vorbital = GMe/Re = gRe

Simulation for Kepler Laws of Motion:Satellite in circular orbit:For different velocities, the trajectory of the satellite would be different. Let us consider these cases.
If v is the velocity given to a satellite and vO represents the velocity of a circular orbit and ve the escape velocity.
i.e. v0 = GMe/(Re+h)

ve = 2GMe/(Re+h)
Where, h is the distance of the satellite from the surface of the Earth.
Note:

When, v < v0, the satellite follows an elliptical path with center of the Earth as the further focus. In this case, if satellite is
projected from near surface of the Earth, it will hit the Earth's surface without completing the orbit.

If v = v0, obviously the satellite follows a circular orbit with center of Earth as the center of the orbit.

If v0 < v < ve, then the satellite follows an elliptical orbit with center of the Earth as the nearer focus.

If v = ve, the satellite escapes the gravitational field of the Earth along a parabolic trajectory.

If v > ve, the satellite escapes the gravitational field of Earth along a hyperbolic trajectory

Question:What should be the energy required to shift a satellite orbiting around the Earth to infinity?
Answer:At infinity the Potential Energy of the satellite would be zero and if we want to supply minimum Energy then its kinetic energy would
also be zero. Let us first find the total Energy of the satellite.
Total Energy = Kinetic Energy + Potential Energy
= 1/2 mv2 + (-GMem/r)
= 1/2 m (GMe/r) - GMem/r
= -GMem/2r
Now, Binding Energy would be equal to - (Total Energy) as it is the energy needed to shift the satellite from its orbit to infinity.
So, the energy required = GMem/2r.
Here, r = Re + h.
Question:If we see a satellite from Earth, how long will it take for one revolution?
Answer:Let us consider a satellite in circular orbit with a time period Ts. The Earth also rotates with the time period Te= 24 hrs. If an observer
on Earth sees this satellite, the angular velocity of the satellite will be

. Hence, the time period of revolution

will seem different from Ts and will be observed as TSE.


Two cases arise for calculation of TSE:(i) If the satellite and Earth are revolving and rotating respectively in the same direction.
Hence, 2/TSE = |2/Ts - 2/Te|
=> TSE = TsTe/|Te - Ts|
(ii) If the satellite and Earth are revolving and rotating respectively in the opposite direction.
Hence,2/TSE = 2/Ts + 2/Te
or TSE = TsTe/|Ts - Te|.

Kepler's Laws- Elliptical Motion of Planets and Satellites

One of the greatest ideas proposed in human history is the fact that the earth is a planet, among the other planets, that orbits the
sun. The precise determination of these planetary orbits was carried out by Jhannes Kepler, using the data compiled by his teacher,
the astronomer Tycho Brahe. Johannes Kelper discovered three empirical laws by using the data on planetary motion.
(a) Each planet moves in an elliptical orbit, with the sun at one foci of the ellipse.
(b) A line from the sun to a given planet sweeps out equal areas in equal intervals of time.
(c) The square of the periods of the planets are proportional to cube of their mean distance (or semi-major axis) from the sun.
These laws go by the name 'Kepler's laws of planetary motion'. It was in order to explain the origin of these laws, among other
phenomena, that Newton proposed the theory of gravitation.
In our discussion, we are not going to derive the complete laws of planetary motion from Newton's law of gravitation. Since most of
the planets actually revolve in near circular orbits, we're going to assume that the planets revolve in circular orbits.
Consider a planet of mass m rotating around the sun (mass M >> m) in a circular orbit of radius r with velocity v. Then, by applying
Newton's law of gravitation and the second law of motion, we can write
Gravitational force = mass centripetal acceleration

i.e. GMm/r2 = m(v2/r)


or, v2 = GMm/r

... (1)
... (2)

As the moment of the gravitational force about S is zero, the angular momentum of the planet about the sun remains constant. This
is the meaning of Kepler's 2nd law of motion, as will be shown later.
The time period of rotation, T, of the planet around the sun is given by,
T = 2r/v = -2r/GM/r = 2/GM r3/2
... (3)
Squaring both sides,
T2 = (42/GM)r3
... (4)

which is Kepler's 3rd law of motion.


Note:

The constant of proportionality in the above equation depends only on the mass of the sun (M) but not on the mass of

the planet.

Kepler's Laws are also valid for the motion satellites around the earth.
Kepler's Second Law:Consider a planet P that moves in an elliptical orbit around the sun, and let P and P' be the positions of the planet at time t and t + t
(where t is a very small time interval). If the angular displacement of the planet is , then the are swept out by the line joining the
planet and sun (SP) in time t is:

A = area of the section SPP'


= 1/2 r2.; where r = the length SP.
The area velocity, vA = A/t = 1/2r2/t = 1/2r2 = constant .... (5)
In other words,
m (2vA) = constant as well (m = mass of the planet)
Areal velocity = dA/dt = L/2m
... (6)
This is the expression for the angular momentum of the planet,
L = I = mr2
= mr2 (d/dt) perpendicular to the plane of its orbit.

The gravitational force,


= -GMm/r2
So,

is centripetal, and the torque on the planet is zero,

Hence, the angular momentum of the planet does not vary, i.e. the areal velocity of the planet remains constant. At its aphelion
(farthest point from the sun, r is large), the planet moves slowly and at its perihelion (nearest point from the sun, r is small) the
planet moves fastest.

The gravitational force between two particles are equal in magnitude but oppositely directed.

The gravitational force between two particles does not depend on the presence of the other bodies.

The gravitational force between two particles does not depend on the nature of the medium between the particles.

The force of gravitation is expressed in terms of the force between the masses of particles.

The value of gravitational force in the case of small bodies is very small but is appreciable in the case of massive bodies.

Problem 1:Calculate the mass of the Sun from the following data; distance between the Sun and the Earth = 1.49 1011 m, G
= 6.67 10-11 SI units and one year = 365 days.
Solution:Force of attraction between the sun and the earth = GmsmE/dSE2
Considering the orbit of the earth as nearly circular, the centripetal force acting on the earth is mE dSE2.
=> mE dSE 2 = GmsmE/dSE2
mS = dSE2.2/G = 42dSE2/GT2
=(4(3.14)2(1.491011)2)/(6.6710-11(365246060)2)
=1.32 1019 kg.
Problem 2:A Saturn year is 29.5 times the earth year. How far is Saturn from the sun (M) if the earth is 1.5 108 km away
from the sun?
Solution:It is given that
TS = 29.5 Te;
Re = 1.5 1011 m
Now, according to kepler's third law
TS2/Te2 = Rs3/Re3
RS=Re(TS/Te)2/3=1.51011 ((29.5Te)/Te)23=1.431012 m =1.43109 km
Problem 3:A planet of mass m moves along an ellipse around the sun so that its maximum and minimum distances form the sun are equal to R
and r, respectively. Find the angular momentum of this planet relative to the center of the sun.
Solution:According to Kepler's second law, the angular momentum of the planet is constant.
So, mv1R = mv2r or v1R = v2r
If the mass of the Sun is M, conserving total mechanical energy of the system at two given positions we have,

-GMm/R + 1/2 mv12 = -(G M m)/r + 1/2 mv22


So, GM[1/R - 1/r] = v12/2 + v22/2 or GM[(r-R)/Rr] = v12/2 + v22R2/2r2

Thus, v12= (2GM(R-r) r2)/Rr(R2-r2 ) = 2GMr/R(R+r)


Now, angular momentum = mv1R = m2GMR/(R+r)

Fluid Mechanics
A flowing liquid may be regarded as consisting of a number of layers one above the other.Fluid Mechanics is of fundamental
importance as it talks both about the fluids both at rest and in motion.

Steady Flow (Stream Line Flow)


It is the flow in which the velocity of fluid particles crossing a particular point is the same at all the times. Thus, each particle takes
the same path as taken by a previous particle through the point.
Steady, streamline or laminar flow

The flow of fluid is said to be streamline if the velocity at any point in the fluid remains constant with time (in magnitude as well as in
direction) in this case energy needed to drive the fluid is used up in overcoming the viscous force between its layers. All particles
passing through a point in a steady flow follow the same path. The paths are known as streamlines> An example is water slowly
flowing through a pipe.
Turbulent flow
When the motion of a particle varies rapidly in magnitude and direction, the flow is said to be turbulent. In other words, when the
velocity exceeds beyond the critical velocity, the paths and velocities of liquid change continuously and haphazardly. This flow is
called turbulent flow. An example is water coming out of a fountain.
Critical velocity
If in the case of a steady flow of fluids the velocity of flow is gradually increased it is found that the motion remains steady
(streamline or laminar) upto a certain limit. If the velocity of flow crosses a certain limit the fluid particles do not follow the path of the
preceding one and the motion becomes turbulent. The maximum velocity upto which fluid motion remains steady is called critical

velocity. According to Reynold, in case of motion of fluids in narrow tubes, critical velocity depends on the density r and coefficient of
viscosity of the fluid as well as radius of the tube. i.e.
Vc = /r

or Vc = R /r

Here R is a dimensionless constant called Reynold's number. For steady flow R < 2000. For 2000<R<3000, flow is transitional and
for R>3000 flow is turbulent.
Pause: We shall be studying about viscosity later on in the course of this chapter.
Note: In further discussions, we shall be considering only the streamline motion of a non-viscous incompressible fluid.
Enquiry: Will the velocity of a fluid remain constant even if the tube through which it flows has constrictions?

In case of a fluid flowing through a tube of non-uniform cross-section, the product of the area of cross-section, density and the
velocity of flow is same at every point in the tube. This is known as the principle of continuity, which actually means that amount of
mass remains constant. For a fluid, mass flowing in through end B=mass flowing out from C. If the fluid is compressible
A1V11 = A2V22
If the fluid is incompressible then 1 = 2.
Therefore, A1V1 = A2V2 = Constant
Therefore, we can infer that if the cross-sectional area of a tube changes, the velocity of flow will also change.
Illustration
Let us analyze the problem above. Why does the fluid flow through the hole? We know that fluid exerts, force on the walls of
container and walls of container produce equal and opposite reaction. But at the hole there is no wall to produce a reaction. Hence
the volume element near the hole experiences a net downward force and accelerates out of the hole. In terms of pressure, we may
say that pressure inside the container (near the hole) is greater than pressure outside the container and fluid flows towards lower
pressure. Also observe that in the container, velocity of an infinitesimal volume element is less than its velocity when it emerges out.
Similarly, consider a waterfall. Water gains speed as it reaches the bottom. Let us think why it falls down. The answer is its tendency
to reduce its potential energy. Its potential energy gets reduced but kinetic energy is increased. Note that pressure was same at
both ends i.e. atmospheric pressure. (This can also be explained by Newton's laws of motion)

Line of Flow

It is the path taken by a particle in flowing liquid. In case of a steady flow, it is called streamline. Consider an area S in a fluid in
steady flow. Draw streamlines from all the points on the periphery of S. These streamlines enclose a tube, on which S in a crosssection.
No fluid enters or leaves across the surface of this tube.
Equation of Continuity

In a time Dt, the volume of liquid entering the tube of flow in a steady flow is (A1v1 t). The same volume must flow out as the liquid
is incompressible. The volume flowing out in time t is A2 v2 t.
=> A1 v1 - A2 v2.
Mass flow rate = AV (where is the density of the liquid).

Stokes Law

When a body moves through a fluid, the fluid in contact with the body is dragged with it. This establishes relative motion in fluid
layers near the body, due to which viscous starts operating. The fluid exerts viscous force on the body to oppose its motion. The
magnitude of the viscous force depends on the shape and size of the body, its speed and viscosity of the fluid. As the viscous force
depends on velocity of the body it keeps on increasing with the increase in velocity. If it balances the net downward force then body
falling freely in a viscous medium attains a constant velocity, which is called terminal velocity. If a shape of radius r moves with a
velocity v through a fluid of viscosity , the viscous force opposing the motion of the sphere is

This law is called Stokes law. The sphere dropped in the fluid moves down with a decreasing acceleration until the net force on it
becomes zero. At this point it is said to have attained terminal velocity.
Viscous force = Weight - Upthrust
or, F = 6prvT. = (4/3 r3 )g - (4/3 r3 )g.
Where r is the density of the solid and is the density of the fluid.

Stokes Law states that the speed or velocity with which particles settle out of a liquid medium is dependent on a constant factor and
the radius of the particles. The bigger the particle, the faster it will fall out of suspension.
Enquiry: Can we study fluids in motion?
Motions of smoke from chimney, flow of water in river, flow of any gas in a tube are examples of flow of fluids. This is generally a
complex stream of mechanics. But, if we restrict ourselves to a simpler type of motion, i.e. steady, streamline or laminar flow and not
turbulent flow then we can study the behavior of such fluids.
Illustration (JEE Advanced)

A cubical block of wood of specific gravity 0.5 and chunk of concrete of specific gravity 2.5 are fastened together. The ratio of the
mass of the wood to the mass of concrete, which makes the combination to float with its entire volume submerged under water is
(0.x). Find value of x.
Solution

W = Upthrust
Or (m1+m2)g = [(m1/0.5

w) + (m2/2.5

w)]wg

Or m1+m2 = 2m1 + 0.4 m2


Thus, m1/m2 = 0.6 = 3/5

The velocity of any molecule, at a point, is independent of time.

The layer of liquid in contact with the solid surface is at rest.

The liquid in streamlined flow can be supposed to be in the form of parallel layers one above the other.

Velocity of flow of liquid varies inversely as the area of cross-section of the opening from where the liquid comes out.

In laminar flow, the velocities of molecules lying on different stream lines may be different.

Thermal Physics:Thermal Physics is a vital branch of Physics and covers various topics like
thermodynamics, statistical mechanics, kinetic theory etc. Thermal Physics
deals with diverse branches including key areas like the study of heat,
temperature and heat transfer. The various concepts of thermodynamics are
closely related to thermal energy which is one of the basic forms of energy.
These concepts have far reaching applications. In most of the energy
transformations there is a production of thermal energy in the form of heat.
Thermal physics also forms the basis of various ancient cycles of heat. This is
also the reason behind the invention of steam engine, rotors, shipments and various other revolutionary machines.
We shall discuss some of the topics included under thermal physics in brief as they have been described in detail in the coming
sections. We proceed with the laws of thermodynamics which describe the physical entities like temperature and energy and form
the basis of thermodynamic systems.
Heat:When a cold body A is placed in contact with a hot body B something is transferred from hot body B to the cold body A which
results in a rise in temperature of the cold body. This transference stops when the two have acquired same temperature. This
indicates that only a part of energy of B is transferred to A. This part is called heat. This transfer or transmission may occur in any
of the ways conduction, radiation or convective circulation.Before we discuss the laws, we shall first discuss some of the words
which will be used in these laws:
The part of thermal energy which flows from one body to another due to temperature difference is called heat. By convention, heat
given to a body is taken as positive while that taken out of the body is taken as negative. Heat is a scalar quantity.
Temperature:-

Temperature is defined as the degree of hotness of a body. it is also a scalar quantity. The state of equality in the temperature of
two bodies is known as thermal equilibrium.
Entropy:-

The physical quantity entropy is a measureable property and describes the ability of the system to do work. Entropy of a substance
is, therefore, said to be a measure of the degree of disorder prevailing among its molecules, just as the temperature is a measure of
the degree of hotness of a substance. At the absolute zero of temperature the thermal motion completely disappears so that the
disorder and hence the entropy tends to zero and the molecules of a substance are in perfect order, i.e., well arranged. Higher the
entropy, higher is the disorder. The entropy of any isolated system increases and approaches, more or less rapidly, to the inert state
of maximum entropy.
Mathematically,
dS = dQ/T
Enthalpy:-

Enthalpy is an extensive thermodynamic property and is given the symbol H. Enthalpy is given by the equation,
H = U+PV

Therefore, enthalpy is defined as the sum of the internal energy and the product of pressure and volume.
First law of thermodynamics:The first law of thermodynamics states that, If the quantity of heat supplied to a system is capable of doing work, then the quantity
of heat absorbed by the system is equal to the sum of the increase in the internal energy of the system, and the external work done
by it.
Therefore, first law of thermodynamics signifies that, energy can neither be created nor destroyed, but it can only be transformed
from one form to another.
Thus, dQ= dU+dW
If work is done by the surroundings on the system (as during the compression of a gas), W is taken as positive so that dQ = dU+W.
if however work is done by the system on the surroundings (as during the expansion of a gas), W is taken as negative so that dQ =
dU dW.
Mathematically,

the

first

law

of

thermodynamics

can

be

stated

as

Let us discuss some of the limitations of the first law of thermodynamics:


Limitations to The First Law of Thermodynamics:-

First law of thermodynamics which tells us that heat can be converted into work and viceversa is merely a quantitative statement of
the equivalence of heat and work. It has the following limitations.
(a) It does not tell us whether any particular process can actually occur. According to the first law of thermodynamics heat may flow
from higher temperature to lower temperature nad vice versa. Pratically we know that heat cannot flow from a lower temperature to
a higher one. This is not predicted by the first law.
(b) According to the first law, we could convert the whole (100%) of heat, available to us, into work and vice versa, while pratically
we know that it is not possible to do so. Thus, first law does not restrict our ability to convert heat into work or work into heat.
Above quoted handicaps lead to the formulation of another law of thermodynamics called second law of thermodynamics.
Second Law of Thermodynamics:-

In terms of entropy the second alw of thermodynamics may be stated as follows:


The entropy of an isolated system is fully conserved in every reversible process, i.e. for every reversible process the sum of all
changes in entropy taking place in an isolated system is zero. If the process is not reversible one, then the sum of all changes in
entropy taking place in an isolated system is greater than zero. In general we can say that in every process taking place in an
isolated system the entropy of the system either increases or remains constant.
Furthermore, if an isolated system is in such a state that its entropy is a maximum, any change from that state would necessarily
involve a decrease in entropy and hecnce will not happen. Therefore, the necessary condition for an equilibrium of an isolated
system is that its entropy shall be maximum.
It is to be noted that the above statements are applicable to isolated system only. The more usual forms of the second law of
thermodynamics are as follows.
(a) It is impossible to construct a device which, operating in a cycle has the sole effect of extracting heat from a single reservior and
performing an equivalent amount of work. (Kelvin-Planck Statement)
(b) It is impossible for heat to flow from a cooler body to another hotter body without the aid of any external agency.(Clausius
Statement)
Most of these limitations of the 1st law of thermodynamics have been taken care off in the second law. The second law of
thermodynamics helps us in estimating whether the reaction is actually viable or not and it also tells the direction of heat. The
second

law

also

clarifies

that

it

is

not

possible

to

convert

energy

completely

into

work.

Third Law of Thermodynamics:In accordance to third law of thermodynamics, the heat capacities of all solids tends to zero as the absolute zero of temperature is
approached and that the internal energies and entropies of all substances become equal there, approaching their common value
asymptotically. This simple statement follows neither from the first law (law of conservation of energy) nor from the second law (law
of transmutability of energy) and is thus of the nature of a new law, usually called the third law of thermodynamics.
This theorem is useful in explaining the nature of bodies in neighbourhood of absolute zero temperature. Its importance lies in the
fact that it permits the calculations of absoluted values of entropy and the physical interpretation of thermodynamic propperties such
as Helmholtz nad Gibbs free energies etc. It can be conceived that as the temperature of a system tends to absolute zero, its
entropy tends to a constant value which is indepent of pressure and state of agregation etc. We may put it equal to zero so that the
entropy of every substance becomes normalized in an absolute way.

You might also like